*NURSING > Final Exam Review > Exam (elaborations) NR 511 (NR511Final practice questions) (NR 511 (NR511Final practice questions) (All)

Exam (elaborations) NR 511 (NR511Final practice questions) (NR 511 (NR511Final practice questions)

Document Content and Description Below

Final Exam Practice Questions Question 1. Matthew, age 52, is a chef who just severed 2 of his fingers with a meat cutter. You would recommend that he: 1. Wrap the severed fingers tightly in a dr... y towel for transport to the emergency department with him. 2. Leave the severed fingers at the scene because fingers cannot be reattached. 3. Immediately freeze the severed fingers for reattachment in the near future. 4. Wrap the fingers in a clean, damp cloth; seal them in a plastic bag; and place the bag in an ice water bath. Rationales Option 1: Severed fingers should be wrapped in a clean, damp cloth; sealed in a plastic bag; and placed in an ice water bath. Option 2: Severed fingers can be reattached after 1 to 2 days—or more—if properly stored. Option 3: Severed fingers should be wrapped in a clean, damp cloth; sealed in a plastic bag; and placed in an ice water bath, not frozen or kept at body temperature. Option 4: If a client has severed his fingers, the fingers should be wrapped in a clean, damp cloth; sealed in a plastic bag; placed in an ice water bath; and transported to the emergency room along with the client. The valgus stress test, varus stress test, Lachman test, and thumb sign are all considered standard tests to check the integrity of the ligaments of the knee. Which test would the nurse practitioner choose to assess the anterior cruciate ligament (ACL), which is the most commonly involved structure in severe knee injury? 1. Valgus stress test. 2. Varus stress test. 3. Lachman test. 4. Thumb sign. Rationales Option 1: The valgus stress test assesses stability of the medial collateral ligament (MCL). Option 2: The varus stress test assesses stability of the lateral collateral ligament (LCL). Option 3: The Lachman test assesses the ACL. Option 4: The thumb sign assesses the posterior cruciate ligament (PCL). Lillian, age 70, was told that she has osteoporosis. When she asks you what this is, you respond that osteoporosis: 1. Develops when loss of bone occurs more rapidly than new bone growth. 2. Is a degenerative joint disease characterized by loss of cartilage in certain joints. 3. Is a chronic inflammatory disorder that affects multiple joints. 4. Is a bone disorder that has to do with inadequate mineralization of the bones. Rationales Option 1: Osteoporosis develops when bone resorption occurs more rapidly than bone deposition. Option 2: Osteoarthritis is a degenerative joint disease characterized by degeneration and loss of articular cartilage in synovial joints. Option 3: Rheumatoid arthritis is a chronic, systemic inflammatory disorder characterized by persistent synovitis of multiple joints. Option 4: Osteomalacia is a metabolic bone disorder characterized by inadequate mineralization of the bone matrix, often caused by vitamin D deficiency. Mike, age 42, a golf pro, has had chronic back pain for many years. His workup reveals that it is not the result of a degenerative disk problem. His back “goes out” about twice per year, and he is out of work for about a week each time. Which of the following should the nurse practitioner advise him to do? 1. Consider changing careers to something less physical. 2. Begin a planned exercise program to strengthen back muscles. 3. Make an appointment with a neurosurgeon for a surgical consultation. 4. Start on a daily low-dose narcotic to take away the pain. Rationales Option 1: Suggesting a career change should be considered only in cases of disability or inability to safely continue one’s current employment. Option 2: In this case, Mike may benefit from a regular planned exercise program to strengthen his back muscles and attempt to reduce the probability of future episodes of back pain. Option 3: Surgery is recommended only for clients with low back pain caused by degenerative disk disorders, and then only when severe neurologic involvement has occurred. Surgery benefits only approximately 1% of people with low back problems. Option 4: Narcotic pain medications are not considered first-line treatment for mechanical back pain. Course Topic: Musculoskeletal Problems Concept(s): 21820Cognitive Level: Analysis [Analyzing] Janine, age 69, has class III rheumatoid arthritis. According to the American Rheumatism Association, which of the following describes her ability to function? 1. Adequate for normal activities despite a handicap of discomfort or limited motion of one or more joints. 2. Largely or wholly incapacitated, bedridden, or confined to a wheelchair, permitting little or no self-care. 3. Completely able to carry out all usual duties without handicap. 4. Adequate to perform only a few or none of the duties of usual occupation or self-care. Rationales Option 1: Class II refers to the client whose function is adequate for normal activities despite a handicap of discomfort or limited motion of one or more joints. Option 2: Class IV refers to the client who is largely or wholly incapacitated, bedridden, or confined to a wheelchair, permitting little or no self-care. Option 3: Class I refers to the client who can carry out all usual duties without handicap. Option 4: The American Rheumatism Association has established functional classes I to IV to describe a client’s ability to accomplish activities of daily living. Because Janine is class III, her function would be adequate to perform only a few or none of the duties of usual occupation or self-care. For an adult patient with a knee injury, the nurse practitioner orders a nonsteroidal anti-inflammatory drug (NSAID) to be taken on a routine basis for the next 2 weeks. Patient teaching should include which of the following? 1. “You may take this medication on an empty stomach as long as you eat within two to three hours of taking it.” 2. “If one pill does not seem to help, you can double the dose for subsequent doses.” 3. “If you notice nausea, vomiting, or black or bloody stools, take the next dose with a glass of milk or a full meal.” 4. “If you have additional pain, an occasional acetaminophen (Tylenol) is permitted in between the usual doses of the NSAID.” Rationales Option 1: When teaching clients about NSAIDs, tell them not to take these drugs on an empty stomach but to take them with food or milk. Option 2: Clients should be taught to never take more than the prescribed dose of an NSAID due to the likelihood of increasing the chances of gastrointestinal (GI) and kidney damage. Option 3: When teaching clients about NSAIDs, tell them to stop the medication and call immediately if they notice any nausea, vomiting, coffee-grounds emesis, black stools, or blood in the stool. Option 4: If the client is having additional pain, acetaminophen (Tylenol) may be taken in conjunction with an NSAID because it is not an NSAID and will not potentiate gastric bleeding. Sean, a factory line worker, has osteoarthritis (OA) of the right hand. According to the American College of Rheumatology (ACR), the guidelines for pharmacologic treatment include: 1. Acetaminophen, tramadol, and intra-articular corticosteroid injections. 2. Oral nonsteroidal anti-inflammatory drugs (NSAIDs), tramadol, and articular corticosteroid injections. 3. Acetaminophen, topical capsaicin, and topical nonsteroidal anti-inflammatory drugs (NSAIDs). 4. Topical capsaicin, topical nonsteroidal anti-inflammatory drugs (NSAIDs), and oral NSAIDs. Rationales Option 1: Acetaminophen, oral nonsteroidal anti-inflammatory drugs (NSAIDs), tramadol, and intra-articular corticosteroid injections are recommended for initial management of hip OA. Option 2: Acetaminophen, oral NSAIDs, tramadol, and intra-articular corticosteroid injections are recommended for initial management of hip OA. Option 3: The ACR conditionally recommends acetaminophen, oral NSAIDs, topical NSAIDs, tramadol, and intra-articular corticosteroid injections for OA of the knee. Option 4: Topical capsaicin, topical NSAIDs, oral NSAIDs, and tramadol are recommended by the ACR for the treatment of OA of the hand. Margaret, age 55, presents to you for evaluation of left hand and wrist pain and swelling after a slip and fall on the ice yesterday. On examination, you note tenderness at her “anatomical snuffbox.” You know this probably indicates a(n): 1. Ulnar styloid fracture. 2. Scaphoid fracture. 3. Hamate fracture. 4. Radial head fracture. Rationales Option 1: An ulnar styloid fracture would produce tenderness at the distal ulna. Option 2: There is tenderness over the “anatomical snuffbox” in a scaphoid (aka navicular) fracture, the most common injury of the carpal bones. Poor blood supply puts the scaphoid bone at risk of avascular necrosis; therefore, wrist pain and tenderness in the anatomical snuffbox, even without history of antecedent trauma, warrant a wrist x-ray. Option 3: A fracture of the hook of the hamate is an uncommon injury seen in golfers and players of racket sports that involves pain and tenderness on the ulnar side of the palm. Option 4: A radial head fracture would result in pain at the elbow joint where the radial head lies proximal to the distal humerus. Be sure not to confuse the radial head (proximal end of the radius) with the radial styloid (distal end of the radius at the wrist). Jim, age 22, a stock boy, has an acute episode of low back pain. The nurse practitioner orders a nonsteroidal anti-inflammatory drug (NSAID) and should educate him in which of the following? 1. Maintaining moderate bed rest for 3 to 4 days. 2. Calling the office for narcotic medication if there is no relief with the NSAID after 24 to 48 hours. 3. Beginning lower back strengthening exercises depending on pain tolerance. 4. Wearing a Boston brace at night. Rationales Option 1: Years ago, muscle relaxants and bed rest were the treatments of choice for low back pain. Studies have now shown that resuming normal activity within the limits imposed by the pain has an effect as good as, if not better than, 2 days of bed rest. Option 2: NSAIDs, not narcotics, are generally the first-line medication treatment of low back pain and have no risk of opioid dependency. Option 3: Years ago, muscle relaxants and bed rest were the treatments of choice for low back pain. Studies have now shown that resuming normal activity within the limits imposed by the pain has an effect as good as, if not better than, 2 days of bed rest. The key here is letting pain be your guide. Exercise should begin as soon as possible after the acute injury and be directed at building endurance and stamina, with consideration given to one’s pain tolerance. Option 4: A Boston brace may be used in the treatment of scoliosis. Jessie, age 49, states she thinks she has rheumatoid arthritis. Before any diagnostic tests are ordered, the nurse practitioner completes a physical examination and makes a diagnosis of osteoarthritis rather than rheumatoid arthritis. Which clinical manifestation ruled out rheumatoid arthritis? 1. Fatigue. 2. Affected joints are swollen, cool, and bony hard on palpation. 3. Decreased range of motion. 4. Joint stiffness. Rationales Option 1: Fatigue is common to both diseases. Option 2: In osteoarthritis, the affected joints are swollen, cool, and bony hard on palpation. With rheumatoid arthritis, the affected joints are red, hot, and swollen and boggy and tender on palpation. Option 3: Decreased range of motion is common to both diseases. Option 4: Joint stiffness is common to both diseases. Greg, age 26, runs marathons and frequently complains of painful contractions of his calf muscles after running. You attribute this to: 1. Hypercalcemia. 2. Hyponatremia. 3. Heat exhaustion. 4. Dehydration. Rationales Option 1: Hypercalcemia may affect gastrointestinal, renal, and neurologic function. Symptoms may include constipation and polyuria as well as nausea, vomiting, and anorexia at times. Option 2: Painful contractions of muscles after exertion, such as heat cramps, may be related to hyponatremia or other electrolyte imbalances. Usually the gastrocnemius and hamstring muscles are involved. Treatment of heat cramps includes passive muscle stretching, cessation of activities, transfer to a cooler environment, and drinking cool liquids. Sports drinks that contain electrolytes, such as Gatorade, may be beneficial. Option 3: Heat exhaustion is a more serious condition, with symptoms ranging from nausea, vomiting, headache, loss of appetite, and dizziness to irritability, tachycardia, and hyperventilation. Option 4: Greg’s painful muscle contractions are attributed to his hyponatremia, but this would be a good second choice. Course Topic: Musculoskeletal Problems Concept(s): 21820Cognitive Level: Analysis [Analyzing] The nurse practitioner is considering a diagnosis of calcium pyrophosphate deposition disease (CPPD), or pseudogout, in a 72-year-old man who presents with complaints of pain and stiffness in his wrists and knees. The most useful diagnostic tests to assist in confirming this diagnosis would be: 1. Synovial fluid analysis and x-ray. 2. Bacterial cultures. 3. Bone scan and magnetic resonance imaging (MRI). 4. Anticitrullinated protein antibodies (ACPA) and rheumatoid factor (RF). Rationales Option 1: CPPD (pseudogout) may appear clinically similar to gouty arthritis; however, in CPPD crystals form in the cartilage and lead to inflammation. The typical age of onset is later than that of gout, initially presenting in the sixth decade of life or later. Diagnosis is made through synovial fluid analysis and will reveal positive calcium pyrophosphate dihydrate crystals. An x-ray will show radiographic evidence of chondrocalcinosis or calcification in the hyaline cartilage and/or fibrocartilage of the affected joint. Option 2: Bacterial cultures would be warranted to aid in the diagnosis of cellulitis. Option 3: Bone scan and MRI are used to differentiate gouty arthritis from Charcot neuro-osteoarthropathy. Option 4: ACPA and RF assist in the diagnosis of rheumatoid arthritis. Ethan, age 10, jumped off a 2-foot wall, twisting his foot and ankle upon landing. His ankle x-ray demonstrates a fracture of the distal tibia, over the articular surface, through the epiphysis and physis (growth plate). Based on the Salter-Harris classification of growth plate injuries, you know this is a: 1. Salter-Harris II fracture. 2. Salter-Harris III fracture. 3. Salter-Harris IV fracture. 4. Salter-Harris V fracture. Rationales Option 1: A Salter-Harris II fracture travels through the metaphysis and physis. Option 2: The Salter-Harris classification system of growth plate injuries divides most growth plate injuries into 5 categories based on the damage: A Salter-Harris III fracture travels through the epiphysis and physis. A Salter-Harris I fracture travels through the physis. Option 3: A Salter-Harris IV fracture travels through the metaphysis, epiphysis, and physis. Option 4: A Salter-Harris V fracture involves a compression injury of the physis. In assessing a patient, you place the tips of your first 2 fingers in front of each ear and ask the patient to open and close his mouth. Then you drop your fingers into the depressed area over the joint and assess for smooth motion of the mandible. With this action, you are checking for: 1. Maxillomandibular integrity. 2. Well-positioned permanent teeth or well-fitting dentures. 3. Temporomandibular joint syndrome. 4. Mastoid inflammation. Rationales Option 1: Maxillomandibular integrity indicates a properly functioning temporomandibular joint (TMJ). Option 2: In most cases, well-positioned permanent teeth and well-fitting dentures prevent the development of temporomandibular joint (TMJ) syndrome. Option 3: In assessing your client, place the tips of your first 2 fingers in front of each ear and ask him to open and close his mouth. Then drop your fingers into the depressed area over the temporomandibular joint (TMJ) and check for smooth motion of the mandible. With this action, you are assessing for TMJ syndrome. Clicking or popping noises, decreased range of motion, pain, or swelling may indicate TMJ syndrome. However, an audible and palpable snap or click does occur in many normal people as they open their mouths. In rare cases, this may indicate osteoarthritis. Option 4: Mastoid inflammation is manifested behind, not in front of, the ear. Sandra, a computer programmer, has just been given a new diagnosis of carpal tunnel syndrome. The nurse practitioner’s next step is to: 1. Refer her to a hand surgeon. 2. Take a more complete history. 3. Try neutral position wrist splinting and order an oral nonsteroidal anti-inflammatory drug (NSAID). 4. Order nerve conduction studies and electromyography (EMG). Rationales Option 1: For refractory cases, median nerve decompression may be accomplished with surgery, but complete recovery is not possible if atrophy is pronounced. Option 2: Taking a more complete history is not essential at this point because a diagnosis has already been made. Option 3: For the client who has just been given a diagnosis of carpal tunnel syndrome, your next step is to try neutral position wrist splinting and order an oral NSAID. For symptoms of less than 10 months’ duration, conservative treatment should be tried first. Option 4: Nerve conduction studies and EMG confirm focal median nerve conduction delay within the carpal canal and also provide information about disease severity. Alexander, age 12, sprained his ankle playing ice hockey. He is confused as to whether he should apply heat or cold. What should the nurse practitioner tell him? 1. “Use continuous heat for the first 12 hours and then use heat or cold to your own preference.” 2. “Use continuous cold for the first 12 hours and then use heat or cold to your own preference.” 3. “Apply cold for 20 minutes, then remove it for 30 to 45 minutes; repeat this for the first 24 to 48 hours while awake.” 4. “Alternate between cold and heat for 20 minutes each for the first 24 to 48 hours.” Rationales Option 1: Applying heat may increase swelling and subsequently slow recovery. Option 2: Ice has been proven to speed recovery in ankle sprains; however, ice should never be applied continuously because it could hinder proper circulation and cause frostbite. Therefore, always recommend protective padding between the ice and the skin. Option 3: Tell a client who has sprained his ankle to apply cold for 20 minutes, then remove it for 30 to 45 minutes, and repeat that procedure for the first 24 to 48 hours while awake. Cold will cause vasoconstriction and decrease edema, preventing any further bleeding into the tissues. After any sprain, use the principles of RICE: rest, ice, compression, and elevation. Option 4: Applying heat may increase swelling and subsequently slow recovery. Daniel, age 45, is of Northern European ancestry and has a dysfunctional and disfiguring condition affecting the palmar tissue under the skin of the distal palm and fourth and fifth fingers. What do you suspect? 1. Hallux valgus. 2. De Quervain tenosynovitis. 3. Dupuytren contracture. 4. Hallux rigidus. Rationales Option 1: Hallux valgus, commonly referred to as a bunion, is an osseous deformity of the metatarsophalangeal (MTP) joint of the great toe that involves medial deviation of the toe. Option 2: De Quervain tenosynovitis is a painful inflammation of tendons around the thumb. Option 3: Dupuytren contracture affects the palmar tissue under the skin of the distal palm and fingers—most often the fourth and fifth fingers, but also the thumb-index finger web space. It is progressive and results in a flexion contracture, though it does not affect the flexor tendons. Occurring most frequently in males between the ages of 40 and 60, it is common among people of Northern European ancestry. It is dysfunctional and disfiguring. Although it is not actually painful, it may be tender. Surgery is recommended when the inability to straighten the fingers limits the client’s hand function. Option 4: Hallux rigidus is a common type of arthritis that affects the base of the great toe at the metatarsophalangeal (MTP) joint, causing stiffness and decreased movement of the great toe. Course Topic: Musculoskeletal Problems Concept(s): 21820Cognitive Level: Analysis [Analyzing] Paul has a malignant fibrosarcoma of the femur. He recently had surgery and is now on radiation therapy. You want to order a test to determine the extent of the tumor invasion of the surrounding tissues and the response of the bone tumor to the radiation. Which of the following tests should you order? 1. An x-ray. 2. A magnetic resonance imaging (MRI) scan. 3. A computed tomography (CT) scan. 4. A needle biopsy. Rationales Option 1: A conventional x-ray will show the location of the tumor and the extent of bone involvement. Metastatic bone destruction has a characteristic “moth-eaten” pattern in which the growth has a poorly defined margin that cannot be separated from normal bone. Option 2: For Paul, who has a malignant fibrosarcoma of the femur, an MRI scan will determine the extent of the tumor invasion of the surrounding tissues and the response of the bone tumor to the radiation. It will also determine response to chemotherapy and detect recurrent disease. Option 3: A CT scan will evaluate the extent of the tumor invasion of bone, soft tissues, and neurovascular structures. Option 4: A needle biopsy, usually performed at the time of surgery, will determine the type of tumor. Lois, age 52, who has just been given a diagnosis of sarcoidosis, has joint symptoms, including arthralgias and arthritis. Your next plan of action would be to: 1. Order a bone scan. 2. Obtain a tissue biopsy. 3. Begin a course of glucocorticoids. 4. Order daily doses of vitamin B. Rationales Option 1: There is no present need for a bone scan. Option 2: There is no present need for a tissue biopsy. Option 3: Sarcoidosis is the result of an exaggerated immune system response to a class of antigens or self-antigens. Fifty percent of clients experience joint symptoms, including myopathy and polyarthritis; glucocorticoids are prescribed to suppress the immune process, thus relieving symptoms. Option 4: Vitamin B is not routinely ordered, as it has not been shown to impact sarcoidosis. The nurse practitioner (NP) is assessing Maya, a 69-year-old Asian woman, for the first time. When trying to differentiate between scoliosis and kyphosis, the NP recalls that kyphosis involves: 1. Asymmetry of the shoulders, scapulae, and waist creases. 2. A lateral curvature and vertebral rotation on posteroanterior x-rays. 3. One leg appearing shorter than the other. 4. A posterior rounding at the thoracic level. Rationales Option 1: Scoliosis involves asymmetry of the shoulders, scapulae, and waist creases. Option 2: Scoliosis involves a lateral curvature and vertebral rotation on posteroanterior x-rays. Option 3: In a client with scoliosis, the legs may appear uneven in length. Option 4: Kyphosis involves a posterior rounding at the thoracic level and a kyphotic curve of more than 45 degrees on an x-ray. There may be moderate pain with kyphosis. Marsha, age 34, presents with symptoms resembling both fibromyalgia and chronic fatigue syndrome, which have many similarities. Which of the following is more characteristic of fibromyalgia than of chronic fatigue syndrome? 1. Musculoskeletal pain. 2. Difficulty sleeping. 3. Depression. 4. Fatigue. Rationales Option 1: Musculoskeletal pain is not characteristic of chronic fatigue syndrome; rather, it is characteristic of fibromyalgia. The musculoskeletal pain, usually an achy muscle pain that may be localized or involve the entire body, is usually gradual in onset, although the onset may occasionally be sudden, such as after a viral illness. Option 2: Difficulty sleeping occurs with both disorders. Option 3: Depression occurs with both disorders. Option 4: Fatigue is a more significant feature of chronic fatigue syndrome. [Page reference: 1009-1012] Upon assessment, the nurse practitioner notes unilateral back pain of acute onset that increases when standing and bending. A straight leg raise test is negative. The most likely diagnosis is: 1. Herniated nucleus pulposus. 2. Muscle strain. 3. Osteoarthritis. 4. Spondylolisthesis. Rationales Option 1: A herniated nucleus pulposus causes unilateral back pain that is acute and increases when sitting or bending but decreases when standing. The straight leg raise test is positive, and the plain x-ray is negative. Option 2: Muscle strain is usually located in the low back and is unilateral in location. The onset is acute, and pain increases with standing and bending and decreases with sitting. The straight leg raise test and plain x-ray are both negative. Option 3: Back pain related to osteoarthritis is usually bilateral in nature and has an insidious onset. Pain increases when standing and decreases when sitting. A straight leg raise test is negative, and a plain x-ray is positive. Option 4: Spondylolisthesis has an insidious onset, and low back pain increases when standing or bending and decreases when sitting. A straight leg raise test is negative, and a plain x-ray is positive. [Page reference: 800] A 55-year-old patient presents with complaints of paresthesias in the lower lateral arm, thumb, and middle finger. The nerve roots most commonly related to these symptoms are C6 and C7. The most likely diagnosis would be: 1. Brachial plexus neuritis. 2. Cervical radiculopathy. 3. Peripheral polyneuropathy. 4. Thoracic outlet syndrome. Rationales Option 1: Brachial plexus neuritis presents with dysfunction ranging from momentary paresthesias to completely flail extremities and is usually caused by injuries to the C5 and C6 nerve roots. Option 2: Cervical nerve root compression of C6 and C7 causes cervical radiculopathy. Option 3: Peripheral polyneuropathy is associated with distal sensorimotor paresthesia and diminished deep tendon reflexes. Option 4: Thoracic outlet syndrome is caused by compression of the brachial plexus and/or subclavian vessels as they exit the space between the superior shoulder girdle and the first rib. Symptoms are color changes in the hand and sensory changes and weakness in the fourth and fifth fingers. [Page reference: 797] Christian, age 22, is a carpenter who is right hand dominant. He comes to the clinic for follow-up from the emergency department, where he was seen for right forearm pain. He states he was diagnosed with right forearm tendinitis and wants the provider to explain this diagnosis to him. Patient teaching should explain that he has inflammation of one or more tendons, which are: 1. The ropelike bundles of collagen fibrils that connect bone to bone. 2. The collagen fibers that connect muscle to bone. 3. The pouches of synovial fluid that cushion bone and other joint structures. 4. The fibrocartilaginous disks that separate bony surfaces. Rationales Option 1: Ligaments connect bone to bone at the joints. Option 2: Tendons are the collagen fibers that connect muscle to bone. Option 3: Bursae are the pouches of synovial fluid that reduce friction between bones, muscles, and tendons. Option 4: Fibrocartilaginous disks separate bony surfaces, such as those between the vertebrae. [Page reference: 836-839] Steve, age 32, fell off a roof while shingling it. He is complaining of pain in his left hip and leg area. Other than an x-ray, what would make you suspect a fractured pelvis? 1. A clicking sensation when moving the hips. 2. A positive pelvic tilt test. 3. Hematuria. 4. Absence of distal reflexes. Rationales Option 1: A clicking sensation when moving the hips is not diagnostic of a fractured pelvis. Option 2: A positive pelvic tilt test is not diagnostic of a fractured pelvis. Option 3: If a client has a fractured pelvis, a test for hematuria will usually prove positive. A fracture of the pelvis usually results in hypovolemia due to a generally significant associated blood loss. Surrounding blood vessels rupture, which results in a large retroperitoneal hematoma and shock. Pelvic fractures also commonly injure the urinary bladder or urethra. A client with a fracture in several locations of the pelvis may need a pneumatic antishock garment to control the blood loss and stabilize the pelvis. Only x-ray studies will confirm the diagnosis. Option 4: Absence of distal reflexes is not correlated with a fracture of the pelvis. [Page reference: 1203-1204] Beth, age 49, comes in with low back pain. An x-ray of the lumbosacral spine is within normal limits. Which of the following diagnoses do you explore further? 1. Scoliosis. 2. Osteoarthritis. 3. Spinal stenosis. 4. Herniated nucleus pulposus. Rationales Option 1: A plain x-ray will show scoliosis. Option 2: A plain x-ray will show osteoarthritis. Option 3: A plain x-ray will show spinal stenosis. Option 4: A plain x-ray will not show a herniated nucleus pulposus or a muscle strain. It will show spondylolisthesis, scoliosis, osteoarthritis, and spinal stenosis. Note that x-rays of the spine are not indicated in low back pain unless the cause of the pain is thought to be bony in origin or traumatic in nature or there is a need to rule out systemic disease. [Page reference: 824-826] Joyce, age 87, broke her wrist after falling off a curb. She just had a plaster cast applied to her wrist. In instructing Joyce and her family on allowing the cast to dry properly, tell them to: 1. Continuously elevate Joyce’s arm on a pillow. 2. Change the position of Joyce’s arm every hour. 3. Position a fan near Joyce during the night to ensure even drying of the cast. 4. Put a blanket over the cast to absorb the dampness. Rationales Option 1: Elevating her arm will prevent edema, but continuous elevation is not needed. Option 2: Instructions to the client and family on how to allow a cast to dry properly should include advising them to change the position of the casted extremity every hour. In this case, Joyce’s arm should be repositioned frequently to prevent indentations in the cast itself (caused by continuous placement on a pillow) and to ensure drying of all surfaces of the cast. Option 3: A fan will dry only the outside of the cast. Option 4: A blanket will prevent drying of the cast. [Page reference: 1239-1240] Sandy, age 49, presents with loss of anal sphincter tone, impaired micturition, incontinence, and progressive loss of strength in the legs. You suspect cauda equina syndrome. What is your next action? 1. Ordering physical therapy. 2. Ordering a lumbosacral x-ray. 3. Ordering extensive lab work. 4. Referring to a neurosurgeon. Rationales Option 1: A prompt referral to a neurosurgeon is required when a diagnosis of cauda equina syndrome is suspected. Option 2: An x-ray is not helpful in the diagnosis of cauda equina syndrome, and precious time should not be wasted with a client who has suspected cauda equina syndrome. Magnetic resonance imaging (MRI) can be a useful diagnostic tool, but prompt evaluation by a neurosurgeon is an essential first step in preventing permanent neurologic damage. Option 3: A prompt referral to a neurosurgeon is required when a diagnosis of cauda equina syndrome is suspected. Option 4: A prompt referral to a neurosurgeon is required when a diagnosis of cauda equina syndrome is suspected. Cauda equina syndrome is a widespread neurologic disorder in which there is loss of anal sphincter tone; impaired micturition; incontinence; saddle anesthesia (ie, loss of sensation in the anus, perineum, genitals, and inner thighs); and motor weakness or sensory loss in both legs. [Page reference: 828-829] When teaching Alice, age 77, to use a cane because of osteoarthritis of her left knee, an important point to stress is: 1. Carrying the cane in the ipsilateral hand. 2. Advancing the cane with the ipsilateral leg. 3. Making sure the cane length equals the height of the iliac crest. 4. Using the cane to aid in joint protection and safety. Rationales Option 1: The cane should be carried in the contralateral hand. Option 2: When teaching clients about using a cane, tell them to advance the cane with the ipsilateral (affected) leg. Option 3: The cane length should equal the height of the greater trochanter. Option 4: The use of assistive devices is an important strategy for protecting the joints and providing safety, but clients must be taught the proper use of all devices. [Page reference: 856-862] You are caring for a patient who has a history of psoriasis and is now showing signs of joint involvement. Seropositivity provides a definitive diagnosis of psoriatic arthritis (PsA). The initial treatment choice for management of the patient is: 1. Disease-modifying antirheumatic drugs (DMARDs). 2. Nonsteroidal anti-inflammatory drugs (NSAIDs). 3. Tumor necrosis factor-alpha (TNF-α) inhibitors. 4. Uricosuric medications. Rationales Option 1: DMARDs, such as methotrexate, are used for early-stage treatment of active disease with structural damage and inflammation. Option 2: NSAIDs are the first-line treatment for musculoskeletal signs and symptoms with joint involvement. Option 3: Biologic agents, or TNF-a inhibitors, are considered for patients with active disease and inadequate response to one or more systemic DMARDs or very active PsA. Option 4: A uricosuric such as probenecid, with the addition of the anti-inflammatory colchicine, is indicated for chronic gouty arthritis. [Page reference: 1004] Hilda, age 73, presents with a complaint of low back pain. Red flags in her history of a minor fall, osteopenia, and prolonged steroid use for systemic lupus erythematosus suggest the possibility of which of the following serious underlying conditions as the cause of her low back pain? 1. Cancer. 2. Cauda equina syndrome. 3. Neurologic compromise. 4. Spinal fracture. Rationales Option 1: Cancer may be suspected if the low back pain is accompanied by unexplained weight loss and immunosuppression in a person over age 50. Option 2: Low back pain accompanied by acute onset of urinary retention or overflow incontinence, loss of anal sphincter tone or fecal incontinence, loss of sensation in the buttocks and perineum, and motor weakness in the lower extremities is a red flag for cauda equina syndrome or severe neurologic compromise. Option 3: Low back pain accompanied by acute onset of urinary retention or overflow incontinence, loss of anal sphincter tone or fecal incontinence, loss of sensation in the buttocks and perineum, and motor weakness in the lower extremities is a red flag for cauda equina syndrome or severe neurologic compromise. Option 4: The red flags for spinal fracture include major trauma or direct blow to the back in an adult, a minor fall or heavy lifting in a potentially osteoporotic or elderly person, prolonged steroid use, and age greater than 70. [Page reference: 829-830] A nurse practitioner is driving home from work and stops at the scene of a motorcycle accident that must have just occurred, as there are no rescue vehicles present. The driver is lying unconscious at the side of the road with an obvious open fracture of his femur. Which of the following actions should take priority? 1. Stopping the bleeding from the wound. 2. Determining if there has been a cervical fracture. 3. Establishing an airway. 4. Palpating the peripheral pulses. Rationales Option 1: This is an important action, but establishing an airway is the first priority. Option 2: This is an important action, but establishing an airway is the first priority. Option 3: Follow the ABCs of first aid: airway, breathing, and circulation. Establishing an airway is the first priority, followed by breathing, and then circulation. Stopping the bleeding from the wound, assessing if there has been a cervical fracture, and palpating the peripheral pulses are all important actions, but if the client is not breathing, the other actions will not be necessary. Option 4: This is an important action, but establishing an airway is the first priority. [Page reference: 1236] Mr. McKinsey, age 69, was recently given a diagnosis of degenerative joint disease. Which assessment should the nurse practitioner use to check for effusion of the patient’s knee? 1. Thomas test. 2. Tinel test. 3. Bulge test. 4. Phalen test. Rationales Option 1: The Thomas test is used to assess for hip problems. Option 2: The Tinel test assesses for carpal tunnel syndrome. Option 3: The bulge test assesses for an effusion of the knee. If effusion is present, a bulge will appear at the sides of or below the patella when the practitioner compresses the area above the patella. Option 4: The Phalen test assesses for carpal tunnel syndrome. [Page reference: 806-807] Sam, age 50, presents with Paget disease that has been stable for several years. Recently, his serum alkaline phosphatase level has been steadily rising. The nurse practitioner determines that it is time to start him on pharmacologic management. Which of the following should she initially prescribe? 1. Nonsteroidal anti-inflammatory drugs (NSAIDs). 2. Corticosteroids. 3. Bisphosphonates. 4. Calcitonin. Rationales Option 1: NSAIDs are helpful for clients with Paget disease who have mild symptoms and pain. Option 2: Corticosteroids do inhibit bone metabolism, but their use is limited by the side effects of long-term therapy with high doses. Option 3: When the serum alkaline phosphatase level rises—which indicates the disease has progressed—bisphosphonates, which decrease bone resorption by inhibiting osteoclast activity, are the treatment of choice. Option 4: Calcitonin (Calcimar) also inhibits osteoclastic bone resorption but is not as powerful as the bisphosphonates and does not suppress disease activity for as long after cessation. [Page reference: 868-869] James, age 17, has been complaining of a painful knob below his right knee that has prevented him from actively participating in sports. He has recently been given a diagnosis of Osgood-Schlatter disease and asks you about his treatment options. The nurse practitioner should tell him that the initial treatment is: 1. Relative rest; he could benefit from hamstring, heel cord, and quadriceps stretching exercises. 2. Immobilization; a long-leg knee immobilizer is recommended. 3. Surgical intervention; removal of the bony fragments is necessary. 4. Bed rest for 1 week. Rationales Option 1: Osgood-Schlatter disease is an overuse injury that results from the excessive tension and pull of the patellar tendon on the tibial tuberosity. Treating the client conservatively while an adolescent will prevent potential problems as an active adult. Initially, relative rest should be used with hamstring, heel cord, and quadriceps stretching exercises. Option 2: If the problem persists, a long-leg knee immobilizer may be used. Option 3: Surgical intervention is rarely required and then only in adults after bone growth is complete. Option 4: Rest should be combined with stretching and strengthening exercises. [Page reference: 805-807] The nurse practitioner (NP) suspects a herniated disk in a 72-year-old patient. The NP elevates the patient’s affected leg when she is in the supine position, and it elicits back and sciatic nerve pain, which indicates a positive test. This is known as which test or sign? 1. Femoral stretch test. 2. Crossed straight leg raise test. 3. Doorbell sign. 4. Straight leg raise test. Rationales Option 1: The femoral stretch test is done with the client prone and the leg extended and knee flexed. Pain radiating to the anterior thigh indicates L4 radiculopathy. Option 2: In the crossed straight leg raise test, the uninvolved leg is elevated; sciatic pain down the contralateral leg indicates a herniated disk. Option 3: The doorbell sign indicates development of radiculopathy when the patient’s symptoms are reproduced by compressing the neurologic and muscular structures in the lateral aspect of the neck. Option 4: All of the tests listed are tests done to assess for a herniated disk. In the straight leg raise test, you elevate the affected leg when the client is in the supine position; back pain and sciatic nerve pain (ie, radiating leg pain) indicate a herniated disk. [Page reference: 799-800] Jill, age 49, has recently begun a rigorous weightlifting regimen. She presents to the primary care office with a shoulder dislocation. Which of the following clinical manifestations leads the nurse practitioner to suspect an anterior shoulder dislocation over a posterior dislocation? 1. Inability to shrug the shoulder. 2. Absence of pain. 3. Inability to rotate the shoulder externally. 4. Shortening of the arm. Rationales Option 1: Clinical manifestations of an anterior shoulder dislocation, which is far more common than a posterior dislocation, include the inability to shrug the shoulder, pain, and lengthening of the arm. Option 2: Pain is a clinical manifestation of anterior shoulder dislocation. Option 3: The inability to rotate the shoulder externally, along with the inability to elevate the arm, is a clinical manifestation of a posterior shoulder dislocation. Option 4: Lengthening of the arm is a clinical manifestation of anterior shoulder dislocation. [Page reference: 801-802] Karen, who is postmenopausal, is taking 1200 mg of calcium daily but does not understand why she also needs to take vitamin D. You tell her that: 1. A deficiency of vitamin D results in inadequate mineralization of bone matrix. 2. All vitamins need to be supplemented. 3. Vitamin D increases intestinal absorption of dietary calcium and mobilizes calcium from the bone. 4. Vitamin D binds with calcium to allow active transport into the cells. Rationales Option 1: Vitamin D deficiency does result in inadequate mineralization of bone matrix (ie, rickets)—which is more commonly seen in children—but that is not the reason why Karen should take vitamin D. Option 2: Not all vitamins need to be supplemented; indeed, harm can result if certain vitamins are taken in higher than recommended doses. Option 3: Advise clients taking calcium supplements that they also need to take vitamin D because vitamin D raises serum calcium levels by increasing the intestinal absorption of dietary calcium and mobilizing calcium from the bone. Option 4: Vitamin D does not bind with calcium to allow active transport into the cells. [Page reference: 723] Endocrine and Metabolic Questions Sigrid, age 48, appears with a 3-month history of heat intolerance, increased sweating, palpitations, tachycardia, nervousness, irritability, fatigue, and muscle weakness. Which test would you order first? 1. A blood chemistry panel. 2. Thyroid-stimulating hormone (TSH) level. 3. Liver function studies. 4. Electrocardiogram. Rationales Option 1: If antithyroid drugs are used, a complete blood count (CBC) will need to be performed. Option 2: For a client with the symptoms experienced by Sigrid, a TSH level should be ordered first because the symptoms suggest hyperthyroidism. The TSH level is the best screening test for hyperthyroidism. Other laboratory and isotope tests for hyperthyroidism include a free triiodothyronine (T3) or thyroxine (T4) level, T3 resin uptake, and thyroid autoantibodies, including thyrotropin receptor antibody (TRAb). Tests not routinely performed but that may be helpful include radioactive iodine uptake and a thyroid scan (with iodine-123 [123I] or technetium-99m), which help to determine the etiology of the hyperthyroidism and assess the functional status of any palpable thyroid irregularities or nodules associated with a toxic goiter. Option 3: If antithyroid drugs are used, liver function tests will need to be performed. Option 4: An electrocardiogram may be ordered because of the palpitations, but once the thyroid is stabilized, the cardiac rhythm usually returns to normal. [Page reference: 884] Eunice, age 32, has type 2 diabetes. She said she heard she should take an aspirin a day after she reaches menopause for its cardioprotective action. She does not have coronary artery disease, but her father does. How do you respond? 1. “You’re right. Your hormones protect you against coronary artery disease until menopause; then you should start on aspirin therapy.” 2. “The American Diabetes Association recommends that you start on low-dose aspirin therapy now.” 3. “Aspirin therapy is recommended for all patients over age 55 as a precautionary measure.” 4. “If you maintain good glycemic control, you don’t need aspirin therapy.” Rationales Option 1: The American Diabetes Association (ADA) recommends aspirin therapy as a primary prevention strategy in high-risk men and women with diabetes who have a family history of coronary heart disease. Option 2: The American Diabetes Association’s position statement on aspirin therapy in patients with diabetes recommends low-dose (81 mg) aspirin use as a secondary prevention strategy in men and women with diabetes who have evidence of large-vessel disease, such as a history of myocardial infarction, vascular bypass procedures, and stroke, and have no contraindications for the use of aspirin. They also recommend aspirin therapy as a primary prevention strategy in high-risk men and women with type 1 or type 2 diabetes who have a family history of coronary heart disease and for individuals who smoke, are hypertensive or obese, or who have albuminuria, cholesterol levels greater than 200 mg/dL, low-density lipoprotein cholesterol levels greater than 130 mg/dL, high-density lipoprotein cholesterol levels less than 40 mg/dL, and triglyceride levels greater than 250 mg/dL. Option 3: Aspirin therapy is not recommended as a precautionary measure in all patients over age 55. Option 4: The American Diabetes Association (ADA) recommends aspirin therapy as a primary prevention strategy in high-risk men and women with diabetes who have a family history of coronary heart disease. [Page reference: 932] Marie, age 50, has type 1 diabetes and checks her blood glucose level several times every day. Her blood glucose level ranges from 250 to 280 mg/dL in the morning and is usually about 140 at lunch, about 120 at dinner, and about 100 at bedtime. In the morning, she takes 30 units of neutral protamine Hagedorn (NPH) insulin and 4 units of regular insulin, and before dinner she takes 18 units of NPH insulin and 4 units of regular insulin. Although she has had her insulin dose adjusted several times in the past month, it has had no effect on her high morning blood glucose level. What is your next course of action? 1. Increase the evening NPH insulin dose by 2 more units. 2. Have her check her blood glucose level between 2 am and 4 am for the next several days. 3. Increase the morning regular insulin dose by 2 units. 4. Order a fasting blood sugar test. Rationales Option 1: If Marie’s blood glucose level from 2 am to 4 am is greater than 70 mg/dL, the evening dose of NPH insulin should be increased and changed from before dinner to before bedtime. Option 2: Marie is experiencing the Somogyi phenomenon (rebound hyperglycemia). If her blood glucose level from 2 am to 4 am is greater than 70 mg/dL, the evening dose of NPH insulin should be increased and changed from before dinner to before bedtime. This should prevent most cases of nocturnal hypoglycemia, which results in morning hyperglycemia. Many providers prefer the longer acting insulins, such as insulin glargine (Lantus) and insulin detemir (Levemir), because they are mostly “peakless” and have less risk of hypoglycemia than NPH. Option 3: Marie’s morning hyperglycemia is caused by her nocturnal hypoglycemia, so it is the evening dose that may need to be adjusted. Option 4: A fasting blood sugar test will not confirm the Somogyi phenomenon. The blood sugar level needs to be checked during the night to “catch” the Somogyi phenomenon. [Page reference: 915-917] Which class of antihypertensive agents may be problematic for clients with diabetes? 1. Angiotensin-converting enzyme (ACE) inhibitors. 2. Calcium channel blockers. 3. Beta blockers. 4. Alpha blockers. Rationales Option 1: ACE inhibitors are the first choice for clients with diabetes who have hypertension because they slow the progression of diabetic nephropathy. Option 2: Calcium channel blockers provide pressure reduction without adverse effects on lipids and glucose control. Option 3: Beta blockers may be problematic in clients with diabetes because they block what is often the first sign of hypoglycemia—tachycardia. Many clients with diabetes have compelling indications (such as coronary artery disease) for the use of beta blockers. In these clients, the need for a beta blocker outweighs any risk that might occur. Decreasing the possibility of low blood sugar by selecting appropriate agents and adjusting dosages may be necessary. If a client with diabetes is on a beta blocker, it is important to explain that instead of tachycardia, he or she will notice other signs of hypoglycemia (such as sweating) that are not affected by beta blockers. Option 4: Alpha blockers provide smooth control and an improved lipid profile. [Page reference: 925] Marty has pheochromocytoma. You instruct him to: 1. Void frequently in small amounts. 2. Not exercise for more than 30 minutes at a time. 3. Avoid sleeping in the prone position. 4. Take steroids. Rationales Option 1: Clients with pheochromocytoma should be told to void frequently in small amounts and to avoid a full bladder. In addition, to prevent stimulating a paroxysm, clients should also be advised to avoid smoking; drugs that may influence catecholamine release, such as some anesthetics, atropine, opiates, steroids, and glucagon; and activities that might displace abdominal organs, such as bending, exercising, straining, and vigorous palpation of the abdomen. For women, pregnancy should be discouraged. Option 2: Clients should avoid activities that might displace abdominal organs. Option 3: Clients should avoid activities that might displace abdominal organs. Option 4: Clients should avoid drugs that may influence catecholamine release, including steroids. [Page reference: 900] Jeffrey, age 17, has gynecomastia. You should also assess him for: 1. Obesity. 2. Endocrine abnormalities. 3. Testicular cancer. 4. Tuberculosis. Rationales Option 1: While Jeffrey may be obese, which would probably accentuate his gynecomastia, it is the more serious problem of testicular cancer that needs to be addressed. Option 2: Gynecomastia can occur secondary to hyperthyroidism and other endocrine imbalances. Option 3: Gynecomastia may be the first sign of testicular cancer. It is also associated with breast, adrenal, pituitary, lung, and hepatic malignancies. Hypogonadism produces low testosterone levels in men with normal estrogen levels. Alteration in breast tissue responsiveness to hormonal activity can result in gynecomastia. Gynecomastia can occur secondary to cirrhosis, chronic obstructive lung disease, malnutrition, hyperthyroidism and other endocrine imbalances, tuberculosis, and chronic renal disease. Option 4: Gynecomastia can occur secondary to tuberculosis. [Page reference: 774] Jeremiah, age 72, has gout and is obese. When teaching him about diet, which of the following do you tell him? 1. “Beer and wine are okay because they have no effect on uric acid.” 2. “Keeping your weight stable, even if you are a little overweight, is better than fluctuating.” 3. “You must go on a restricted, very low calorie diet to effect immediate change.” 4. “Fluid intake should exceed three thousand milliliters daily to prevent formation of uric acid kidney stones.” Rationales Option 1: Because both wine and beer in excessive amounts impair the ability of the kidneys to excrete uric acid, they should be used in moderation. Clients must be aware that binge drinking may provoke an acute attack. Option 2: If the client is obese, weight loss should be encouraged because loss of excess body fat may normalize serum uric acid without pharmacological intervention. Weight loss will also decrease stress on weight-bearing joints. Option 3: Caution regarding severe, rapid weight loss should be given because secondary hyperuricemia may result. A restricted, very low calorie diet may precipitate an acute attack. Option 4: Fluid intake should exceed 3000 mL daily to prevent formation of uric acid kidney stones. Clients should avoid dehydration because it may precipitate an acute attack. [Page reference: 952] An elderly client presents with atrial fibrillation. Which of the following lab tests is important in forming the diagnosis? 1. Complete blood count (CBC). 2. C-reactive protein (CRP). 3. Comprehensive metabolic panel (CMP). 4. Thyroid-stimulating hormone (TSH). Rationales Option 1: A CBC is not specific. Option 2: A CRP will detect inflammation and the possibility of heart disease but not atrial fibrillation. Option 3: A CMP is not specific. Option 4: Atrial fibrillation is a common presentation in elderly clients with hyperthyroidism. If the TSH is suppressed, a free thyroxine (T4) and triiodothyronine (T3) should be drawn. [Page reference: 884] Martin, age 62, has acute nontransient abdominal pain that grows steadily worse in the epigastric area and radiates straight through to the back. The pain has lasted for days. He is also complaining of nausea, vomiting, sweating, weakness, and pallor. Physical examination reveals abdominal tenderness and distention and a low-grade fever. What do you suspect? 1. Cholecystitis. 2. Acute pancreatitis. 3. Cirrhosis. 4. Cushing syndrome. Rationales Option 1: The pain with cholecystitis is in the upper right quadrant and is intermittent, usually after a fatty meal. Option 2: Acute pancreatitis is an inflammation of the pancreas caused by the release of activated pancreatic enzymes into the surrounding parenchyma, with subsequent destruction of tissue, blood vessels, and supporting structures. Although pancreatitis may be acute or chronic, acute symptoms include continuous abdominal pain of several days’ duration that increases in the epigastric area and radiates to the back, nausea, vomiting, sweating, weakness, pallor, abdominal tenderness, distention, and low-grade fever. Pancreatitis occurs primarily in middle-aged adults and slightly more often in women than in men. Option 3: The gastrointestinal (GI) manifestations of cirrhosis include parotid enlargement, esophageal or rectal varices, peptic ulcers, and gastritis. Option 4: The clinical manifestation of Cushing syndrome related to the gastrointestinal (GI) system is a peptic ulcer, which would result in intermittent pain related to meals. [Page reference: 607] Leah, age 70, has had diabetes for many years. When teaching her about foot care, you want to stress: 1. That her calluses will protect her from infection. 2. The need to assess the bottom of her feet carefully after walking barefoot. 3. That painless ulcerations might occur and feet should be examined with a mirror. 4. That mild pain is to be expected because of neuropathy. Rationales Option 1: She should try to avoid the development of calluses because preparations used to remove them are very caustic. Option 2: Leah should not be walking barefoot because her sensation is probably decreased as a result of neuropathy. Option 3: Painless ulcerations are very common in clients with diabetes, and the only way to assess for them in the feet is for clients to use a mirror to examine the bottoms of their feet. Option 4: Sensation may be decreased as a result of neuropathy. [Page reference: 921-922] Betty, age 40, has had type 1 diabetes for 20 years and takes a combination of neutral protamine Hagedorn (NPH) and regular insulin every day. She comes to the office because she has developed a severe upper respiratory infection with chills, fever, and production of yellow sputum. Because of her acute infection, you know that Betty is likely to require: 1. A decrease in her daily insulin dosage. 2. An increase in her daily insulin dosage. 3. A high-calorie dietary intake and no insulin change. 4. A change in her insulin from NPH to insulin aspart (NovoLog). Rationales Option 1: Betty is likely to require an increase in her daily insulin dosage. Option 2: For clients with diabetes requiring insulin, an increase in their daily insulin dosage is usually required in the presence of an acute infection. Betty should begin by increasing her regular insulin dose by just 2 units and then monitoring her blood sugar level. Option 3: Betty is likely to require an increase in her daily insulin dosage and no increase in dietary caloric intake. Option 4: At some point, a more physiologic insulin regimen (such as basal-bolus) might be considered. [Page reference: 922] A client with hyperthyroidism presents with a complaint of a “gritty” feeling in her eyes. Over the past week, her visual acuity has diminished, and her ability to see colors has changed. She also has a feeling of pressure behind her eyes. The next step for the nurse practitioner is to: 1. Order a thyroid ultrasound. 2. Refer the client for immediate evaluation by an ophthalmologist. 3. Order a total thyroxine (T4). 4. Prescribe a beta-adrenergic blocker. Rationales Option 1: This course of action would prolong treatment and does not address the client’s need to seek an immediate evaluation by an ophthalmologist. Option 2: The practitioner should refer the client for an immediate evaluation by an ophthalmologist. Clinically recognized Graves ophthalmopathy occurs in about 50% of cases of Graves disease. A client with Graves orbitopathy with these complaints is at risk of blindness if there is compression of the optic nerve. Additional symptoms include photophobia and diplopia. Autoantibodies present in Graves disease can cause increased muscle thickness in the eye, leading to edema and compression of the optic nerve. Fundal exam may reveal disk swelling. This is an emergency situation that may require hospitalization and treatment with prednisone to diminish the inflammation. Artificial tears are also helpful. In 75% of clients, the onset of Graves orbitopathy occurs within a year before or after the diagnosis of thyrotoxicosis but can sometimes precede or follow thyrotoxicosis by several years. Option 3: This course of action would prolong treatment and does not address the client’s need to seek an immediate evaluation by an ophthalmologist. Option 4: This course of action would prolong treatment and does not address the client’s need to seek an immediate evaluation by an ophthalmologist. [Page reference: 888] Morton has type 2 diabetes. His treatment, which includes diet, exercise, and 3 oral antidiabetic agents at maximum dose, is insufficient to achieve acceptable glycemic control. Your next course of action is to: 1. Give the patient a sliding scale with mealtime coverage with regular insulin. 2. Add a dosage of long-acting insulin at bedtime to the regimen. 3. Discontinue the oral antidiabetic agents and start insulin therapy with N and R. 4. Suggest treatment using an insulin pump. Rationales Option 1: As a first step, the addition of a bedtime injection of long-acting insulin such as insulin glargine (Lantus) or insulin detemir (Levemir) is recommended. Option 2: If treatment with diet, exercise, and oral antidiabetic agents is insufficient to achieve acceptable glycemic control in clients with type 2 diabetes, adding a dosage of insulin at bedtime to the regimen may be necessary. As a first step, the addition of a bedtime injection of long-acting insulin such as insulin glargine (Lantus) or insulin detemir (Levemir) is recommended. Intermediate-acting insulin such as neutral protamine Hagedorn (NPH) is no longer recommended because of the peaks in drug levels that can cause hypoglycemia. Initially, the dosage is 10 units at bedtime; then the dose is adjusted to reduce overnight hepatic glucose production and achieve a normal or near-normal fasting blood glucose concentration. If this regimen does not achieve the desired effect, the oral antidiabetic agents should be discontinued, and mealtime analogue rapid-acting insulin can be added to the largest meal. Most clients will eventually require 4 injections with the basal-bolus regimen. Option 3: As a first step, the addition of a bedtime injection of long-acting insulin such as insulin glargine (Lantus) or insulin detemir (Levemir) is recommended. Option 4: Insulin pumps are sometimes ordered for type 1 diabetes. [Page reference: 916-917] Jay has had diabetes for 10 years. He recently had a physical and was told he has some evidence of nephropathy. What is the first manifestation of this condition? 1. Microalbuminuria. 2. Development of Kimmelstiel-Wilson nodules. 3. Decreased serum urea nitrogen levels. 4. Decreased serum creatinine levels. Rationales Option 1: Microalbuminuria is the first symptom indicative of nephropathy in clients who have had diabetes for about 10 years (although some studies suggest 5 years). There is increased permeability of the capillaries, with resultant leakage of albumin into the glomerular filtrate, causing microalbuminuria. Option 2: The development of Kimmelstiel-Wilson nodules occurs in people with type 1 diabetes but does not necessarily precede microalbuminuria. Option 3: As renal function deteriorates, serum urea nitrogen levels increase. Option 4: As renal function deteriorates, serum creatinine levels increase. [Page reference: 920] Question 15. Sadie, age 40, has just been given a diagnosis of Graves disease. She has recently lost 25 lb, has palpitations, is very irritable, feels very warm, and has a noticeable bulge on her neck. The most likely cause of her increased thyroid function is: 1. Hyperplasia of the thyroid. 2. An anterior pituitary tumor. 3. A thyroid carcinoma. 4. An autoimmune response. Rationales Option 1: Hyperplasia of the thyroid results from hyperthyroidism; it does not cause it. Option 2: Although pituitary tumors can cause hyperthyroidism, they do not cause Graves disease. Option 3: Although thyroid carcinoma can cause hyperthyroidism, it does not cause Graves disease. Option 4: Graves disease is the result of an autoimmune response wherein antibodies are produced that act against the body’s own organs and tissues. Thyroid-stimulating immunoglobulins are found in 95% of people with Graves disease and are evidence of this autoimmune process. [Page reference: 880] A patient presents to your primary care office with abnormal lab results. On physical exam, you tap the patient’s facial nerve around the zygomatic arch, just anterior to the earlobe. This describes which of the following tests and is associated with which of the following lab abnormalities? 1. Trousseau sign, hypocalcemia. 2. Chvostek sign, hypocalcemia. 3. Chvostek sign, hypercalcemia. 4. Lachman test, hypercalcemia. Rationales Option 1: Trousseau sign is positive in hypocalcemia but is described as spasms of the hand and wrist following inflation of a blood pressure cuff in the arm. Option 2: The test described in the question is positive in someone with hypocalcemia and is called Chvostek sign. Option 3: The test described in the question is consistent with Chvostek sign but is not consistent with hypercalcemia; it is consistent with hypocalcemia. Option 4: A Lachman test is positive in the presence of anterior cruciate ligament tears in the knee. [Page reference: 875] Which of the following conditions is a common pathological cause of hirsutism? 1. Polycystic ovary syndrome. 2. Addison disease. 3. Hyperthyroidism. 4. Alopecia. Rationales Option 1: Ninety percent of people with polycystic ovary syndrome have associated hirsutism. Option 2: Addison disease is associated with hair loss. Option 3: Hyperthyroidism is associated with hair loss, not excessive hair growth. Option 4: Alopecia is the loss of hair on the body. [Page reference: 877] What is the primary pathological irregularity associated with diabetes mellitus type 1? 1. Nonfunctioning beta cells. 2. Insulin resistance. 3. Overproducing beta cells. 4. Elevated free fatty acids in the plasma. Rationales Option 1: Beta cells in the pancreas secrete insulin. When they don’t work, your body can’t lower its blood sugar; this is the pathophysiology of diabetes mellitus type 1. Option 2: This is most commonly associated with diabetes mellitus type 2. Option 3: This is a beta cell reaction associated with hyperglycemia. Option 4: This is a product of hyperglycemia, not the cause of diabetes mellitus type 1. [Page reference: 911] Which of the following is not a risk factor for diabetes mellitus type 2? 1. Body mass index (BMI) greater than 25. 2. History of gestational diabetes. 3. Caucasian race. 4. History of polycystic ovary syndrome (PCOS). Rationales Option 1: A BMI greater than 25 is a risk factor for diabetes mellitus type 2. Option 2: A history of gestational diabetes is associated with the development of diabetes mellitus type 2 later in life. Option 3: Caucasians are less likely than African Americans, Asian Americans, Latin Americans, and Native Americans to develop diabetes mellitus type 2. Option 4: Women diagnosed with PCOS are more likely to develop diabetes mellitus type 2. [Page reference: 923] A 55-year-old Asian male presents with a history of severe left great toe pain. He states he cannot even touch the toe with a sheet without it causing pain. He denies trauma but states he cannot ambulate without pain. He admits to drinking alcohol but not to excess. On physical exam, he has normal vital signs, and you note erythema of the great toe at the interphalangeal (IP) joint. Which of the following is the gold standard for diagnosis of this problem? 1. Joint aspiration with crystal analysis. 2. Serum uric acid level. 3. X-ray. 4. Diagnosis is made with physical exam only. Rationales Option 1: This is the gold standard diagnostic test for diagnosis of gout. Option 2: Hyperuricemia supports a diagnosis of gout but is not in itself diagnostic. Option 3: X-rays are not diagnostic of gout. Option 4: When you have a clinical suspicion of gout, you can treat the patient, but aspiration of the affected joint with crystal analysis is the gold standard for diagnosing gout. [Page reference: 949] What is the medication of choice for an initial acute attack of gout? 1. A nonsteroidal anti-inflammatory drug (NSAID). 2. Colchicine. 3. A corticosteroid. 4. Allopurinol (Zyloprim). Rationales Option 1: The medication of choice for an initial acute attack of gout is an NSAID. Indomethacin (Indocin) is the most commonly prescribed NSAID for this use. An initial dose of 50 to 75 mg is given, followed by 25 to 50 mg every 8 hours for 5 to 10 days. An alternative to indomethacin is naproxen (Naprosyn). The first dose of naproxen is 750 mg, followed by 250 mg every 8 hours for 5 to 10 days. Option 2: Colchicine is an effective medication to terminate an acute attack only if administered within 48 hours of the initial onset of symptoms. Unfortunately, the attack is usually not diagnosed within this time frame. Option 3: Corticosteroids can provide dramatic systematic relief but are contraindicated in septic conditions; therefore, they should not be administered before analysis of the synovial aspirate. Option 4: Allopurinol (Zyloprim) is used to decrease uric acid production. Although it is effective, it may take weeks to decrease the uric acid level and therefore is not the initial choice in an acute attack. [Page reference: 950] The process of aging results in: 1. An increase in liver weight and mass. 2. A decreased absorption of fat-soluble vitamins. 3. An increase in enzyme activity. 4. Constricted pancreatic ducts. Rationales Option 1: There is a decrease in the number and size of hepatic cells, leading to a decrease in liver weight and mass. Option 2: The process of aging results in a decreased absorption of fat-soluble vitamins. Option 3: There is a decrease in enzyme activity, which diminishes the liver’s ability to detoxify drugs. This increases the risk of toxic levels of many medications in older adults. Option 4: There is calcification of the pancreatic vessels, and the ducts distend and dilate. These changes lead to a decrease in the production of lipase. [Page reference: 1284] A client with newly diagnosed diabetes who has a glycated hemoglobin (HbA1c) of 7.5 is started on therapeutic lifestyle changes (TLCs) and medical nutrition therapy (MNT). Which oral antidiabetic agent is recommended as monotherapy? 1. Glipizide (Glucotrol). 2. Sitagliptin (Januvia). 3. Exenatide (Byetta). 4. Metformin (Glucophage). Rationales Option 1: Insulin secretagogues such as sulfonylureas can cause hypoglycemia and are often added as a second, cost-effective choice. Option 2: The dipeptidyl peptidase-4 (DPP4) inhibitors such as sitagliptin (Januvia) increase insulin secretion, suppress glucagon secretion, and suppress hepatic glucose production and peripheral glucose uptake and metabolism. Option 3: An injectable agent, exenatide (Byetta), is an incretin mimetic that often induces weight loss. Both sitagliptin and exenatide have a lower risk of hypoglycemia, making them a good choice over a sulfonylurea as dual therapy. The cost of both of these newer agents may be prohibitive for many clients, and evidence for long-term reduction of morbidity and mortality is lacking. There is also a risk of pancreatitis with oral and injectable incretin-like drugs. Option 4: Because of its safety, efficacy, and cost, metformin is the cornerstone of monotherapy unless there is a contraindication, such as renal disease, hepatic disease, gastrointestinal intolerance, or risk of lactic acidosis. Metformin often has beneficial effects on components of metabolic syndrome, including mild to moderate weight loss, improvement of the lipid profile, and improved fibrinolysis. It improves the effectiveness of insulin in suppressing excess hepatic glucose production and increases insulin sensitivity in peripheral tissues. The risk of hypoglycemia with metformin is low. Gastrointestinal side effects can be diminished by starting at the lowest dose of 500 mg daily and gradually increasing as needed to a maximum dose of 2000 mg per day. Use of metformin with alcohol can increase the risk of lactic acidosis. [Page reference: 927-929] Mr. Reynolds is on the antithyroid drug (ATD) methimazole (Tapazole), so you make it a point to check his: 1. Glycated hemoglobin (HbA1c). 2. Complete blood count (CBC) and liver transaminases. 3. Uric acid level. 4. Total thyroxine (T4). Rationales Option 1: HbA1c is monitored in clients with diabetes. Option 2: ATDs can cause agranulocytosis and hepatic injury; therefore, CBC and liver studies should be done. Option 3: Uric acid is monitored in clients with gout. Option 4: Thyroid function tests are needed in patients taking ATDs; however, thyroid-stimulating hormone (TSH) and free T4 are preferred. [Page reference: 886] After an oral cholecystogram, Sam complains of burning on urination. This is because of: 1. A mild reaction to the contrast medium. 2. Biliary obstruction. 3. Contraction of the gallbladder. 4. The presence of dye in the urine. Rationales Option 1: A reaction to the contrast medium would produce symptoms such as urticaria, nausea, vomiting, and dyspnea. Option 2: An oral cholecystogram is done to assess for biliary obstruction. Option 3: To obtain a better reading during a cholecystogram, contraction of the gallbladder may be desirable and may be accomplished by having the client consume a high-fat meal during the procedure. Option 4: After an oral cholecystogram, some people experience burning on urination because of the presence of dye in the urine. This is helped by forcing fluids. [Page reference: 539] Ben, a client with type 1 diabetes, is hospitalized with an admitting diagnosis of diabetic ketoacidosis (DKA). Which of the following signs and symptoms would be consistent with this condition? 1. Hypoglycemia and glycosuria. 2. Decreased respiratory rate with shallow respirations. 3. Polydipsia and an increased blood pH. 4. Ketonuria and polyuria. Rationales Option 1: Hyperglycemia, not hypoglycemia, is a symptom of DKA. Option 2: Very deep, not shallow, respirations are a symptom of DKA. Option 3: A decreased, not increased, blood pH is a symptom of DKA. Option 4: Signs and symptoms of diabetic ketoacidosis include Kussmaul breathing (very deep respiratory movements), hyperglycemia, glycosuria, polyuria, polydipsia, anorexia, and headache, as well as ketonuria and a decreased blood pH. [Page reference: 915] Your client with diabetes asks you about insulin glargine (Lantus). You tell her that: 1. It may be administered subcutaneously at home or intravenously in the hospital if need be. 2. The onset of action is 15 minutes. 3. Insulin glargine (Lantus) stays in your system for 24 hours. 4. It can be mixed with any other insulin. Rationales Option 1: Insulin glargine (Lantus) must be administered subcutaneously, not intravenously. Regular insulin may be administered by the intravenous route. The newer insulin analogues, such as insulin aspart (NovoLog), are also approved for intravenous use. Option 2: Insulin glargine (Lantus) has an onset of action of just over 1 hour. Option 3: Insulin glargine (Lantus) has an onset of action of just over 1 hour and stays in the system for 24 hours. Option 4: Insulin glargine (Lantus) and insulin detemir (Levemir) may not be mixed with any other insulin. [Page reference: 916-917] Jennifer has diabetes mellitus (DM) and is injecting 30 units of Novolin 70/30 with breakfast and 18 units at bedtime. She is complaining that she woke up once in the middle of the night with palpitations and sweating. Based on this information, what do you recommend? 1. Decreasing the am dose of 70/30. 2. Decreasing the pm dose of 70/30. 3. Eating a snack before going to bed. 4. Changing the time of the nighttime insulin injection. Rationales Option 1: Decreasing the morning dose of insulin will not affect her symptoms. Option 2: While decreasing her nighttime dose might have some effect, the Novolin 70/30 must be given 30 minutes prior to the evening meal, not at bedtime. Option 3: While eating a snack before bedtime might have some effect, the Novolin 70/30 must be given 30 minutes prior to the evening meal, not at bedtime. Option 4: The Novolin 70/30 should be given before a meal. Jennifer was not given appropriate instructions on the timing of the injections. She should inject her nighttime dose about 30 minutes prior to the evening meal. Injecting this type of insulin before bed will cause the blood sugar to drop because 30% of the insulin is regular insulin. [Page reference: 916] Dan, age 45, is obese and has type 2 diabetes. He has been having trouble getting his glycohemoglobin under control. He has heard that exenatide (Byetta) causes weight loss and wants to try it. What do you tell him? 1. “Let’s adjust your oral antidiabetic agents instead.” 2. “That’s a myth. People usually change their eating habits when taking this, and that’s what causes the weight loss.” 3. “With type 2 diabetes, you never want to be on injectable insulin.” 4. “Let’s try it. Your glycohemoglobin will be lowered and you may lose weight.” Rationales Option 1: Adjusting the client’s oral antidiabetic agents may not be as effective, but these drugs are less expensive. Option 2: Exenatide (Byetta) can cause weight loss in some individuals. Option 3: Glucagon-like peptide-1 (GLP-1) injectables like exenatide are not injectable insulins. Option 4: Unlike many oral antidiabetic agents, injectable exenatide (Byetta) can cause weight loss in some individuals. The active ingredient is a protein that encourages digestion and the production of insulin. Glucagon-like peptide-1 (GLP-1) injectables like exenatide and liraglutide (Victoza) may be used as an adjunct to diet and exercise to improve glycemic control in adults with type 2 diabetes mellitus. [Page reference: 930] You suspect that Sharon has hypoparathyroidism because, in addition to her other signs and symptoms, she has: 1. Elevated serum phosphate levels. 2. Elevated serum calcium levels. 3. Decreased neuromuscular activity. 4. Increased bone resorption, as implied by her bone density test. Rationales Option 1: Signs of hypoparathyroidism include elevated serum phosphate levels; decreased serum calcium levels; increased neuromuscular activity, which may progress to tetany; decreased bone resorption; hypocalciuria; and hypophosphatemia. Option 2: Signs of hypoparathyroidism include decreased serum calcium levels. Option 3: Signs of hypoparathyroidism include increased neuromuscular activity. Option 4: Signs of hypoparathyroidism include decreased bone resorption. [Page reference: 906] Morris has had type 1 diabetes for 10 years. Several recent urinalysis reports have shown microalbuminuria. Your next step would be to: 1. Order a 24-hour urinalysis. 2. Start him on an angiotensin-converting enzyme (ACE) inhibitor. 3. Stress the importance of strict blood sugar control. 4. Send him to a dietitian because he obviously has not been following his diet. Rationales Option 1: Ordering a 24-hour urinalysis will not give you any additional information. Option 2: Morris should be started on an ACE inhibitor such as enalapril (Vasotec). ACE inhibitors offer renoprotective effects by reducing intraglomerular pressure. They do this by inhibiting the renin-angiotensin system, which causes efferent dilation, and by improving glomerular permeability, which causes a reduction of glomerulosclerosis. ACE inhibitors also have this beneficial effect on clients with diabetes who are normotensive and even hypotensive. Diabetic nephropathy is the leading cause of end-stage renal disease in the United States. Monitoring for microalbuminuria is one method for identifying early nephropathy. Option 3: You do want to stress tight glycemic control, but Morris needs to be started on an angiotensin-converting enzyme (ACE) inhibitor now because he is already exhibiting microalbuminuria. Option 4: You may possibly send Morris to a dietitian, but he needs to be started on an angiotensin-converting enzyme (ACE) inhibitor now because he is already exhibiting microalbuminuria. [Page reference: 920] Jason, age 14, appears with tender discoid breast tissue enlargement (2-3 cm in diameter) beneath the areolae. Your next action would be to: 1. Perform watchful waiting for 1 year. 2. Order an ultrasound. 3. Obtain laboratory tests. 4. Refer Jason to an endocrinologist. Rationales Option 1: Pubertal gynecomastia is common and is characterized by tender discoid breast tissue enlargement of about 2 to 3 cm in diameter beneath the areolae. The swelling usually subsides spontaneously within a year, and watchful waiting along with reassurance is recommended for that time period. Option 2: Because this condition is fairly common, there is no need to order an ultrasound. Option 3: Because this condition is fairly common, there is no need to obtain laboratory tests. Option 4: Because this condition is fairly common, there is no need to refer Jason to an endocrinologist. [Page reference: 876] What is the most common cause of gynecomastia? 1. Puberty. 2. Drug use. 3. Testicular failure. 4. Malnutrition. Rationales Option 1: Puberty is the most common cause of gynecomastia; most of these cases are self-limiting. Option 2: Drug use is a less common cause of gynecomastia. Option 3: Testicular failure is a less common cause of gynecomastia. Option 4: Malnutrition is a less common cause of gynecomastia. [Page reference: 876] QuWhat is the most common cause of Cushing disease? 1. Pituitary adenoma. 2. Prednisone use. 3. Adrenal tumor. 4. Nonpituitary tumor. Rationales Option 1: Pituitary adenomas are the cause of 70% of cases of Cushing disease; they secrete excessive adrenocorticotropic hormone (ACTH). Option 2: Exogenous glucocorticoids are the second most common cause of Cushing disease. Option 3: Adrenal tumors can cause Cushing disease but are responsible for only 30% of adrenocorticotropic hormone (ACTH)–independent cases. ACTH-independent Cushing disease is less common than ACTH-dependent Cushing disease. Option 4: Nonpituitary tumors that secrete adrenocorticotropic hormone (ACTH) are rare. [Page reference: 901] Which of the following statements about metformin is untrue? 1. Metformin works by decreasing hepatic glucose production and increasing peripheral cell sensitivity to insulin. 2. Metformin can cause lactic acidosis. 3. Metformin is typically used in conjunction with other diabetes medications. 4. Metformin cannot be used in patients with significant renal impairment. Rationales Option 1: This is metformin’s mechanism of action. Option 2: Metformin has a black box warning due to this side effect associated with its use. Option 3: Metformin is first-line treatment for diabetes type 2 and can be used as monotherapy. Option 4: The glomerular filtration rate must be greater than 45 in order to use metformin. [Page reference: 928] Which of the following body mass index (BMI) values defines class 1 obesity? 1. 30. 2. 25. 3. 40. 4. 35. Rationales Option 1: Class 1 obesity is defined as a BMI of 30 to less than 35. Option 2: Overweight is defined as a BMI of 25 to less than 30. Option 3: Class 3 obesity is defined as a BMI of 40 or higher. Option 4: Class 2 obesity is defined as a BMI of 35 to less than 40. [Page reference: 939] Joan has severe asthma and has been on high doses of oral corticosteroids for 2 years. She has been reading some home remedy books and stops all of her medications. What condition may she develop? 1. Myxedema crisis. 2. Diabetes insipidus. 3. Hypoparathyroidism. 4. Addisonian crisis. Rationales Option 1: With a myxedema crisis, there is the possibility of secondary hypothyroidism and associated hypopituitarism, so hydrocortisone should be administered until adrenal insufficiency has been ruled out. Option 2: Diabetes insipidus should not be affected by Joan’s stopping her corticosteroid medication. Option 3: Hypoparathyroidism should not be affected by Joan’s stopping her corticosteroid medication. Option 4: Addisonian crisis is a serious, life-threatening response to acute adrenal insufficiency and may be precipitated by abruptly stopping glucocorticoid medications. Other causes include major stressors, especially if the person has poorly controlled Addison disease, and hemorrhage into the adrenal glands from either septicemia or anticoagulant therapy. The primary problems in Addisonian crisis are severe hypotension, circulatory collapse, shock, and coma. Treatment involves rapid intravenous replacement of fluids and glucocorticoids. The most common cause of hyperthyroidism is: 1. Graves disease. 2. A toxic uninodular goiter. 3. Subacute thyroiditis. 4. A pituitary tumor. Rationales Option 1: The most common cause of hyperthyroidism is an autoimmune condition known as Graves disease, which accounts for 90% of hyperthyroid conditions in young adults. Option 2: A toxic uninodular goiter is the second most common cause of hyperthyroidism. Option 3: Subacute thyroiditis is a less common cause of hyperthyroidism. Option 4: A pituitary tumor is a less common cause of hyperthyroidism. Steve, age 42, has never been hypertensive but appears today in the office with a blood pressure of 162/100 mm Hg. He also complains of “attacks” of headache, perspiration, and palpitations, with frequent bouts of nausea, pain, weakness, dyspnea, and visual disturbances. He has lost 10 lb over the past 2 months and seems very anxious today. Your next action would be to: 1. Start him on an antianxiety agent. 2. Obtain a 24-hour urine test for catecholamines. 3. Start him on a diuretic or beta blocker. 4. Recheck his blood pressure in 1 week. Rationales Option 1: Starting Steve on an antianxiety agent will only delay the correct diagnosis. Option 2: Steve’s signs and symptoms are diagnostic of a pheochromocytoma, which can be detected with an assay of urinary catecholamine (total and fractionated), metanephrine, vanillylmandelic acid, and creatinine levels. A 24-hour urine specimen is usually obtained, but an overnight or shorter collection may also be obtained. Pheochromocytoma typically causes attacks of severe headache (85%), palpitations (65%), and profuse sweating (65%). The absence of all 3 of these symptoms can exclude the diagnosis of pheochromocytoma with 99% certainty. Option 3: Starting Steve on a diuretic and/or beta blocker will only delay the correct diagnosis. Option 4: Rechecking Steve’s blood pressure in 1 week will only delay the correct diagnosis. Jenny, age 46, has hypertension that has been controlled with hydrochlorothiazide 50 mg every day for the past 3 years. She is 5 ft 8 in tall and weighs 220 lb. Her fasting blood sugar (FBS) is 300 mg/dL, serum cholesterol level is 250 mg/dL, serum potassium level is 3.4 mEq, and she has 4+ glucosuria. Your next course of action would be to: 1. Discontinue her hydrochlorothiazide. 2. Order a glucose tolerance test (GTT). 3. Repeat her FBS and do a glycated hemoglobin (HbA1c). 4. Start insulin therapy. Rationales Option 1: Hyperglycemia can be an adverse reaction to high doses of hydrochlorothiazide, but the first action would be to repeat the FBS. If it is still high on a second reading, the diuretic should be reduced. Option 2: A GTT to confirm a diagnosis of diabetes is usually not needed. Diabetes is not usually diagnosed with a single high glucose reading unless symptoms of polyphagia, polydipsia, and polyuria are present. Option 3: Jenny’s FBS should be repeated along with an HbA1c. An HbA1c of greater than 6.5% can now be used to diagnosis diabetes. Option 4: Insulin therapy would not be started until Jenny was given a positive diagnosis, and even then oral antidiabetic agents would be considered first. Juanita, age 23, complains of palpitations that started a few weeks ago; they occur 2 to 4 times a day and last 5 to 10 minutes. She feels nervous and is having trouble sleeping. Her stools have been frequent (1-3 per day) and loose. She is taking levothyroxine 150 µg daily. Her labs indicate free thyroxine (T4) 2.28 and thyroid-stimulating hormone (TSH) 0.022. She has a history of Graves disease and had radioactive iodine (RAI) treatment a few months ago. She has been on thyroid replacement for 2 months. Based on these data, you decide to: 1. Increase the levothyroxine dosage. 2. Decrease the levothyroxine dosage. 3. Keep the dosage the same. 4. Start propranolol every 8 hours. Rationales Option 1: A lower dosage of levothyroxine will probably correct Juanita’s symptoms. Option 2: It appears that she may be overcorrected. The usual dosage of thyroid replacement is 1.6 µg/kg/d. She could skip a dose and then resume at a lower dosage of 125 to 137 µg per day. In an older individual, the lower dose would be preferred because overcorrection can lead to atrial fibrillation. She should take the levothyroxine on an empty stomach with a full glass of water and wait 30 minutes before eating for maximum absorption. Option 3: A lower dosage of levothyroxine will probably correct Juanita’s symptoms. Option 4: Propranolol, which may help palpitations in some patients, is not necessary here, as the correct dosage of levothyroxine will probably correct Juanita’s symptoms. A low thyroid-stimulating hormone (TSH) can lead to: 1. Osteoporosis. 2. Weight gain. 3. Bradycardia. 4. Brittle hair. Rationales Option 1: Hyperthyroidism presents with a suppressed TSH and elevated free thyroxine (T4). Manifestations include weight loss, tachycardia, diarrhea, anxiety, and warm, silky skin. The increased metabolic state of hyperthyroidism can cause cardiac dysrhythmias and osteoporosis. The clinical manifestations can also occur when there is excessive thyroid replacement. Option 2: Manifestations of hyperthyroidism include weight loss (not weight gain). Option 3: Manifestations of hyperthyroidism include tachycardia (not bradycardia). Option 4: Manifestations of hyperthyroidism include warm, silky skin (not brittle hair). Sara, age 40, has diabetes and is now experiencing anhidrosis on the hands and feet, increased sweating on the face and trunk, dysphagia, anorexia, and heartburn. Which complication of diabetes do you suspect? 1. Macrocirculation changes. 2. Microcirculation changes. 3. Peripheral neuropathies. 4. Autonomic neuropathies. Rationales Option 1: Macrocirculation changes include an early onset of atherosclerosis and peripheral vascular insufficiency with claudication, ulcerations, and gangrene of the legs. Option 2: Microcirculation changes include diabetic retinopathy with retinal ischemia and loss of vision and diabetic nephropathy with hypertension, albuminuria, edema, and progressive renal failure. Option 3: Peripheral neuropathies include changes in sensation in the feet and hands; palsy of cranial nerve III with headache, eye pain, and inability to move the eye up, down, or to the middle; pain or loss of cutaneous sensation over the chest; and motor and sensory deficits in the anterior thigh and medial calf. Option 4: Autonomic neuropathies include anhidrosis (absence of sweating) on the hands and feet, increased sweating on the face and trunk, dysphagia, anorexia, heartburn, constricted pupils, nausea and vomiting, constipation, and diabetic diarrhea. Mary, age 72, has been taking insulin for several years. She just called you because she realized that yesterday she put her short-acting insulin in the long-acting insulin box and vice versa. She just took 22 units of regular insulin when she was supposed to take only 5 units. She says that she tried to do a fingerstick to test her glucose level but was unable to obtain any blood. She states that she feels fine. What do you tell her to do first? 1. “Keep trying to get a fingerstick and call me back with the results.” 2. “Call 911 before you collapse.” 3. “Drive immediately to the emergency room.” 4. “Drink four ounces of fruit juice.” Rationales Option 1: Mary should eventually be able to get a fingerstick but does not need to call if it is okay. Option 2: Drinking 4 oz of fruit juice will probably correct the problem, so she may not need to call 911. Option 3: If her blood sugar is low, Mary should certainly not drive herself to the emergency room. Option 4: Treatment of hypoglycemia is 15 g of carbohydrates and can be achieved by drinking 8 oz of milk or 4 oz of orange juice. The milk option is preferred to decrease the elevation in blood sugar that occurs with orange juice. The patient should wait for 15 minutes to see if the symptoms subside and repeat the treatment again if needed. All patients on insulin should be prescribed a glucagon pen, and family or friends should be instructed on its use. A client with diabetes on a sulfonylurea and metformin with a glycated hemoglobin (HbA1c) of 6.5% is complaining of episodes of low blood sugar. Which of the following changes would be the most appropriate? 1. Decreasing the dosage of the metformin. 2. Discontinuing the metformin. 3. Increasing carbohydrate intake. 4. Decreasing the dosage of the sulfonylurea. Rationales Option 1: There is less risk of hypoglycemia with metformin than with sulfonylureas. Option 2: There is less risk of hypoglycemia with metformin than with sulfonylureas. Option 3: Increasing the carbohydrate intake is never a good choice in a patient with diabetes. Option 4: Metformin, dipeptidyl peptidase-4 (DPP4) inhibitors such as sitagliptin (Januvia), and incretin mimetics such as exenatide (Byetta) are gaining favor over sulfonylureas because the risk of hypoglycemia is less than with sulfonylureas. Mindy is scheduled to have an oral glucose tolerance test (OGTT). She is instructed to discontinue many of her medications for 3 days before the test. Which one is it safe to continue taking? 1. Vitamin C. 2. Aspirin. 3. Calcium. 4. Oral contraceptives. Rationales Option 1: Vitamin C should be discontinued for 3 days before the test. Option 2: Aspirin should be discontinued for 3 days before the test. Option 3: Calcium does not affect an OGTT. The following medications may interfere with the results of an OGTT and should be discontinued for 3 days before the test: vitamin C, aspirin, oral contraceptives, corticosteroids, synthetic estrogens, phenytoin (Dilantin), thiazide diuretics, and nicotinic acid. Option 4: Oral contraceptives should be discontinued for 3 days before the test. Be sure to recommend that clients use another form of birth control for this period of time. Sandra, age 28, has secondary obesity. Which of the following may have caused this? 1. Taking in more calories than are expended. 2. Polycystic ovary syndrome. 3. Antihypertensive medications. 4. A sedentary lifestyle. Rationales Option 1: Essential obesity is the most prevalent type of obesity and is the result of taking in more calories than are expended. This type of obesity results from the multiple interactions of genetic and environmental factors (cultural, metabolic, social, and psychological). Option 2: Secondary obesity is rare; possible causes include Cushing disease, polycystic ovary syndrome, hypothalamic disease, hypothyroidism, and insulinoma. Some medications associated with weight gain include glucocorticoids, tricyclic antidepressants, and phenothiazines. Option 3: Antihypertensive medications do not lead to obesity. Option 4: A sedentary lifestyle may lead to primary, not secondary, obesity. Which of the following statements about hypothyroidism is not true? 1. The most common worldwide cause of hypothyroidism is iodine deficiency. 2. The most common cause of autoimmune hypothyroidism is Hashimoto thyroiditis. 3. The rate of hypothyroidism decreases with age. 4. Lithium use is a risk factor for hypothyroidism. Rationales Option 1: This is true; hypothyroidism is less common in the United States due to access to iodinated salt. Option 2: This is true; Hashimoto thyroiditis is the most common cause of autoimmune hypothyroidism. Option 3: This is not true; hypothyroidism becomes more common as we age. Option 4: This is true; lithium and amiodarone are 2 drugs that can cause hypothyroidism. To reduce the incidence of flares, foods high in what amino acid need to be limited in the diets of patients with gout? 1. Purine. 2. Glutamine. 3. Phenylalanine. 4. Alanine. Rationales Option 1: High-purine foods need to be avoided in order to reduce the risk of gout flares. High-purine foods include fish, red meat, and beans. Beer is also rich in purine. Option 2: Purine-rich foods need to be limited in order to reduce the risk of gout. Option 3: Purine-rich foods need to be limited in order to reduce the risk of gout. Option 4: Purine-rich foods need to be limited in order to reduce the risk of gout. Marsha, age 24, is preparing for radioactive iodine therapy for her Graves disease. Which test must she undergo first? 1. Beta-human chorionic gonadotropin. 2. Basal metabolism rate. 3. Lithium level. 4. Serum calcium. Rationales Option 1: Radioactive iodine therapy is the most commonly used treatment in the United States for Graves disease (hyperthyroidism); however, it is contraindicated during pregnancy. Therefore, for women, a pregnancy test (beta-human chorionic gonadotropin) needs to be performed before initiating therapy. Women of childbearing age should also be told to delay conception for a few months after radioactive iodine therapy. It is also contraindicated in women who are breastfeeding. Older adults or clients at risk of developing cardiac complications may be pretreated with antithyroid drugs (ATDs) before therapy to deplete the thyroid gland of stored hormone, thereby minimizing the risk of exacerbation of hyperthyroidism because of radioactive iodine (131I)–induced thyroiditis. Option 2: Marsha’s basal metabolism rate will be affected by her Graves disease but has no bearing on her preparation for radioactive iodine therapy. Option 3: Lithium levels are usually not performed before radioactive iodine therapy. They may be done if lithium is being used to block the release of thyroid hormone from the thyroid gland in clients who are intolerant of antithyroid drugs (ATDs). Option 4: Although parathyroid hormone secretion is dependent on the serum calcium level, it is usually not necessary to obtain a serum calcium level measurement before radioactive iodine therapy. When teaching Marcy how to use her new insulin pump, you tell her that she needs to monitor her blood glucose level: 1. At least once a day. 2. Only occasionally because glycemic levels are maintained very steadily. 3. At least 4 times a day. 4. On an as needed basis when she feels she needs to give herself an extra dose of insulin. Rationales Option 1: Marcy needs to monitor her blood glucose level at least 4 times a day because the only insulin used in the pump is rapid-acting. Option 2: Marcy needs to monitor her blood glucose level at least 4 times a day because the only insulin used in the pump is rapid-acting. Option 3: Clients using an insulin pump need to monitor their blood glucose levels at least 4 times a day. Clients can develop diabetic ketoacidosis in as little as 4 hours if there is mechanical failure of the pump because the only insulin used in the pump is rapid-acting. Option 4: Marcy needs to monitor her blood glucose level at least 4 times a day because the only insulin used in the pump is rapid-acting. The major risk factor for thyroid cancer is: 1. Inadequate iodine intake. 2. Presence of a goiter. 3. Exposure to radiation. 4. Smoking. Rationales Option 1: There is an increased incidence of thyroid cancer in areas where iodine deficiency is more common. Option 2: There is an increased incidence of thyroid cancer in areas where goiter is more common. Option 3: The major risk factor for thyroid cancer is exposure to radiation, usually from treatment to the head and neck. Until 1950, radiation treatments were given to children for an enlarged thymus, enlarged tonsils, and acne. Several million children were exposed in this manner. It may also occur in individuals who have had radiation therapy to the face or upper chest. Option 4: Cigarette smoking is a risk factor for bladder and lung cancer but not thyroid cancer. The American Diabetes Association (ADA) recommends which of the following quarterly blood tests be performed on all clients with diabetes? 1. Thyroid-stimulating hormone (TSH). 2. Liver function studies. 3. Glycated hemoglobin. 4. Serum glucose. Rationales Option 1: The American Thyroid Association recommends measuring thyroid function in all adults beginning at age 35 and every 5 years thereafter. Option 2: Liver function studies should be done on an annual basis as part of a routine examination. Option 3: The ADA recommends that the glycated hemoglobin (HbA1c) test be performed quarterly because it reports the serum glucose concentration of the previous 3 months. HbA1c can now be used for diagnosis of diabetes (greater than 6.5%). The ADA also recommends an annual urine test to assess for urine protein, which might be an early sign of kidney damage. Option 4: Although a serum glucose test is an excellent test for clients with diabetes, it reports only the serum glucose of that day. Mason, age 52, has diabetes mellitus (DM) and is overweight. You now find that he is hypertensive. How should you treat his hypertension? 1. You should treat it the same as in a client without diabetes. 2. Because insulin affects most antihypertensive drugs, you should try diet and exercise first before ordering any antihypertensives. 3. You should treat it very aggressively, preferably with angiotensin-converting enzyme (ACE) inhibitors. 4. You should initiate therapy when the blood pressure is 5 to 10 mm Hg more than the conventional therapeutic guidelines. Rationales Option 1: Because hypertension is implicated in accelerating the microangiopathy of diabetes (especially retinopathy and nephropathy), according to the Eighth Joint National Committee (JNC 8) therapy should be initiated when the patient has a blood pressure at or above 140/90 mm Hg. Other organizations recommend the lower goal of 130/80 in clients with DM. Option 2: Owing to the beneficial effects of reducing albuminuria and glomerular pressure, angiotensin-converting enzyme (ACE) inhibitors are the drugs of choice in clients with DM with hypertension. Option 3: ACE inhibitors should be initiated in patients with diabetes and an elevated blood pressure and are recommended in normotensive patients with albuminuria. Option 4: Depending on the guidelines, therapy should be initiated when blood pressure reaches levels equal to or 10 mm Hg less than conventional therapeutic guidelines. Tamika, who has diabetes, states that she heard fiber is especially good to include in her diet. How do you respond? 1. “Fiber is important in all diets.” 2. “Too much fiber interferes with insulin, so include only a moderate amount in your diet.” 3. “Fiber, especially soluble fiber, helps improve carbohydrate metabolism, so it is more important in the diet of persons with diabetes.” 4. “You get just the amount of fiber you need with a normal diet.” Rationales Option 1: While fiber is important in all diets, this answer does not address why fiber is especially good to include in the diet of a patient with diabetes. Option 2: A diet high in fiber, especially soluble fiber, helps improve carbohydrate metabolism. Option 3: Fiber is important in the dietary management of diabetes. A diet high in fiber, especially soluble fiber, helps improve carbohydrate metabolism and lowers both total cholesterol and low-density lipoprotein cholesterol. Soluble fiber is found in dried beans, oats, and barley as well as some vegetables and fruits (peas, corn, zucchini, cauliflower, broccoli, prunes, pears, apples, bananas, and oranges). Option 4: It should not be assumed that individuals get enough fiber in their diet because most dietary habits are not perfect. An intake of 20 to 30 g of fiber per day is recommended. Question 10. Mark has type 1 diabetes and has mild hyperglycemia. What effect does physical activity (exercise) have on his blood glucose level? 1. It may cause it to vary a little. 2. It may decrease it. 3. It may elevate it. 4. It may fluctuate greatly either way. Rationales Option 1: For individuals without diabetes, the blood glucose level generally varies little during physical activity unless the activity is intense and of very long duration, such as marathon running. Option 2: Clients with insulin-dependent diabetes mellitus (IDDM)—ie, type 1 diabetes—who have mild hyperglycemia may experience a drop in their blood glucose level during physical activity, whereas those with marked hyperglycemia may experience a rise in their blood glucose level. Clients with IDDM should check their blood glucose level before exercising and refrain from exercising if their level is too high (greater than 300 mg/dL). Option 3: Clients with marked hyperglycemia may experience a rise in their blood glucose level during physical activity. Option 4: Since Mark has mild hyperglycemia, his blood glucose level may drop during physical activity. When you inspect the integumentary system of clients with endocrine disorders, a finding of coarse hair may be an indicator of: 1. Addison disease. 2. Diabetes mellitus. 3. Cushing syndrome. 4. Hypothyroidism. Rationales Option 1: Indicators of Addison disease include hyperpigmentation. Option 2: Indicators of diabetes mellitus include hypopigmentation. Option 3: Indicators of Cushing syndrome include hirsutism, hyperpigmentation, purple striae over the abdomen, and bruising. Option 4: During inspection of the integumentary system of clients with endocrine disorders, a finding of coarse hair may be an indicator of hypothyroidism. Fine hair is seen in clients with hyperthyroidism; hirsutism with Cushing syndrome; hyperpigmentation with both Addison disease and Cushing syndrome; hypopigmentation with diabetes mellitus, hyperthyroidism, and hypothyroidism; and purple striae over the abdomen and bruising with Cushing syndrome. Question 13. Which is the only curative treatment option for primary hyperparathyroidism (PHPT)? 1. Type II calcimimetic cinacalcet. 2. Hormone therapy. 3. Parathyroidectomy. 4. Bisphosphonates. Rationales Option 1: The type II calcimimetic cinacalcet treats the underlying cause of PHPT by binding to the calcium-sensing receptor on the surface of the parathyroid glands, which increases the sensitivity to extracellular calcium, which then reduces the excess secretion of parathyroid hormone (PTH). It is used for the treatment of secondary hyperparathyroidism but not for PHPT. Option 2: Hormone therapy is not used by itself. Low doses of estrogen have been shown to reduce calcium, prevent bone loss, and improve bone density. Option 3: The only curative treatment option for PHPT is a parathyroidectomy. It is successful in 90% to 98% of cases. Option 4: Although the first generation of bisphosphonates was found to be ineffective for treatment of the skeletal manifestations of PHPT, the newer bisphosphonates, such as alendronate (Fosamax), increase bone density a little, but they do not affect parathyroid hormone (PTH) secretion and thus will not reduce serum calcium. A patient presents to your primary care office complaining of polydipsia, polyuria, and polyphagia. Which of the following diagnoses would not be in your differential diagnosis? 1. Diabetes mellitus (DM). 2. Diabetes insipidus (DI). 3. Psychiatric disorders. 4. Hyperthyroidism. Rationales Option 1: Polydipsia, polyuria, and polyphagia are common presenting complaints in DM. Option 2: Polydipsia, polyuria, and polyphagia are common presenting complaints in DI. Option 3: Many psychiatric disorders can have polydipsia, polyuria, and polyphagia as presenting symptoms. Option 4: Symptoms of hyperthyroidism could include polyphagia due to increased metabolism, but symptoms generally are weight loss, tachycardia, and other physical manifestations of increased metabolism. A 35-year-old female presents to your primary care office for review of her laboratory results. Her physical exam shows a blood pressure (BP) of 140/90, pulse (P) of 105, oxygen saturation of 97%, and temperature of 98.6°F. She has complaints of palpitations, weight loss, hair loss, and anxiety. Her labs are all normal except for a low thyroid-stimulating hormone (TSH) and an elevated thyroxine (T4). What would your next course of treatment be? 1. Start metoprolol and propylthiouracil (PTU). 2. Refer the patient to psychiatry for treatment of anorexia. 3. Refer the patient for radioactive iodine treatment. 4. Refer the patient for thyroidectomy. Rationales Option 1: This is the presentation of a patient with hyperthyroidism. A beta blocker would help treat her tachycardia and hypertension and PTU would help normalize her thyroid hormones. Option 2: The patient’s physical complaints of palpitations, weight loss, and hair loss can be present in anorexia as well as hyperthyroidism; however, thyroid hormone levels would likely be normal in anorexia. Option 3: Radioactive iodine treatment is reserved for patients that have failed medical management of hyperthyroidism. Option 4: Thyroidectomy is reserved for patients with compressive goiter, symptoms of which would include stridor and hoarseness. A 35-year-old male presents to your office complaining of fatigue, weight loss, nausea, and abdominal pain. On physical exam, you notice he has orthostatic hypotension and hyperpigmented skin. You do a morning cortisol level, which is low. The plasma adrenocorticotropic hormone (ACTH) is elevated. How would you treat this patient? 1. Oral prednisone. 2. Hydrocortisone. 3. Androgen replacement. 4. Fludrocortisone. Rationales Option 1: This is reserved for patients that don’t respond well to hydrocortisone. Option 2: Hydrocortisone is the treatment of choice for primary adrenal insufficiency (Addison disease). Option 3: This is supplemental in the treatment of primary adrenal insufficiency. Option 4: This is reserved for patients with electrolyte disturbances associated with primary adrenal insufficiency. Which of the following statements about diabetes mellitus is untrue? 1. Diabetes mellitus is the most common etiology of renal failure in the United States. 2. Diabetes mellitus is the most common endocrine disorder in the United States. 3. Diabetes mellitus is characterized by impaired insulin secretion and insulin action. 4. Diabetes mellitus is curable. Rationales Option 1: This is true; it is also the most common etiology of acquired blindness. Option 2: This is true; it affects over 30 million people. Option 3: These are the 2 characterizations that apply to diabetes mellitus. Option 4: Diabetes mellitus is manageable with lifestyle modifications and medications but is not curable. Question 18. Which of the following would not confirm a diagnosis of diabetes? 1. Glycated hemoglobin (HbA1c) of 7.0. 2. Fasting glucose of 155. 3. Random glucose of 198. 4. Oral glucose tolerance test with a plasma glucose of 250. Rationales Option 1: An HbA1c greater than 6.5 is diagnostic of diabetes. Option 2: A fasting glucose greater than 126 is diagnostic of diabetes. Option 3: A random glucose greater than 200, with associated symptoms such as weight loss, polyuria, and polydipsia, is diagnostic of diabetes. Option 4: An oral glucose tolerance test with a plasma glucose greater than 200 is diagnostic of diabetes. Question 19. Which of the following would not be ordered on a regular basis to evaluate diabetic patients for end organ damage associated with diabetes? 1. Ophthalmology evaluation. 2. Lipid panel. 3. Urinalysis and basic metabolic panel (BMP). 4. Complete blood count (CBC). Rationales Option 1: The general recommendation for diabetic patients is to evaluate for diabetic retinopathy at diagnosis and at least every 2 years thereafter. Option 2: Hyperlipidemia is common in diabetes, and a lipid panel should be ordered annually. Option 3: A urinalysis and BMP should be ordered regularly for the evaluation of kidney damage associated with diabetes. Option 4: A CBC should be done in all patients at an annual physical, but diabetics don’t typically have end organ damage that would be evident on a CBC. Question 2. Harriet, age 62, has type 1 diabetes that is well controlled by insulin. Recently, she has been having marital difficulties that have left her emotionally upset. As a result of this stress, it is possible that she will: 1. Have an insulin reaction more readily than usual. 2. Have an increased blood sugar level. 3. Need less daily insulin. 4. Need more carbohydrates. Rationales Option 1: Harriet will not have an insulin reaction (such as hypoglycemia) more readily than usual. Option 2: Stress causes the adrenal glands to secrete more cortisol, which leads to gluconeogenesis and insulin antagonism, raising the blood sugar. It is possible, then, that Harriet will have an increased blood sugar level. She will not need less daily insulin or more carbohydrates and will not have an insulin reaction (such as hypoglycemia) more readily than usual. Harriet may, in fact, need to increase her insulin use. Option 3: She will not need less daily insulin; she may need to increase her insulin use. Option 4: She will not need more carbohydrates; it is possible she will have an increased blood sugar level. Question 4. Lynne has Cushing syndrome. You would expect her to have or develop: 1. Onychomycosis. 2. Generalized increased pigmentation of the skin. 3. Hair loss. 4. Excitability and nervousness. Rationales Option 1: Cushing syndrome results in an excessive amount of adrenocorticotropic hormone, which stimulates the secretion of glucocorticoids, mineralocorticoids, and androgenic steroids from the adrenal cortex. In the presence of excessive cortisol, fungal infections of the skin, nails, and oral mucosa, such as onychomycosis and tinea versicolor, are common and skin wounds heal very slowly. Option 2: Addison disease, which is a deficiency in the secretion of adrenocortical hormones, usually results in increased pigmentation of the skin in its entirety. Option 3: Other symptoms of Cushing syndrome include excessive hair growth (not hair loss). Option 4: Other symptoms of Cushing syndrome include fatigue and weakness (not excitability and nervousness). Joy has gout. In teaching her about her disease, which food do you tell her is allowed in the diet? 1. Asparagus. 2. Beans. 3. Broccoli. 4. Mushrooms. Rationales Option 1: Asparagus is high in purine. Option 2: Beans are high in purine. Option 3: Foods high in purine should be avoided by clients with gout. Broccoli is not high in purine. Foods high in purine include all meats and seafood, meat extracts and gravies, yeast and yeast extracts, beans, peas, lentils, oatmeal, spinach, asparagus, cauliflower, and mushrooms. Wine and alcohol in excessive amounts impair the ability of the kidneys to excrete uric acid and should be used in moderation. Option 4: Mushrooms are high in purine. Urgent Care Questions Question 1. What lifestyle choice increases the risk of upper respiratory infection? 1. Smoking. 2. Alcohol use. 3. Cocaine use. 4. Multiple sexual partners. Rationales Option 1: Smoking increases the risk of upper respiratory infection. Option 2: Alcohol use does not increase the risk of upper respiratory infection. Option 3: Drug use does not increase the risk of upper respiratory infection. Option 4: Having multiple sexual partners does not increase the risk of upper respiratory infection. Question 2. A 65-year-old female presents to your urgent care center complaining of a cough. She has a past medical history of myocardial infarction, hypertension, hyperlipidemia, and diabetes. She states she had this cough last year and received a Z-Pak, and it made her feel better, so she is requesting one now. Her vital signs are as follows: pulse (P) 85, blood pressure (BP) 140/90, oxygen saturation 95% on room air, temperature 99.0°F. Upon further questioning, her cough has been going on for 3 weeks and is nonproductive. She also notes some shortness of breath, mostly with periods of ambulation. She denies chest pain. She notes that recently her feet and legs have become more swollen. You do not have access to an x-ray at your facility. What is the most concerning cause of the patient’s cough that would be in your differential diagnosis? 1. Congestive heart failure. 2. Pneumonia. 3. Bronchitis. 4. Deep vein thrombosis. Rationales Option 1: The patient has dyspnea on exertion, nonproductive cough, and no fever. This is likely not an infectious cough. She also has lower extremity edema. Option 2: The patient has had a chronic nonproductive cough without fever; this is unlikely pneumonia. Option 3: Dyspnea on exertion and lower extremity swelling are not symptoms of bronchitis. Option 4: While the patient has swelling in the lower extremities, it is most likely caused by congestive heart failure. There is no mention of palpable cords, calf pain, or erythema, all of which generally accompany deep vein thrombosis. Question 3. A 15-year-old female presents to your urgent care center complaining of a sore throat for 3 days. Her vital signs are as follows: temperature 102.1°F, pulse (P) 70, blood pressure (BP) 130/85, oxygen saturation 97%. The patient denies cough. On physical exam, she has pearly white exudates on the tonsils, erythema of the throat, and palpable anterior cervical chain lymphadenopathy. What is the recommended treatment? 1. Starting the patient on antibiotics. 2. Starting the patient on prednisone. 3. Awaiting culture results prior to treatment. 4. Recommending supportive treatment. Rationales Option 1: The patient has signs and symptoms of streptococcal (strep) pharyngitis. This requires antibiotic treatment. If you have access to one, a rapid strep test can be done in the office. Option 2: Prednisone is not the treatment for bacterial pharyngitis. Option 3: The patient has many criteria for the diagnosis of streptococcal (strep) pharyngitis. Treatment with antibiotics is recommended. Option 4: The patient has streptococcal (strep) pharyngitis. Supportive treatment will not help and will increase the risk of strep-related end organ damage to organs such as the heart. Question 4. A 10-year-old female presents to your urgent care center with her mother complaining of ear pain. The patient spends most of her summer at the local swim club. She notes right ear pain. She has no fever. She awoke today with green drainage on her pillow. On physical exam, the patient has pain with palpation of the affected ear, and you cannot visualize the tympanic membrane. What is the most likely diagnosis? 1. Otitis externa. 2. Otitis media. 3. Ruptured tympanic membrane. 4. Acute mastoiditis. Rationales Option 1: This is the classic presentation of otitis externa. Option 2: The patient has drainage consistent with otitis externa and no upper respiratory complaints. Option 3: This usually presents with drainage following trauma or chronic pain/infection. Option 4: This presents with fever, chronic ear pain, and pain with palpation of the mastoid. Question 5. An 18-year-old high school cross country runner presents to your office complaining of foot pain for 2 weeks. The patient runs 3 to 6 miles per day. He states it feels better when he runs on the track as opposed to the road. On physical exam, he is tender to palpation over his fifth metatarsal head, but there is no ecchymosis, erythema, or edema of the foot. X-ray of his foot is negative for fracture. What is the best diagnostic study to order for further evaluation of his complaint? 1. Bone scan. 2. Computed tomography (CT) scan. 3. Magnetic resonance imaging (MRI). 4. X-ray in 1 week. Rationales Option 1: A bone scan is the most sensitive test for diagnosing a stress fracture. Option 2: A CT would show a fracture but is not as sensitive as a bone scan. Option 3: An MRI would show a fracture but is not as sensitive as a bone scan. Option 4: The patient has a stress fracture; reevaluation in 1 week with an x-ray may show a fracture, but there are more sensitive tests that can be done now. Question 6. A patient presents to your primary care clinic with diarrhea. What about the diarrhea would be concerning for a parasitic infection? 1. It presented 8 hours after eating rare fish. 2. It has lasted longer than 7 days. 3. The patient describes it as “rice water.” 4. It is bloody. Rationales Option 1: This timeline is typical with a foodborne illness and does not necessarily describe a parasitic infection. Option 2: Diarrhea that lasts longer than 7 days is concerning for a parasitic infection. Option 3: Rice water stool is described when a patient has a cholera infection. Option 4: This description would lead you to think of a bacterial cause. Question 7. A 50-year-old female diabetic patient presents to your urgent care center complaining of chest discomfort. Which of the following symptoms would lead you to believe her chest pain is related to gastroesophageal reflux rather than a cardiac etiology? 1. Chest pain radiates to the jaw and left arm. 2. Chest pain is worse with walking. 3. Chest pain started while eating spicy food. 4. Chest pain is associated with nausea. Rationales Option 1: Chest pain associated with radiation to the jaw or arm is typical of angina; reflux pain does not radiate to the arm or jaw. Option 2: Chest pain associated with exercise is concerning for angina. Option 3: Angina is typically not associated with eating. Option 4: A female diabetic can experience chest pain associated with nausea. Females and diabetics often experience atypical angina. Question 8. The ABCDEs of skin cancer can help you diagnose a cancerous skin lesion. Which of the following definitions does not accurately describe the corresponding letter of the acronym? 1. A = asymmetry. 2. B = boarder irregularity. 3. C = color. 4. D = diameter greater than 8 mm. Rationales Option 1: An asymmetric lesion is more concerning for cancer than a symmetric lesion. Option 2: A mole with an irregular boarder is concerning for cancer. Option 3: Multiple colors in a skin lesion or mole would be concerning for cancer. Option 4: A diameter greater than 6 mm would be concerning for cancer. Question 9. A patient presents to your primary care office for a blood pressure check. You have recently started them on an antihypertensive medication. However, on physical exam, the patient continues to have an elevated blood pressure. Which of the following symptoms would not be concerning for a hypertensive crisis? 1. Chest pain. 2. Visual changes. 3. Tinnitus. 4. Severe headache. Rationales Option 1: Chest pain associated with elevated blood pressure is concerning for a myocardial infarction or aortic dissection. Option 2: Visual changes are concerning for a hypertensive crisis. Option 3: Tinnitus is nonspecific for end organ damage associated with a hypertensive crisis. Option 4: Severe headache and elevated blood pressure are concerning for a subarachnoid hemorrhage or impending hemorrhagic stroke. Question 10. Which of the following headache descriptions does not match the accompanying diagnosis? 1. Worst headache of your life = subarachnoid hemorrhage. 2. Headache associated with aura = migraine headache. 3. Unilateral temporal headache with associated pulsatile sensation = tension headache. 4. Headache associated with loss of consciousness = intracranial pressure change. Rationales Option 1: A “thunderclap” headache, or worst headache of your life, is how many describe the headache associated with a subarachnoid hemorrhage. Option 2: Many patients experience an aura or predictable neurologic symptoms prior to migraine headaches. Option 3: Typically, unilateral pulsatile headaches of the temporal area are associated with temporal arteritis. Tension headaches are bilateral. Option 4: A headache associated with a loss of consciousness is associated with an infection, tumor, or brain bleed due to a change in intracranial pressure. Question 11. In which of the following scenarios is antibiotic treatment necessary following an injury to the skin? 1. Laceration requiring sutures from a knife cut while cutting an avocado. 2. Laceration requiring sutures on the foot of a diabetic. 3. Laceration requiring sutures on the hand of someone who has not had a tetanus booster in the last 5 years. 4. “Road rash” abrasion from a bicycle accident. Rationales Option 1: This is a clean laceration; the patient does not need antibiotics. Option 2: The foot in general gets poor blood supply and has a high risk of poor wound healing. Being a diabetic also increases the risk of poor wound healing. Antibiotics should be given in this scenario. Option 3: Tetanus status has no bearing on whether antibiotics should be prescribed. Option 4: Antibiotics are not necessary unless the abrasion is contaminated. Normally, road rash can be treated with topical antibiotic dressings. Question 12. A 20-year-old female presents to your urgent care clinic complaining of a cat bite. The patient recently adopted a cat. She was playing with the cat yesterday when the cat bit her on the arm. What antibiotic should be prescribed to prevent infection? 1. Augmentin. 2. Amoxicillin. 3. Bactrim. 4. No antibiotic is necessary. Rationales Option 1: Cat bites have a high risk of infection. Augmentin should be prescribed to reduce the risk of Pasteurella infection. Option 2: Amoxicillin will not prevent the Pasteurella infection that could be caused by a cat bite. Option 3: Bactrim will not prevent the Pasteurella infection that could be caused by a cat bite. Option 4: In all cat bites, antibiotic prophylaxis is necessary. Question 13. Which of the following patients needs a computed tomography (CT) scan of the head following a minor head trauma? 1. A patient taking Coumadin. 2. A child under the age of 5. 3. A patient complaining of a headache. 4. A patient with nausea following a head injury. Rationales Option 1: Aspirin therapy alone does not necessitate a CT scan, but use of Plavix or Coumadin does. Option 2: CT scans should be avoided, especially in children, unless absolutely necessary to limit radiation exposure. Reasons for a CT include loss of consciousness, altered mental status, and clear neurologic dysfunction. Option 3: A headache alone is not sufficient to warrant a CT scan following a minor head injury. Option 4: Nausea alone is not sufficient to warrant a CT scan following a minor head injury Following a sprain/strain injury, the PRICE acronym is helpful in treating a patient’s symptoms. Which of the following does not correspond to the PRICE acronym? 1. Rest. 2. Ice. 3. Crutches. 4. Elevation. Rationales Option 1: Resting the injured area will make the pain better and aid in healing. Option 2: Icing the injured area for short periods of time helps decrease swelling and pain. Option 3: While the use of crutches can help with a lower extremity injury, the C in this acronym stands for compression. Option 4: Elevating the injured area will help decrease swelling. Question 15. According to the Glasgow Coma Scale (GCS), what is the number assigned to someone with a normal level of consciousness? 1. 15. 2. 12. 3. 18. 4. 20. Rationales Option 1: A normal GCS score is 15. Less than 7 is considered a coma. Option 2: A normal GCS score is 15. Less than 7 is considered a coma. Option 3: A normal GCS score is 15. Less than 7 is considered a coma. Option 4: A normal GCS score is 15. Less than 7 is considered a coma. Question 16. What bacterium causes Rocky Mountain spotted fever? 1. Rickettsia rickettsii. 2. Borrelia burgdorferi. 3. Orientia tsutsugamushi. 4. Centruroides exilicauda. Rationales Option 1: This is the bacterium that causes Rocky Mountain spotted fever; it is transmitted by dog ticks. Option 2: This bacterium causes Lyme disease and is transmitted by deer ticks. Option 3: This bacterium can cause scrub typhus and is transmitted by chiggers. Option 4: This is a species of scorpion; it is not a bacterium. A patient presents to the emergency department by ambulance directly from football practice. The patient is tachycardic, tachypneic, and hypotensive. Their skin is hot and dry, and their core body temperature is 104°F. What is this patient’s diagnosis? 1. Heat stroke. 2. Heat exhaustion. 3. Heat syncope. 4. Heat cramps. Rationales Option 1: This patient has a core body temperature of 104°F; this is the diagnostic criterion that differentiates heat stroke from heat exhaustion. Option 2: In heat exhaustion you would expect the patient to have a lower temperature and be compensating more through sweating and hypertension. Option 3: This refers to a syncopal event due to heat. Option 4: This refers to cramps associated with heat or dehydration. The remodeling of a scar can take how long? 1. 6 months. 2. 3 months. 3. 1 month. 4. 2 weeks. Rationales Option 1: The remodeling of a scar can take up to 6 months, and many surgeons wait 6 months prior to revising a scar due to pain or deformity. Option 2: The remodeling of a scar can take up to 6 months. Option 3: The remodeling of a scar can take up to 6 months. Option 4: The remodeling of a scar can take up to 6 months. How soon after a facial laceration should sutures be removed? 1. 4 to 6 days. 2. 3 to 4 days. 3. 7 to 10 days. 4. 10 to 14 days. Rationales Option 1: Generally, you want to remove facial laceration sutures 4 to 6 days after the injury to avoid excessive scarring. Option 2: At 3 to 4 days after the injury you risk dehiscence of the wound. Option 3: Seven to 14 days after a facial laceration is too long to leave sutures in; the scar would likely be prominent. Option 4: Seven to 14 days after a facial laceration is too long to leave sutures in; the scar would likely be prominent. A patient on which of the following medications is at risk of sun poisoning? 1. Doxycycline. 2. Lexapro. 3. Labetalol. 4. Percocet. Rationales Option 1: Many other antibiotics, including amoxicillin and Bactrim, can also cause sun sensitivity. Option 2: Sun sensitivity is not a side effect of selective serotonin reuptake inhibitors (SSRIs). Option 3: Sun sensitivity is not a side effect of beta blockers Option 4: Opioid medications do not cause sun sensitivity. A 5-year-old male presents to your urgent care clinic with his mother. The patient was sent home from school for eye redness. He has bilateral erythema of his conjunctivae and watery drainage. He complains of pruritus of both eyes. The patient also has clear drainage from his nose and a sore throat. On physical exam of his throat, you notice erythema of his tonsils but no exudates. What is the most likely cause of the patient’s symptoms? 1. Adenovirus. 2. Rhinovirus. 3. Streptococcus pneumoniae. 4. Moraxella catarrhalis. Rationales Option 1: This is the most common cause of viral conjunctivitis. Option 2: This is a common cause of viral illness but is not the most common cause of viral conjunctivitis. Option 3: This is a bacterial cause of pneumonia and other upper respiratory problems. Option 4: This is a bacterial cause of many issues, including respiratory and sinus problems, but is not the most common cause of viral conjunctivitis. [Page reference: 1201] Question 2. A 70-year-old female presents to your emergency department complaining of right hip pain. She cannot ambulate. She had a fall in the bathroom of her house and had to be picked up by the emergency medical technicians (EMTs) her husband called. She complains of groin pain. What would you expect to see on physical exam? 1. Normal leg length on both sides. 2. A shortened and externally rotated right leg. 3. A shortened and internally rotated right leg. 4. A shortened and externally rotated left leg. Rationales Option 1: A patient with a right hip fracture normally has a shortened and externally rotated right leg. Option 2: This is the typical presentation of a right hip fracture. Option 3: This is the typical presentation of a dislocated hip; hip dislocations are rare. Option 4: The uninjured leg would not have an abnormal physical exam. [Page reference: 1204] A 20-year-old male presents to your urgent care center complaining of headaches for 2 weeks. The patient’s headaches are intermittent but severe and last 15 to 30 minutes. He has had 3 to 4 severe headaches in the last 2 weeks. He denies a past medical history and has had no recent trauma. The patient’s physical exam shows right-sided rhinorrhea, conjunctivitis, and facial swelling. What is the likely cause of the patient’s symptoms? 1. Brain tumor. 2. Cluster headaches. 3. Migraine headaches. 4. Sinus infection. Rationales Option 1: The patient has no neurologic changes that would suggest a brain tumor. Option 2: The patient’s symptoms are consistent with cluster headaches. Option 3: Migraine headaches normally have triggers; there is no noted trigger for this patient’s headaches. Option 4: The patient has no fevers and has episodic complaints that are more consistent with cluster headaches. [Page reference: 1205] Question 5. Which of the following is not associated with panic disorder? 1. Family history of mental health issues. 2. Female sex. 3. Smoking. 4. History of molestation as a child. Rationales Option 1: A family history of mental health issues increases the risk of developing panic disorder. Option 2: More women than men experience panic disorder. Option 3: Smoking does not increase the risk of panic disorder. Option 4: A history of a traumatic event increases the risk of developing panic disorder. [Page reference: 1205] A 20-year-old female presents to your urgent care clinic complaining of a cat bite. The patient recently adopted a cat. She was playing with the cat yesterday when the cat bit her on the arm. What antibiotic should be prescribed to prevent infection? 1. Augmentin. 2. Amoxicillin. 3. Bactrim. 4. No antibiotic is necessary. Rationales Option 1: Cat bites have a high risk of infection. Augmentin should be prescribed to reduce the risk of Pasteurella infection. Option 2: Amoxicillin will not prevent the Pasteurella infection that could be caused by a cat bite. Option 3: Bactrim will not prevent the Pasteurella infection that could be caused by a cat bite. Option 4: In all cat bites, antibiotic prophylaxis is necessary. [Page reference: 1222] Question 8. A patient presents to your urgent care office complaining of lightheadedness, dizziness, and problems concentrating following a motor vehicle accident 7 days ago. Originally, these complaints were more severe, but they have slowly decreased in intensity. What is the patient’s diagnosis? 1. Concussion. 2. Postconcussion syndrome. 3. Cerebral hemorrhage. 4. Malingering. Rationales Option 1: This patient likely suffered a concussion but is now experiencing symptoms consistent with postconcussion syndrome, which can last up to 6 months after a concussion. Option 2: The patient is experiencing postconcussion syndrome. If they have not received one since the car accident, you should refer them to their primary doctor or neurologist to get a computed tomography (CT) scan of the head. Option 3: There are no neurologic changes to suggest the patient is having a cerebral hemorrhage. Option 4: There are no signs of malingering in the patient’s complaint, and there is nothing to suggest secondary gain. [Page reference: 1231] When a patient is diagnosed with an epidermal hematoma, which artery is ruptured in the brain? 1. Middle meningeal artery. 2. Temporal artery. 3. Basilar artery. 4. Vertebral artery. Rationales Option 1: The middle meningeal artery is ruptured, causing an epidural hematoma. Option 2: The temporal artery is pathologic in the setting of temporal arteritis. Option 3: The basilar artery is not associated with an epidural hematoma. Option 4: The vertebral artery is not associated with an epidural hematoma. [Page reference: 1232] Ingestion of which of the following objects always requires surgical or endoscopic removal? 1. Button batteries. 2. Coins. 3. Sharp objects. 4. Food bolus. Rationales Option 1: Batteries must be removed; if the acid in the battery is exposed to the stomach mucosa, it can perforate it. Option 2: Most coins can be allowed to pass through the body, especially if they have passed through the pylorus. Option 3: Some sharp objects can be allowed to pass through the body, especially if they have passed through the pylorus. Option 4: If a food bolus is lodged in the esophagus, it can cause necrosis, but there are medical interventions that can be tried prior to surgery or endoscopy. [Page reference: 1245] If a patient has palsy of cranial nerve (CN) XI, what would they not be able to do? 1. Move the tongue. 2. Smell. 3. Move the neck. 4. Shrug the shoulders. Rationales Option 1: CN XII is responsible for tongue movement. Option 2: CN I is responsible for smell. Option 3: There is not a specific cranial nerve that is responsible for movement of the neck. Option 4: CN XI is the motor nerve that shrugs the shoulders. [Page reference: 1234] You are examining a patient in the emergency department following a closed head injury and you notice their pupils do not constrict when bright light is shone into them. To what cranial nerve would this suggest damage? 1. Cranial nerve (CN) III. 2. Cranial nerve (CN) II. 3. Cranial nerve (CN) IV. 4. Cranial nerve (CN) VI. Rationales Option 1: CN III controls pupillary response as well as upward and medial movement. Option 2: CN II controls visual acuity. Option 3: CN IV controls downward and medial eye movement. Option 4: CN VI controls lateral eye movement. [Page reference: 1234] A patient presents to the emergency department. You are concerned they have taken too many aspirin. What would you expect to see on an arterial blood gas? 1. Metabolic acidosis. 2. Metabolic alkalosis. 3. A normal blood gas. 4. Respiratory acidosis. Rationales Option 1: Aspirin is a salicylate, or salicylic acid; you would see metabolic acidosis. Option 2: Aspirin is a salicylate, or salicylic acid; you would see metabolic acidosis. Option 3: Aspirin is a salicylate, or salicylic acid; you would see metabolic acidosis. Option 4: Aspirin is a salicylate, or salicylic acid; you would see metabolic acidosis. [Page reference: 1249] Following a diagnosis of heat stroke, the goal is to decrease the patient’s temperature to what number in the first hour? 1. 103°F. 2. 102°F. 3. 100°F. 4. 99°F. Rationales Option 1: The goal is to reduce the patient’s temperature to 102°F within the first hour of care. Option 2: The goal is to reduce the patient’s temperature to 102°F within the first hour of care. Option 3: If you decrease the temperature below 101°F, you risk hypothermia. Option 4: If you decrease the temperature below 101°F, you risk hypothermia. [Page reference: 1267] Question 15. Hypothermia is defined as a core body temperature less than? 1. 96°F. 2. 95°F. 3. 93°F. 4. 90°F. Rationales Option 1: Hypothermia is defined as a core body temperature less than 95°F. Option 2: Hypothermia is defined as a core body temperature less than 95°F. Option 3: Hypothermia is defined as a core body temperature less than 95°F. Option 4: Hypothermia is defined as a core body temperature less than 95°F [Page reference: 1267] Which joint of the hand is most susceptible to a “fight bite”? 1. Metacarpophalangeal (MCP) joint. 2. Distal interphalangeal (DIP) joint. 3. Proximal interphalangeal (PIP) joint. 4. Interphalangeal (IP) joint. Rationales Option 1: Most fight bites are the result of the other person’s teeth lacerating the metacarpophalangeal (MCP) joint during a punch to the mouth. Option 2: Most fight bites are the result of the other person’s teeth lacerating the metacarpophalangeal (MCP) joint during a punch to the mouth. Option 3: Most fight bites are the result of the other person’s teeth lacerating the metacarpophalangeal (MCP) joint during a punch to the mouth. Option 4: The IP joint of the hand is found only in the thumb and is not commonly injured during a fight. Most fight bites are the result of the other person’s teeth lacerating the metacarpophalangeal (MCP) joint during a punch to the mouth. [Page reference: 1220] A patient arrives via ambulance with a friend. The patient is not responsive. The friend states the patient was walking across an intersection when a car struck him. The patient was unresponsive for a moment and then got up from the ground and began talking to his friend. He then started to become more confused and eventually became unresponsive. What type of brain injury does the patient have? 1. Epidural hematoma. 2. Subdural hematoma. 3. Concussion. 4. Skull fracture. Rationales Option 1: This patient had a loss of consciousness associated with a lucid moment after a traumatic injury; this history is likely indicative of an epidural hematoma. Option 2: The typical mechanism of injury for a subdural hematoma is less traumatic. Option 3: The patient likely has a contusion of the brain, but his symptoms are concerning for a brain bleed. Option 4: The patient may have a skull fracture along with a brain bleed, but the skull fracture cannot be predicted by the above anecdote. [Page reference: 1232] Question 19. You are examining a patient in the intensive care unit (ICU) after a motor vehicle accident. The patient is intubated but responds to painful stimuli. What score would you give them on the motor portion of the Glasgow Coma Scale (GCS)? 1. 5. 2. 4. 3. 3. 4. 2. Rationales Option 1: For a score of 5, the patient would be able to localize the pain. Option 2: The patient responds to painful stimuli; therefore, the score on the motor portion of the GCS is 4. Option 3: For a score of 3, the patient would respond to pain with general flexion of the body. Option 4: For a score of 2, the patient would respond to pain with general extension of the body. [Page reference: 1234] Question 20. You are examining a patient in the intensive care unit (ICU) after a motor vehicle accident. The patient is intubated but opens his eyes in response to verbal commands. What score would you give him on the eye-opening portion of the Glasgow Coma Scale? 1. 4. 2. 3. 3. 2. 4. 1. Rationales Option 1: A score of 4 is for patients that open and close their eyes spontaneously. Option 2: A score of 3 is for patients that open their eyes to verbal commands. Option 3: A score of 2 is for patients that open their eyes to painful stimuli. Option 4: A score of 1 is for patients that do not open their eyes at all. [Page reference: 1234] Question 1. Which of the following patients would not be a candidate for outpatient treatment of influenza? 1. A 62-year-old male with a history of myocardial infarction 5 years ago. 2. A 2-year-old female with a history of a congenital heart defect. 3. A 72-year-old male without a past medical history. 4. A 15-year-old female with a history of recent outpatient knee surgery. Rationales Option 1: This patient has a history of a heart attack but is 5 years out; this does not necessitate an inpatient stay. Option 2: Patients less than 5 and older than 65, as well as immunocompromised patients, are at great risk of complications from influenza and would be best treated in an inpatient environment. Option 3: This patient is elderly but healthy and can be followed closely as an outpatient. Option 4: As long as she is otherwise healthy, recent knee surgery does not make the patient a candidate for an inpatient stay. A 70-year-old man presents to your urgent care clinic complaining of a cough. He states he has had the cough for 6 weeks. The cough is dry and nonproductive. He denies fevers, chills, and weight loss. The patient admits to a 50-year history of smoking 1 pack per day. His pulse oximetry is 93% on room air. His chest x-ray shows enlarged lung fields. What would you recommend as the next step in his care? 1. That he see his primary doctor to discuss a computed tomography (CT) scan of the chest. 2. Oxygen through a nasal cannula until the cough stops. 3. An albuterol inhaler. 4. A course of oral steroids. Rationales Option 1: The patient has a greater than 40-pack-year history of smoking; his chronic cough is concerning for lung cancer. A CT scan should be recommended. Option 2: This may help the patient if he is short of breath, but he is not complaining of this, and his oxygen saturation is average for a long-time smoker. Option 3: This is the treatment for asthma. Option 4: This is the treatment for acute lung issues, such as asthma exacerbation. Question 4. Which of the following is not a risk factor for the development of sinusitis? 1. Asthma. 2. Smoking. 3. Deviated septum. 4. History of tonsillectomy. Rationales Option 1: Asthma is a risk factor for sinusitis. Option 2: Smoking is a risk factor for sinusitis. Option 3: Having a deviated septum is a risk factor for sinusitis. Option 4: Having a history of a tonsillectomy does not increase the risk of sinusitis. What is the most common cause of a urinary tract infection? 1. Escherichia coli. 2. Enterococcus faecalis. 3. Klebsiella pneumoniae. 4. Proteus mirabilis. Rationales Option 1: This is the most common cause of a urinary tract infection. Option 2: This is a common cause of a urinary traction infection but is not the most common. Option 3: This is a common cause of a urinary traction infection but is not the most common. Option 4: This is a common cause of a urinary traction infection but is not the most common. A patient presents to your urgent care center for evaluation following a motor vehicle accident. On physical exam, the patient seems lethargic and has thin, clear nasal drainage. You also note bruising around the eyes and mastoid process. What injury did the patient sustain in the car accident? 1. Basilar skull fracture. 2. Epidural hematoma. 3. Subdural hematoma. 4. Concussion. Rationales Option 1: The drainage from the patient’s nose is cerebrospinal fluid. The bruising around the eyes and mastoid process is associated with basilar skull fracture, an emergency condition. Option 2: A computed tomography (CT) scan would be needed to diagnose this condition. Option 3: A computed tomography (CT) scan would be needed to diagnose this condition. Option 4: Concussions are not associated with cerebrospinal fluid leaks. [Page reference: 1232] A 25-year-old construction worker comes into the emergency department after a third-story fall at his construction site. The patient’s main complaint is bilateral heel pain. The patient’s x-rays confirm bilateral calcaneal fractures. Based on the patient’s mechanism of injury, what other part of the body should you x-ray? 1. Lumbar spine. 2. Bilateral shoulders. 3. Ribs. 4. Bilateral wrists. Rationales Option 1: Following a fall and calcaneal fracture, you need to be concerned about possible fracture of the lumbar spine and hips. The force from the fall can move through the body and cause other fractures. The patient may not complain of pain in these areas due to being distracted by the pain in both heels. Option 2: Shoulder fractures are not necessarily associated with calcaneal fractures. Option 3: Rib fractures are not necessarily associated with calcaneal fractures. Option 4: Wrist fractures are not necessarily associated with calcaneal fractures. As a general rule, when treating hypothermia, at what rate do you want to warm the patient’s temperature? 1. 1-2°C an hour. 2. 0-1°C an hour. 3. 0.5°C an hour. 4. 2-3°C an hour. Rationales Option 1: Generally, you want to warm a hypothermic patient’s body temperature 1-2°C an hour. Option 2: Generally, you want to warm a hypothermic patient’s body temperature 1-2°C an hour. Option 3: Generally, you want to warm a hypothermic patient’s body temperature 1-2°C an hour. Option 4: Generally, you want to warm a hypothermic patient’s body temperature 1-2°C an hour. GENDER-RELATED PROBLEMS A male patient presents to the clinic for evaluation of infertility. Subjectively, the patient complains of pain and fullness of the testes and states, “My testicles feel like a bag of worms.” On physical examination, the nurse practitioner notes tortuous veins posterior to and above the testes that extend up into the external inguinal ring. Based on the preceding assessment, the nurse practitioner refers the patient to surgery for a diagnosis of: 1. Hydrocele. 2. Orchitis. 3. Urethritis. 4. Varicocele. Rationales Option 1: A hydrocele is a collection of fluid around the testes, within the scrotum. Option 2: Orchitis is an acute inflammatory reaction of one or both testes secondary to infection. Option 3: Pain with urination is the main symptom of urethritis, which is inflammation of the urethra, commonly occurring as a result of infection by bacteria. Option 4: A varicocele is caused by abnormal venous dilation of the pampiniform plexus above the testes, which results in pain and swelling. A patient often describes the sensation as feeling like a “bag of worms.” [Page reference: 772-773] Bill, age 43, appears with a tender, ulcerated, exudative, papular lesion on his penis. It has an erythematous halo, surrounding edema, and friable base. The nurse practitioner should suspect: 1. A chancre. 2. A chancroid. 3. Condylomata acuminata. 4. Genital herpes. Rationales Option 1: A chancre is a small papular lesion that enlarges and undergoes superficial necrosis to produce a sharply marginated ulcer on a clean base; it is the lesion of primary syphilis. Painless. Option 2: A chancroid is a tender, ulcerated, exudative, papular lesion with an erythematous halo, surrounding edema, and friable base. It is caused by inoculation of Haemophilus ducreyi through tiny breaks in epidermal tissue. Option 3: Condylomata acuminata (genital warts) range from pinhead-size papules to cauliflower-like groupings of skin-colored, pink, or red lesions. They are caused by human papillomavirus (HPV) infection of the epithelial cells. Option 4: Genital herpes simplex virus appears as erythematous plaques that develop into vesicular lesions that may become pustular. [Page reference: 780] Milton, a 72-year-old unmarried, sexually active white man, presents to your clinic with complaints of hesitancy, urgency, and occasional uncontrolled dribbling. Although the nurse practitioner suspects benign prostatic hyperplasia, what else should the differential diagnosis include? 1. Antihistamine use. 2. Urethral stricture. 3. Detrusor hyperreflexia. 4. Renal calculi. Rationales Option 1: Antihistamine use generally will result in hesitancy and urinary retention but not incontinence. Option 2: Urethral strictures may develop as a result of sexually transmitted diseases and should be considered in sexually active individuals (primarily males) no matter their age. Option 3: Detrusor hyperreflexia involves urge incontinence, characterized by a strong, sudden urgency (not hesitancy), followed immediately by a bladder contraction, resulting in an involuntary loss of urine. Option 4: Renal calculi commonly present as pain. [Page reference: 751] Herb, a 47-year-old with diabetes, is complaining of a rash on his penis. Before examining him, you suspect that he may have: 1. Tinea cruris. 2. Genital herpes. 3. Candida. 4. Intraepithelial neoplasia. Rationales Option 1: Tinea cruris is a fungal infection of the groin that appears as erythematous plaques whose scaling, papular lesions have sharp margins and occasionally clear centers. Option 2: Genital herpes is caused by skin-to-skin contact with the herpes simplex virus. It causes epidermal degeneration and erythematous plaques; the plaques develop into vesicular lesions that may become pustular. Option 3: A Candida infection is fairly common in clients with diabetes. Candida on the penis appears as multiple discrete, flat pustules with slight scaling and surrounding edema. It is a superficial mycotic infection that occurs in moist cutaneous sites. Other predisposing factors may include moisture, antibiotic therapy, and immunosuppression. Option 4: Intraepithelial neoplasia is associated with chronic human papillomavirus (HPV) infection and presents as multiple red, maculopapular, plaque-like lesions on the glans and inner aspect of the foreskin. [Page reference: 168-169] Question 5. Roger, a healthy 68-year-old man, comes in to see you with a complaint of sudden episodes of an urgent need to void. He has had several episodes of moderate amounts of unintentional urine loss during these times. Other than these episodes, he is voiding in amounts “normal” for him, with no leakage when he coughs or sneezes. The practitioner’s initial diagnosis is which type of incontinence? 1. Stress incontinence. 2. Urge incontinence. 3. Overflow incontinence. 4. Mixed incontinence. Rationales Option 1: Stress incontinence is a result of failure of the urethral sphincter, producing leakage with maneuvers that increase intra-abdominal pressure, such as coughing or sneezing. Option 2: Urge incontinence results from overactive bladder (OAB) and has many possible etiologies, each of which causes the bladder muscle (detrusor) to contract spontaneously. Common non-neurogenic causes are bladder irritants, infections, medications, urethritis, and pelvic tumors. Neurogenic causes involve the loss of cortical inhibition of the voiding reflex; this is seen in conditions such as dementia, Parkinson disease, multiple sclerosis, and stroke, ie, cerebrovascular accident (CVA) or brain attack. Option 3: Overflow incontinence is the result of leakage that occurs when the bladder is distended due to states such as a spinal cord injury (causing abnormal innervation between the bladder and spinal cord) or obstruction from a large prostate or tumor. Option 4: Mixed incontinence is a combination of stress and urge incontinence. [Page reference: 630] Josh and Martha have 5 children and do not want any more. Josh said he heard about a no-scalpel vasectomy (NSV) and asks the nurse practitioner how it works. What would be the best response? 1. “For the vasectomy to be permanent, you must have the vas deferens excised.” 2. “It’s safer for Martha to be sterilized.” 3. “A loop of vas deferens is delivered through the scrotal skin and occluded.” 4. “The testes are twisted, which occludes the vas deferens.” Rationales Option 1: NSV does result in permanent sterility. Option 2: Because there is no incision, NSV is safer for a man than tubal ligation is for a woman. Minimal postoperative complications include swelling, bruising, and pain in the scrotal area. Option 3: NSV is a method by which a loop of vas deferens is delivered through the scrotal skin for occlusion. A hemostat is used to puncture the scrotal skin, and then the skin is gently stretched to create an opening, which speeds the procedure and avoids the need for cutting through tissue. Postoperative complications are minimized this way. Option 4: NSV is a method by which a loop of vas deferens is delivered through the scrotal skin for occlusion. [Page reference: 678] Question 7. Harry, age 60, has benign prostatic hyperplasia and complains of some incontinence. The nurse practitioner’s first step in diagnosing overflow incontinence would be to order a: 1. Urinalysis. 2. Cystometrogram. 3. Cystoscopy. 4. Postvoid residual (PVR) measurement. Rationales Option 1: A urinalysis is commonly performed to confirm the cause and diagnosis, but a postvoid residual (PVR) measurement is the most important component of the diagnosis. Option 2: A cystometrogram is commonly performed to confirm the cause and diagnosis, but a postvoid residual (PVR) measurement is the most important component of the diagnosis. Option 3: A cystoscopy is commonly performed to confirm the cause and diagnosis, but a postvoid residual (PVR) measurement is the most important component of the diagnosis. Option 4: The first step in diagnosing overflow incontinence is to perform a PVR measurement. Clients with overflow incontinence cannot empty their bladders completely, so residual urine remains after voiding and this measurement is elevated. [Page reference: 631] Question 8. Michael complains of a urinary tract infection (UTI). Which of the following is a risk factor for UTIs in men? 1. A history of circumcision. 2. A history of testicular torsion. 3. Unprotected anal sex. 4. Presence of an inguinal hernia. Rationales Option 1: Lack of circumcision is a risk factor. Option 2: Testicular torsion, a surgical emergency, is not a risk factor. Option 3: Young men can develop a UTI similar to the type of uncomplicated UTI seen in women, which does not require any additional workup. Risk factors include unprotected anal sex, lack of circumcision, a history of prostatitis, unprotected intercourse with a woman who harbors pathogens in her vagina, and unprotected sex with men with AIDS who have a CD4 count less than 200 cells/µL. Option 4: Inguinal hernias have no documented impact on the development of urinary tract infections. [Page reference: 636] Question 9. Mikey had an undescended testicle at birth, and at age 2 it remains in the inguinal region. His mother is afraid of surgery and asks for your advice. How do you respond? 1. “In many children, the testicle descends close to the sixth birthday.” 2. “Even with only one normal testicle, he will have normal development.” 3. “If it hasn’t descended by now, it probably won’t. He needs to have surgery by age six.” 4. “Don’t worry; it can remain in that position forever with no problems.” Rationales Option 1: In boys with undescended testes, less than 1% have their testes descend after the first year. Option 2: Orchiopexy needs to be performed before age 6 to promote normal spermatogenesis and hormone production, prevent tumor formation, and leave the testis in a location where it can be easily palpated. Option 3: In boys with undescended testes, less than 1% have their testes descend after the first year. Orchiopexy needs to be performed before age 6 to promote normal spermatogenesis and hormone production, prevent tumor formation, and leave the testis in a location where it can be easily palpated. Option 4: Testes that remain undescended by puberty should be removed. [Page reference: 774] Question 10. Reiter syndrome is a complication of: 1. Bacterial vaginosis. 2. Syphilis. 3. Chlamydia. 4. Gonorrhea. Rationales Option 1: Bacterial vaginosis seldom results in complications. Option 2: Syphilis may result in disseminated disease, but not Reiter syndrome. Option 3: Left untreated in women, Chlamydia infections may cause scarring in the uterine tubes, leading to infertility and ectopic (tubal) pregnancies. Option 4: Gonorrhea may precipitate Reiter syndrome (reactive arthritis). [Page reference: 783] Lynne, age 43, comes to your office in tears, stating that last night she had unprotected sex and forgot to take her birth control pill. She wants to know about the “morning-after pill.” You tell her: 1. “If your period does not start at the scheduled time, come back and see me.” 2. “I’ll go ahead and order the estrogen-only postcoital contraception pill.” 3. “I’ll go ahead and order the Yuzpe regimen.” 4. “I’ll refer you to a gynecologist.” Rationales Option 1: Waiting for menses to start is too late. Option 2: Estrogen-only pills are not used for postcoital contraception in the United States. Option 3: Emergency contraception, referred to as postcoital contraception, prevents pregnancy after unprotected sexual intercourse. It should ideally be taken within 72 hours after unprotected sex. The Yuzpe regimen consists of taking 2 birth control pills within 72 hours and 2 more 12 hours later. This is 75% effective. A medication for nausea should be taken before the birth control pills. Option 4: Referring the client to a gynecologist will take too long. [Page reference: 678] A 17-year-old female patient requests to start medroxyprogesterone acetate (Depo-Provera) injections as her method of birth control. She discloses that she has had 4 sexual partners in the past year. Her last menstrual period was 12 days ago, and she had unprotected intercourse 3 days ago. The appropriate management for this patient would be to: 1. Administer the injection today. 2. Advise her to use another method for now and return with her next menses. 3. Give the injection after a negative pregnancy test and tell her to use condoms for the next 7 days. 4. Give the injection and tell her to use a barrier method for 7 days. Rationales Option 1: The depot medroxyprogesterone acetate (DMPA) injection can be given as long as the woman is reasonably certain she is not pregnant: in this scenario, the patient had unprotected intercourse 3 days ago. Option 2: Depo-Provera is the most commonly used injectable contraceptive and contains depot medroxyprogesterone acetate (DMPA). The DMPA injection can be given any time during the menstrual cycle as long as the woman is reasonably certain she is not pregnant (World Health Organization). If DMPA is given within 5 to 7 days of a normal last menstrual period, no backup is needed. If it is given after the seventh day in the menstrual cycle, backup contraception is needed. Option 3: In this scenario, the patient had unprotected intercourse 3 days ago, so a pregnancy test would not provide accurate results at this time. Option 4: The depot medroxyprogesterone acetate (DMPA) injection can be given as long as the woman is reasonably certain she is not pregnant: in this scenario, the patient had unprotected intercourse 3 days ago. [Page reference: 676] Marcia, age 59, presents with depression. According to the diagnostic criteria for major depressive disorder (MDD), which of the following must be present? 1. The depression must be related to alcohol, drugs, medication side effects, or physical illness. 2. The symptoms must be severe enough to upset the client’s daily routine, impact his or her work, or interfere with relationships. 3. The depression must be a normal reaction to the death of a loved one. 4. There must be some type of sleep disturbance. Rationales Option 1: The depression should not have a specific cause, such as alcohol, drugs, medication side effects, or physical illness. Option 2: For a diagnosis of MDD to be made, the symptoms must be severe enough to upset the client’s daily routine, impact his or her work, or interfere with relationships. Option 3: The depression should not be just a normal reaction to the death of a loved one. Option 4: Sleep disturbance may be a symptom of depression, but the diagnostic criteria for MDD can be met even if sleep disturbance is not present. [Page reference: 1103-1104] Julia, age 60, asks you about taking alendronate (Fosamax). What do you tell her about using this medication? 1. “If you decide to take it, stick with a lower dose of 5 mg because the side effects are much worse with a 10-mg dose.” 2. “Fosamax works better in younger women, so you should start this now rather than wait until you’re 70.” 3. “You should take a daily dose because the weekly dose is not as effective.” 4. “In addition to its efficacy in the treatment of osteoporosis in postmenopausal women, it is also useful for the prevention of osteoporosis.” Rationales Option 1: The risk of side effects is similar for doses of 5 mg, 10 mg, or greater. Option 2: Fosamax appears to be equally effective in older and younger postmenopausal women with osteoporosis. Option 3: Weekly administration of Fosamax is as effective as daily dosing. Option 4: In addition to its efficacy in the treatment of osteoporosis in postmenopausal women, alendronate (Fosamax) is useful for the prevention of osteoporosis. [Page reference: 868-869] Sharon, age 37, states she has heard that douching effectively washes out sperm after intercourse and that she has been using this as a method of birth control. Which of the following statements about douching is true? 1. Douching prevents sperm from entering the uterus. 2. If not used after intercourse, douching should be used at least once a month after menses. 3. Douching is a reliable contraceptive. 4. Douching may increase the risk of ectopic pregnancy. Rationales Option 1: Douching will not prevent sperm from entering the uterus because sperm may enter the cervical canal as soon as 15 seconds after ejaculation. Douching may even enhance the movement of sperm up the canal because it washes fluids deeper into the vagina and washes away the protective mucus. Option 2: Most gynecologists do not recommend douching at any time. Option 3: Douching is not a reliable contraceptive. Option 4: Douching has been associated with an increased risk of pelvic infection and ectopic pregnancy. [Page reference: 683] The Joneses are thinking about going for infertility counseling because they have been married for 5 years and have been unable to conceive. They ask you whether the man or the woman is usually the cause of infertility. What do you tell them about the etiology of infertility? 1. “In most cases, infertility is related to a female factor.” 2. “In most cases, infertility is related to a male factor.” 3. “In the majority of cases, the etiology cannot be identified.” 4. “Male and female infertility rates are almost the same in the majority of cases.” Rationales Option 1: Male and female infertility rates are almost the same in the majority of cases. Option 2: Male and female infertility rates are almost the same in the majority of cases. Option 3: The etiology of infertility can be identified in 90% of couples. Option 4: Male and female infertility rates are almost the same (males 30% and females 35%), and in 20% of cases there is a combination of male and female factors. It should also be stressed that infertility is no one’s “fault” and that many things may be tried to assist the couple with conception. [Page reference: 704-705] During a pelvic examination, you ask Mrs. Krane, age 54, to perform a Valsalva maneuver (ie, forcibly exhaling while keeping the mouth and nose closed). While doing this strain maneuver, a pouching is seen on the anterior wall of the vagina. This is indicative of: 1. A cystocele. 2. A rectocele. 3. An enterocele. 4. Uterine prolapse. Rationales Option 1: A cystocele is the prolapse into the vagina of the anterior vaginal wall and bladder. Clinically, a pouching is seen on the anterior wall as the client strains. Option 2: A rectocele is the prolapse into the vagina of the posterior vaginal wall and rectum and is seen on the posterior wall as the client strains. Option 3: An enterocele is the herniation of the pouch of Douglas into the vagina and is seen as a bulge emerging from the posterior fornix. Option 4: In first-degree uterine prolapse, the cervix appears at the introitus when the client strains. [Page reference: 681] . A 26-year-old female comes to your office to discuss birth control options. Her history includes migraine headaches with aura while on combination oral contraceptives in the past. She does not want to become pregnant. Which of the following birth control options would be the best choice for her? 1. Combined hormonal contraceptive pills. 2. Birth control patch (Ortho Evra). 3. Mirena intrauterine device (IUD). 4. Vaginal contraceptive ring (NuvaRing). Rationales Option 1: Combined hormonal contraceptive pills contain an estrogen hormone that may cause migraine headaches with aura. Option 2: The birth control patch (Ortho Evra) contains an estrogen hormone that may cause migraine headaches with aura. Option 3: Women who have a history of migraine headaches with aura on combination oral contraceptives should not take estrogen-containing contraceptives. The Mirena IUD contains levonorgestrel, which is a progestin. Option 4: The vaginal contraceptive ring (NuvaRing) contains an estrogen hormone that may cause migraine headaches with aura. [Page reference: 670] Question 19. Laura, age 65, presents to the office complaining of vaginal bleeding for 12 days. She reports she has not had a menses since the age of 52 and that this bleeding looks much like her menses used to look. The initial workup for postmenopausal bleeding (PMB) should include: 1. A pregnancy test. 2. A diagnostic pelvic laparoscopy. 3. An endometrial biopsy. 4. A colonoscopy. Rationales Option 1: A pregnancy test is not appropriate, as women who are postmenopausal are no longer fertile. Option 2: A diagnostic pelvic laparoscopy does not evaluate the internal cavity of the uterus. It examines the external surface of the uterus and surrounding organs. Option 3: Postmenopausal uterine bleeding is defined as uterine bleeding after permanent cessation of menstruation resulting from loss of ovarian follicular activity. In order to exclude or diagnose endometrial cancer, a woman with postmenopausal bleeding needs an initial evaluation with an endometrial biopsy to obtain a sampling of the uterine lining. Option 4: A colonoscopy evaluates the inside of the colon, not the uterine cavity. [Page reference: 739-740] Question 20. Ms. Russo, age 59, is a postmenopausal woman who comes to your office complaining of vaginal dryness. She has been sexually active with a new partner and says that penetration is very painful. She has tried over-the-counter lubricants but has not found one that works. Her history is significant for hypertension only. After examination and wet prep, you diagnose her with atrophic vaginitis. The treatment for this condition includes: 1. Terconazole cream. 2. Metronidazole gel. 3. Vaginal estrogen cream. 4. Hydrocortisone ointment. Rationales Option 1: Terconazole cream is used to treat vaginal yeast infections. Option 2: Metronidazole gel is used for the treatment of bacterial vaginosis. Option 3: Vaginal dryness usually results from a hypoestrogenic environment in the vagina and is very common in both premenopausal and postmenopausal women in the absence of a secondary vaginal infection. In addition to vaginal dryness, symptoms may include burning, itching, abnormal vaginal discharge, and dyspareunia. Hypoestrogenic states, such as lactation and the use of depot medroxyprogesterone acetate, can also cause atrophic vaginitis. This condition is best treated with topical estrogen. Option 4: Hydrocortisone ointment is used to treat inflammation and itching of the external surfaces of the body. [Page reference: 682] Question 1. Jeb, a 72-year-old male, is seen at your practice for follow-up of several episodes of orthostatic hypotension. After obtaining a review of systems and a digital rectal examination, it also appears that he has benign prostatic hyperplasia (BPH) with lower urinary tract symptoms. The nurse practitioner reviews his recent ultrasound evaluation, which reports a prostate volume of over 40 mL, and the results of the American Urological Association (AUA) symptom index for BPH, which shows his score to be 12. Based on the preceding information and the patient’s desire for noninvasive medical therapy, what management should he be offered? 1. Prazosin (Minipress). 2. Doxazosin (Cardura). 3. Finasteride (Proscar). 4. Phenoxybenzamine (Dibenzyline). Rationales Option 1: According to the current AUA guideline for the management of BPH, there is insufficient evidence to support the use of prazosin in the management of BPH with lower urinary tract symptoms. Option 2: Doxazosin is associated with a risk of orthostatic hypotension and thus would not be recommended in this client. Option 3: Finasteride is a 5-alpha-reductase inhibitor and is recommended for individuals with a prostate volume greater than 40 mL to help decrease its size. Option 4: According to the current AUA guideline for the management of BPH, there is insufficient evidence to support the use of phenoxybenzamine in the management of BPH with lower urinary tract symptoms. [Page reference: 752] A patient is being treated for erectile dysfunction. The patient is morbidly obese and is also being treated for cardiovascular disease and coagulopathy. Which of the following medications would be contraindicated? 1. Topical testosterone (AndroGel). 2. Sildenafil (Viagra). 3. Alprostadil (Caverject). 4. Subcutaneous pellet testosterone (Testopel). Rationales Option 1: AndroGel is a topical testosterone gel (hormone replacement therapy) and may transfer to a partner with close contact. Option 2: Sildenafil (Viagra) is an oral vasoactive agent and is contraindicated in patients taking nitrates or alpha blockers. Option 3: The vasoactive injectable alprostadil (Caverject) should not be used in patients with sickle cell anemia, coagulopathy, severe cardiovascular disease, myeloma, leukemia, morbid obesity, or penile deformity, fibrosis, or implants. Option 4: Testopel is a form of hormone replacement that is delivered as an implantable pellet and must be implanted in the subcutaneous tissue every 3 to 4 months. [Page reference: 766] Tommy, age 15, comes to the clinic in acute distress with “belly pain.” When obtaining his history, the nurse practitioner (NP) finds that he fell off his bike this morning and has vomited. Upon closer examination, the NP determines the belly pain to be left-sided groin pain or pain in his left testicle. He is afebrile and reports no dysuria. Which of the following diagnoses is most likely? 1. Testicular torsion. 2. Epididymitis. 3. Hydrocele. 4. Varicocele. Rationales Option 1: Testicular torsion is a condition in which the testis twists on the spermatic cord, thereby compromising blood flow to the testicle. This is a surgical emergency. Examination usually reveals a tender scrotal mass high in the hemiscrotum, and there is frequently a reactive hydrocele around the testis, obscuring anatomical detail. The scrotum can become erythematous and edematous. The cremasteric reflex is frequently blunted on the side of the torsion. Option 2: Epididymitis is usually accompanied by fever, as well as urethral discharge, and usually occurs in boys older than Tommy. Option 3: Although a hydrocele may develop secondary to testicular torsion, the intense discomfort and acute onset, accompanied often by nausea, distinguish the possibility of testicular torsion. Option 4: A varicocele, which usually occurs in young men, may cause pain but does not usually develop acutely. [Page reference: 770] Morris, age 52, is in a new relationship and is not sure whether his erectile dysfunction is organic or is caused by stress about his performance. What simple test could you suggest to determine if he has the ability to have an erection? 1. Nocturnal penile tumescence and rigidity (NPTR) test. 2. Penile duplex ultrasonography. 3. Intracavernous injection. 4. Serum prostate-specific antigen (PSA). Rationales Option 1: The NPTR test is a simple test the client may do at home by himself to determine if he has the ability to have a nocturnal erection, which would rule out an organic cause of erectile dysfunction. Have the client place the simple device on his penis before going to bed. This device records the frequency, as well as the rigidity, of erections. Option 2: A penile duplex ultrasonography can assess the penile arteries and diagnose a vascular cause of erectile dysfunction. It is less simple than the nocturnal penile tumescence and rigidity (NPTR) test. Option 3: An intracavernous injection can be used to test for an erection, thus ruling out vascular disease. This test is less simple than the nocturnal penile tumescence and rigidity (NPTR) test. Option 4: A PSA does not evaluate erectile dysfunction. [Page reference: 764] You are performing a school physical examination on Damon, age 5. You are unable to retract his foreskin over the glans while inspecting his penis. This is referred to as: 1. Phimosis. 2. Paraphimosis. 3. Microphallus. 4. Priapism. Rationales Option 1: An unusually long foreskin or a foreskin that cannot be retracted over the glans penis during physical examination is referred to as phimosis. It occurs in uncircumcised males and is normal in infancy. At Damon’s age, however, one should be able to retract the foreskin. He needs referral to a urologist. Option 2: Paraphimosis occurs when the foreskin is retracted and is unable to be returned to the original position. The penis distal to the foreskin will usually become swollen and gangrenous. Option 3: A microphallus is a normally formed penis that is smaller in size than expected. Option 4: Priapism is a continuous and pathological erection of the penis that does not occur as a result of sexual desire. [Page reference: 764] Tim asks the nurse practitioner (NP) about returning to his normal sex life after surgery for benign prostatic hyperplasia. He should be told: 1. “You probably won’t be able to have an erection after surgery; we need to discuss alternative ways of lovemaking.” 2. “You need to wait several months after surgery to make sure the site has healed.” 3. “You may resume sexual activity 4 to 6 weeks after surgery.” 4. “You’ll have to ask the surgeon.” Rationales Option 1: This surgery does not interfere with the ability to have an erection. Option 2: Several months is longer than necessary to wait to resume sexual activity. Option 3: Within 4 to 6 weeks after surgery, it is safe to resume a full sex life, including intercourse. Before this time, the spasmodic contractions that occur in the prostatic urethra at the time of ejaculation could trigger delayed bleeding. After 6 weeks, the risk of delayed bleeding is very slight. However, if the area has not completely healed, there may be slight discomfort because of the spasms. Option 4: Many clients feel more comfortable talking to their primary care provider—with whom they have an established relationship—than to their surgeon, and when it comes to discussing when to resume sex after surgery for benign prostatic hyperplasia, the same holds true. They may not feel comfortable asking their surgeon; thus, they may resume sexual activity too early or wait an exorbitant amount of time. The NP is able to address this question, and it does not have to be discussed with the surgeon except in unusual instances. [Page reference: 752-753] Jeff, age 20, presents to the college health clinic with complaints of difficulty passing his urine and a discharge from his penis. Upon further investigation, you note that the discharge is urethral in origin. The most common cause of these symptoms in the young adult male population is: 1. Chronic prostatitis. 2. Prostatic abscess. 3. Acute prostatitis. 4. Prostatic hyperplasia. Rationales Option 1: Chronic prostatitis occurs in middle-aged males as a result of nonspecific prostatitis or a previous gonorrhea infection. Option 2: Prostatic abscess is a rare complication of acute bacterial prostatitis. Option 3: The etiology of urethral discharge or difficulty voiding can include acute and chronic prostatitis and prostatic abscess. Young adult males in their 20s usually have acute prostatitis from gonorrhea or other bacterial infections. Option 4: Older males may have prostatic hyperplasia or prostate cancer, and these diagnoses are not correlated with urethral discharge. [Page reference: 755] Bernard, age 59, presents to the emergency department with a diagnosis of priapism. Despite application of cold compresses and pain medications, relief is unsuccessful. What is the treatment of choice? 1. Terbutaline 0.25 mg subcutaneously. 2. Phenylephrine 0.3 to 0.5 mL injected into the corpora cavernosa. 3. Doxazosin 5 mg sublingually. 4. Lidocaine 1% via the glans. Rationales Option 1: Terbutaline is considered for refractory priapism of greater than 6 hours’ duration. Option 2: Phenylephrine (Neo-Synephrine) is the drug of choice for first-line treatment of low-flow priapism because the drug has almost pure alpha-agonist effects and minimal beta activity. In short-term priapism (less than 6 hours), especially drug-induced priapism, intracavernous injection of phenylephrine alone may result in detumescence. Option 3: Doxazosin is used to treat benign prostatic hyperplasia. Option 4: Lidocaine is not considered efficacious for relief of priapism. [Page reference: 763] Jennifer, age 42, presents for her well-woman examination, and you notice “dimpling” on her left breast. Your differential diagnosis includes: 1. Breast cancer. 2. Fibrocystic breast disease. 3. Paget disease. 4. Striae from recent dieting. Rationales Option 1: Dimpling on the breast is known as peau d’orange because it looks like an orange peel. It may indicate a tumor is pulling on and retracting the underlying breast tissue. Option 2: A fibrocystic breast disease nodule is benign, tender, and movable, and may be soft to firm. Option 3: Mammary Paget disease is cancer of the mammary ducts. Option 4: Striae are streaks of light-colored skin that occur after rapid skin stretching. [Page reference: 694] Question 10. Sandra, age 63, says that her previous doctor never discussed why he took her off hormone replacement therapy (HRT) 3 years ago. Her last menstrual period was at age 49, and she started HRT that same year. You share with her some of the results of the Women’s Health Initiative (WHI). Which statement regarding the study is true? 1. Estrogen plus progestin increased the risk of a cardiac event in apparently healthy women after 10 years of use. 2. Women on HRT have a higher risk of colorectal cancer. 3. Postmenopausal hormones do not actually prevent fractures of the hip. 4. Estrogen alone is associated with a greater risk of breast cancer than a combination of estrogen plus progestin. Rationales Option 1: The WHI found that estrogen plus progestin increased the risk of a cardiac event in apparently healthy women if used more than 10 years post menopause. Option 2: Users of postmenopausal hormones actually have a lower risk of colorectal cancer; the mechanisms by which hormone use might reduce risk are unclear. Option 3: The WHI was the first trial with definitive data supporting the ability of postmenopausal hormones to prevent fractures of the hip, vertebrae, and other sites. Option 4: Estrogen plus progestin appears to be associated with a greater risk of breast cancer than estrogen alone. [Page reference: 720-721] Question 11. Gerri, age 26, is thinking about getting pregnant in the near future, and she asks you what kind of vitamins she should be taking. You respond: 1. “You don’t need any additional supplementation to your diet until you become pregnant.” 2. “You need to increase your iron level by taking one 325-mg iron tablet twice a day.” 3. “You need to start taking an extra 1500 mg of calcium a day now to help promote bone growth of the fetus when you become pregnant.” 4. “All women capable of becoming pregnant should take a daily supplement of 0.4 mg of folic acid a day to reduce the risk of neural tube defects.” Rationales Option 1: Most women need a daily supplement of 0.4 mg of folic acid because they are unable to attain this level in their daily diet. Option 2: Further iron supplementation is not recommended and can be potentially harmful. Option 3: Further calcium supplementation is not recommended and can be potentially harmful. Option 4: Studies have shown a significant reduction in neural tube defects (spine and brain issues) in women who take at least 0.4 mg of folic acid per day. Supplementation is usually needed, as most women are unable to attain this level of folic acid in their daily diet. [Page reference: 714] Question 12. Which of the following terms describes the bluish or purplish discoloration of the vulva, vagina, and a portion of the cervix that occurs in pregnancy? 1. Goodell sign. 2. Hegar sign. 3. Piskacek sign. 4. Chadwick sign. Rationales Option 1: The cervix is normally firm; however, at 6 weeks of pregnancy, there is marked softening of the cervix that is referred to as Goodell sign. Option 2: Hegar sign is the softening of the lower uterine segment between the body of the uterus and the cervix. It can be felt on bimanual exam. Option 3: Piskacek sign is the asymmetric enlargement of the body of the pregnant uterus due to implantation. Option 4: In early pregnancy, increased vascularity, blood supply, and congestion of the cervix cause a bluish discoloration that is referred to as Chadwick sign. [Page reference: 690] Question 13. On physical exam, Judy has pubic hair that spreads over her mons pubis with a slight lateral spread. In addition, her breast development shows breast enlargement with secondary mound formation by the developing areolae. Which Tanner stage best describes Judy’s development? 1. Stage II. 2. Stage III. 3. Stage IV. 4. Stage V. Rationales Option 1: In stage II, pubic hair growth is sparse and mostly on the labia. The hair is long and downy, slightly pigmented, and straight or only slightly curly. Option 2: In stage III, the pubic hair is sparse and spreads over the mons pubis. The hair is darker, coarser, and curlier. Option 3: When a girl has adult-type pubic hair over a smaller area, with none on the medial thigh, she is in Tanner stage IV. Option 4: In stage V, pubic hair is adult in type and patterned as an inverse triangle. Hair also appears on the medial thigh surface. [Page reference: 724-725] Question 14. A vaginal pH of 4.2 is an expected finding in: 1. A healthy girl of prepubertal age. 2. A woman with Trichomonas vaginalis. 3. A postmenopausal woman with atrophic vaginitis. 4. A healthy woman of reproductive age. Rationales Option 1: The vaginal pH in prepubescent girls is usually above 7 due to lack of estrogen. Option 2: Women infected with Trichomonas vaginalis have a vaginal pH of greater than 4.5. Option 3: The vagina of a postmenopausal woman with atrophic vaginitis lacks estrogen and has a pH that is usually greater than 6. Option 4: In women of reproductive age, the normal vaginal discharge has a pH of 3.8 to 4.2. [Page reference: 687-688] Question 15. Joy has been breastfeeding and has developed puerperal mastitis. You tell her: 1. “Using cool compresses to the affected breast before pumping will increase milk expression.” 2. “Continue breastfeeding the baby to avoid milk stasis.” 3. “Continue doing your normal activities during the acute phase to keep things flowing.” 4. “Do not massage the breasts.” Rationales Option 1: Moist heat to the affected breast can be useful before feeding and pumping to increase milk expression. Cold compresses may be comfortable between feedings. Option 2: Puerperal mastitis is a cellulitis that develops in lactating and nonlactating breasts after parturition. It is vital that the infant continue to breastfeed during puerperal mastitis to avoid milk stasis. Because the infection is extraductal, continuing to breastfeed poses no risk to the infant. Antibiotic therapy is traditionally recommended to treat the infection. Option 3: Bed rest is imperative during the acute phase of the illness. The mother should be assisted with her household duties and rest in bed with her infant. Option 4: Massaging the breasts during feeding helps to better drain them, and additional pumping may be needed. [Page reference: 693] Question 16. A 27-year-old female presents to your office for a levonorgestrel-releasing intrauterine system (Mirena) insertion. She reports that her menses started 3 days ago and is normal. How soon after insertion will she be able to safely rely on the intrauterine device (IUD) for contraception? 1. Immediately. 2. After 48 hours. 3. In 1 week. 4. In 1 month. Rationales Option 1: The levonorgestrel-releasing intrauterine system (Mirena) is a small T-shaped contraceptive device that releases levonorgestrel (a type of progestin) directly into the endometrial cavity. Although this IUD can be inserted at any time during the menstrual cycle, Mirena is effective immediately if inserted within 7 days after the start of menses. Option 2: In this case, Mirena is effective immediately. Option 3: If Mirena is inserted later than 7 days after the start of menses, another method of birth control should be used during the first week after insertion. Pregnancy protection will begin after 7 days. Option 4: In this case, Mirena is effective immediately. [Page reference: 670] Question 17. Marsha, age 42, is having chronic abnormal uterine bleeding (AUB) and cannot take oral contraceptives (OCs) due to a history of a deep vein thrombosis (DVT). Management includes which medication? 1. Medroxyprogesterone. 2. Ethinyl estradiol (EE). 3. Conjugated estrogens. 4. Piroxicam. Rationales Option 1: Management of AUB is directed toward controlling bleeding and preventing recurrence. For women who cannot take OCs, medroxyprogesterone is offered. For teenagers, management includes observation for those with mild cases and no anemia, medroxyprogesterone for clients not sexually active, and OCs for those who are sexually active. For women of reproductive age, treatment is based on the woman’s desire for fertility or contraception. Option 2: OCs containing EE are used in acute bleeding episodes. Option 3: For women with severe acute bleeding who remain hemodynamically stable, conjugated estrogens are used until bleeding stops. Option 4: Piroxicam is used for dysmenorrhea. [Page reference: 676] Question 19. Julia is nursing her 8-week-old baby and states that he is very irritable and sleeps poorly. What medication or substance do you ask her if she is taking or using? 1. Cimetidine (Tagamet). 2. Ergotamine tartrate (Ergostat). 3. Nicotine. 4. Caffeine. Rationales Option 1: Cimetidine (Tagamet) and ranitidine (Zantac) are concentrated in breast milk and may suppress the infant’s gastric acidity and cause central nervous system stimulation. Option 2: Ergotamine tartrate (Ergostat) in doses sufficient to treat a migraine may cause vomiting, diarrhea, and convulsions, as well as suppress lactation. Option 3: Nicotine increases the incidence of respiratory disease in infants exposed to smoke. Option 4: Caffeine in large amounts will make the infant who is being nursed irritable and give him or her a poor sleep pattern. [Page reference: 1169-1170] Question 20. There are many causes of amenorrhea. In ballet dancers and marathon runners, which anatomical structure is the probable cause? 1. Outflow tract. 2. Ovary. 3. Anterior pituitary. 4. Hypothalamus. Rationales Option 1: The outflow tract is the cause in cases of developmental absence of the vagina or uterus, obstruction of the outflow tract, scarred uterine lining, or cervical obstruction. Option 2: The ovary is the cause of primary amenorrhea in cases involving congenital chromosomal abnormalities, such as Turner syndrome. Option 3: The anterior pituitary is the cause of amenorrhea resulting from prolactin-secreting tumors or nonfunctioning adenomas. Option 4: The hypothalamus is the cause of exercise-induced amenorrhea, such as occurs in dancers and marathon runners. [Page reference: 724] Question 1. A 63-year-old man presents to your office with hematuria, hesitancy, and dribbling. Digital rectal examination (DRE) reveals a smooth, moderately enlarged prostate. The client’s prostate-specific antigen (PSA) is 1.2. What is the most appropriate management strategy at this time? 1. Prescribing an alpha-adrenergic blocker. 2. Recommending saw palmetto extract. 3. Prescribing an antibiotic. 4. Referring the client to urology. Rationales Option 1: The patient’s symptoms appear to be related to benign prostatic hyperplasia (BPH) and not a urinary tract infection. An alpha-adrenergic blocker will relax the smooth muscle of the prostate and bladder, which will improve flow and relieve symptoms. Option 2: Saw palmetto extract is an alternative treatment that has not been proven to improve urinary symptoms in men with benign prostatic hyperplasia (BPH), and its long-term side effects have not been studied. Option 3: The patient does not seem to have a urinary tract infection. Option 4: Benign prostatic hyperplasia (BPH) can be successfully treated by primary health care providers, and a referral is not appropriate until standard treatment is no longer effective. [Page reference: 751-752] Question 3. Drew has erectile dysfunction and says that a friend told him about a method that uses a constricting ring around the base of the penis. What is he referring to? 1. Intracavernous injection therapy. 2. An external vacuum device. 3. Urethral suppositories. 4. Surgery. Rationales Option 1: Intracavernous injection therapy consists of injecting the vasoactive drug alprostadil (Caverject) directly into the corpus cavernosum of the penis, causing an erection that lasts 40 to 60 minutes. Option 2: An external vacuum device is a viable method for alleviating erectile dysfunction regardless of the cause of the disorder. A plastic cylinder is placed around the penis, a vacuum pump causes cavernosal engorgement, and a constrictor ring is applied around the base of the penis, allowing the client to hold an erection for 30 minutes. Option 3: Urethral suppositories such as alprostadil are also effective in causing an erection when inserted into the urethra after voiding. Option 4: Surgery involves inserting a penile prosthesis, of which there are many different types. [Page reference: 766] Question 4. John asks for a prescription for sildenafil (Viagra). He says that the only medication he takes is isosorbide mononitrate (Monoket) oral tablets and that he has diabetes that he controls with diet alone. What should the nurse practitioner tell him? 1. “Let’s try a sample and see how you do.” 2. “Viagra is contraindicated with isosorbide mononitrate; let’s discuss other options.” 3. “Because of your history of diabetes, we can’t use it.” 4. “I’d better refer you to a urologist.” Rationales Option 1: The use of sildenafil with Monoket is contraindicated. Option 2: Because sexual stimulation leads to the release of nitric oxide in the corpus cavernosum of the penis and sildenafil (Viagra) potentiates that release, there is a double hypotensive effect between sildenafil and a nitric oxide donor such as isosorbide mononitrate (Monoket, Imdur, and Ismo). Therefore, the use of sildenafil with Monoket is contraindicated. Option 3: Sildenafil is not contraindicated in clients with diabetes. Option 4: Sildenafil does not require referral to a urologist. [Page reference: 766] Question 6. Martin is complaining of erectile dysfunction. He also has a condition that has reduced arterial blood flow to his penis. The most common cause of this condition is: 1. Epilepsy. 2. Multiple sclerosis. 3. Diabetes mellitus. 4. Parkinson disease. Rationales Option 1: Temporal lobe epilepsy can cause erectile dysfunction. Option 2: Multiple sclerosis can cause erectile dysfunction. Option 3: About 50% of men who have had diabetes for longer than 6 years develop erectile dysfunction to some extent as a result of pathological changes in the vascular wall that lead to a reduction of arterial blood flow to the penis. Many other conditions can cause erectile dysfunction as well. These include cerebrovascular accidents (ie, strokes or brain attacks), spinal cord injury, temporal lobe epilepsy, multiple sclerosis, chronic obstructive pulmonary disease, angina, chronic renal failure, and Parkinson disease. Option 4: Parkinson disease can cause erectile dysfunction. [Page reference: 763] Question 7. The nurse practitioner is performing a rectal examination on James, age 46, for follow-up of melena. What would the provider expect his stool to look like if his condition has not resolved? 1. Grayish tan. 2. Bright red. 3. Pale yellow, greasy, and fatty. 4. Black and tarry. Rationales Option 1: Grayish tan stool is caused by obstructive jaundice. Option 2: Bright red stool results from rectal bleeding. Option 3: Pale yellow, greasy, fatty stool (steatorrhea) is caused by malabsorption syndromes such as cystic fibrosis or celiac disease. Option 4: Melena is black, tarry stool caused by upper gastrointestinal bleeding. [Page reference: 539-540] Question 14. Susan, age 28, is 9 months pregnant and asks you about using umbilical cord blood after birth for newborn DNA identification. What do you tell her? 1. “A cord blood sample is identical to the genetic profile of the infant.” 2. “The fingerprinting and footprinting that have been done in the past work perfectly.” 3. “A better method of obtaining newborn DNA is with a buccal swab.” 4. “The traditional method of using a newborn heel stick to obtain a DNA sample is tried and true.” Rationales Option 1: DNA is the only reliable source of newborn identification and is easily obtained from an umbilical cord blood sample at birth. Option 2: The American Academy of Pediatrics (AAP) and the American College of Obstetricians and Gynecologists (ACOG) stopped recommending the use of fingerprinting and footprinting for newborn identification purposes because children do not produce well-defined fingerprints until 4 to 6 years of age and newborn footprinting has been shown to be inaccurate. Option 3: A buccal swab is another possible method of obtaining newborn DNA, but if it is not processed or frozen soon after collection, degradation may occur. Many additional factors can influence buccal cell DNA as well. Option 4: A newborn heel stick is commonly used to obtain the blood specimen for a newborn DNA sample. However, because the procedure is painful and there are many potential complications that may result, some parents are hesitant to agree to it. [Page reference: 690] Question 15. The best method to diagnose uterine polyps is a: 1. Hysteroscopy. 2. Dilation and curettage. 3. Colposcopy. 4. Laparoscopy. Rationales Option 1: The best method to diagnose uterine polyps is a hysteroscopy. A hysteroscopy involves visualizing the endometrium through a scope to assess for some types of uterine fibroids, polyps, and structural abnormalities. Tissue sampling and removal of polyps can be done through the hysteroscope. Option 2: A dilation and curettage consists of scraping the walls of the uterus. Option 3: A colposcopy is used to examine the vulva, vagina, and cervix. Option 4: A laparoscopy examines the peritoneal cavity. [Page reference: 741] Question 16. Samantha, age 19, has a diagnosis of a Chlamydia vaginal infection. You believe it is questionable whether she will fill the prescription you write or take it for 7 days as ordered. What do you do? 1. Give azithromycin (Zithromax) 1 g orally now. 2. Emphasize the importance of the drug and tell her the consequences of not taking it. 3. Send out the public health nurse to follow up on whether she takes the drug for 7 days. 4. Assume that Samantha is an adult and will follow your instructions. Rationales Option 1: An appropriate first-line treatment for a Chlamydia vaginal infection is azithromycin (Zithromax) 1 g orally. Although doxycycline (Vibramycin) 100 mg orally twice daily for 7 days is the most tried and true and least expensive treatment, azithromycin is the most convenient option for single-dose administration. Azithromycin is contraindicated in pregnant women. For this population, erythromycin 500 mg orally 4 times a day for 7 days should be ordered. Option 2: Talking to the patient about the importance of the drug and the consequences of not taking it will not ensure she will take the drug. Option 3: While public health nurses in some states do directly observed therapy (DOT) for tuberculosis (TB) to make sure patients take their meds, they will not do this for a sexually transmitted disease (STD). Option 4: According to this scenario, you are not confident Samantha will follow your instructions. [Page reference: 783] Question 17. Which of the following ovarian tumors or cysts have the potential for malignancy? 1. Follicular cysts. 2. Brenner tumors. 3. Fibromas. 4. Secondary ovarian tumors. Rationales Option 1: Follicular cysts are frequent in the menstrual years, never occur in the postmenopausal years, and do not have a potential for malignancy. Follicular cysts often disappear after a 2-month regimen of oral contraceptives. Option 2: Brenner tumors account for about 1% of ovarian tumors, occur more than 50% of the time in postmenopausal women, and very rarely have the potential for malignancy. Option 3: Fibromas account for less than 5% of ovarian tumors and very rarely have the potential for malignancy. Option 4: Secondary ovarian tumors account for about 10% of fatal malignant diseases in women. They are usually the result of bowel or breast metastasis to the ovary, and all of the tumors are malignant. [Page reference: 742-744] Question 18. Joanne, age 23, wants to use some form of birth control, but because she is getting married next year, she wants to be able to stop the birth control method after the wedding and have her fertility restored almost immediately. Which method do you recommend for her? 1. Birth control pills. 2. Vaginal ring. 3. Depot medroxyprogesterone acetate (DMPA) injections. 4. Diaphragm. Rationales Option 1: There may be a temporary delay in conception after discontinuing oral contraceptives, although the exact time is unclear. Option 2: The vaginal ring (NuvaRing) releases synthetic hormones and is worn for 21 days and then removed for 7 days to allow for a menstrual period. The vaginal ring has side effects similar to those of oral contraceptives. Option 3: With DMPA, return to fertility may be delayed regardless of the duration of its use. DMPA injections are given every 3 months. Option 4: The diaphragm is the only product listed that does not contain hormones that may delay pregnancy after discontinuing use. The dome-shaped silicone device covers the cervix and allows secretions to exit without sperm coming in. The diaphragm is used with spermicide. [Page reference: 677] Question 19. Mrs. Thomas, age 69, comes to your office for the results of her dual-energy x-ray absorptiometry (DXA) scan. She reports that her mother had osteopenia before she died. The DXA scan results show a T score of -2.7 in the hip and -2.8 in the spine. You explain to her that her results show: 1. Normal bone density. 2. Osteopenia. 3. Osteoporosis. 4. Osteoarthritis. Rationales Option 1: A T score of -1 or above indicates normal bone density. Option 2: A T score between -1.0 and -2.5 is indicative of osteopenia. Option 3: The gold standard test for bone mineral density (BMD) is the DXA scan. The National Osteoporosis Foundation recommends that all women over the age of 65 get a DXA scan. The level of bone loss is determined by the T score, which compares the individual’s bone density to that of a 30-year-old adult. Peak bone density is reached by age 30 and should ideally be maintained at this level throughout life. A T score less than or equal to -2.5 is indicative of osteoporosis. Option 4: Osteoarthritis is a degenerative joint disease and does not result from changes in bone density. [Page reference: 865] Susan, age 35, has been diagnosed with fibrocystic breast disease. Which of the following may exacerbate the condition? 1. A daily dose of aspirin. 2. Spicy foods. 3. Chocolate. 4. Wearing tight bras. Rationales Option 1: Daily aspirin therapy has not been associated with fibrocystic breast disease. Option 2: Spicy foods have not been associated with fibrocystic breast disease. Option 3: There are many interventions that will alleviate the pain of fibrocystic breast disease as well as help decrease the proliferation of breast tissue. Coffee, chocolate, tea, colas, and drugs containing caffeine should be reduced to help alleviate breast tenderness and reduce nodularity. Vitamin E has been shown to decrease the pain and tenderness associated with fibrocystic change as well as the proliferation of breast tissue. Option 4: Wearing a tight-fitting bra does not exacerbate the condition but may increase tenderness. [Page reference: 679-680] Question 2. Regular testicular self-exams have not been studied enough to show if they lower the risk of dying from testicular cancer. This is why the American Cancer Society and other agencies do not have a recommendation about regular testicular self-exams for all men. Still, some practitioners do recommend that all men examine their testicles monthly after reaching puberty. If you are teaching a patient how to do a testicular self-examination, which of the following do you tell him? 1. “Examine your testicles when you are cold because this makes them more sensitive.” 2. “Make sure your hands are dry to create friction.” 3. “If you feel firmness above and behind the testicle, make an appointment.” 4. “Make an appointment if you note any hard lumps directly on the testicle, regardless of whether they are tender.” Rationales Option 1: Testicles should be examined with soapy hands when taking a warm shower or bath to allow easy manipulation of the tissue. Option 2: Testicles should be examined with soapy hands when taking a warm shower or bath to allow easy manipulation of the tissue. Option 3: If parts of the testicle above and behind feel rather firm, this is the epididymis and is normal. The spermatic cord, a small, round, movable tube, extends up from the epididymis and also feels firm and smooth. Option 4: If you advise men to perform a monthly testicular self-examination, instruct them to call if they notice any hard lumps directly on the testicle, regardless of whether the lumps are tender. [Page reference: 774] Question 3. Max, age 70, is obese. He is complaining of a bulge in his groin that has been there for months. He states that it is not painful, but it is annoying. The practitioner notes that the origin of the swelling is above the inguinal ligament and directly behind and through the external ring. This finding is most indicative of a(n): 1. Indirect inguinal hernia. 2. Direct inguinal hernia. 3. Femoral hernia. 4. Strangulated hernia. Rationales Option 1: An indirect inguinal hernia is congenital or acquired and is more common in infants younger than 1 year of age and men aged 16 to 25. The origin of the swelling is above the inguinal ligament. The hernia sac enters the canal at the internal ring and exits at the external ring. Option 2: A direct inguinal hernia usually occurs in middle-aged to older men and is the result of an acquired weakness caused by heavy lifting, obesity, or chronic obstructive pulmonary disease (COPD). The origin of the swelling is above the inguinal ligament and directly behind and through the external ring. Option 3: A femoral hernia, which occurs more frequently in women, is acquired and results from an increase in abdominal pressure as well as muscle weakness. The origin of the swelling is below the inguinal ligament. Option 4: Because Max is not having any pain and the condition has remained the same for months, you know the hernia is not strangulated. A strangulated hernia, which requires immediate referral to a surgeon, results in no blood supply to the affected bowel and causes nausea, vomiting, and tenderness. [Page reference: 576] Question 4. Jake, age 62, has a low International Prostate Symptom Score for lower urinary tract symptoms associated with his benign prostatic hyperplasia (BPH). The nurse practitioner should recommend: 1. No treatment at this time. 2. Immediate referral to urology. 3. Balloon dilation. 4. Starting an alpha blocker. Rationales Option 1: Asymptomatic clients with BPH rarely require treatment. Watchful waiting is an appropriate strategy for following the progression of the disease and the development of any complications. Option 2: Prostate surgery offers the best choice for symptom improvement. A transurethral resection of the prostate is the most commonly used surgical treatment for BPH. Option 3: Balloon dilation of the prostatic urethra has fewer complications than surgery but is not as effective in relieving symptoms. Option 4: Alpha blockers relax the bladder neck and prostate smooth muscle and offer relief for many clients, particularly in regard to nocturia. [Page reference: 751] A patient’s chief complaint is heaviness in the scrotum. The nurse practitioner notes swelling of the testes, along with warm scrotal skin. Which of the following diagnoses is most probable? 1. Cryptorchidism. 2. Orchitis. 3. Testicular torsion. 4. Epididymitis. Rationales Option 1: In cryptorchidism, one or both testicles are undescended. Option 2: Orchitis refers to an acute, painful onset of swelling of the testicle accompanied by warm scrotal skin. The client usually complains of a heavy feeling in the scrotum. It is typically unilateral but after 1 week may progress to the other testicle. Option 3: Testicular torsion is a twisting, or torsion, of the testis. The testicle is enlarged, retracted, in a lateral position, and extremely sensitive. The result is venous obstruction, secondary edema, and eventual arterial obstruction. It is a surgical emergency. Option 4: Epididymitis is caused by a retrograde spreading of pathogenic organisms from the urethra to the epididymis. It results in an indurated, swollen, and tender epididymis. The testicle is also usually enlarged and tender. [Page reference: 772] Temporal lobe epilepsy can cause erectile dysfunction. Sidney, age 72, presents to the clinic with complaints of a weak urine stream, hesitancy, and painful ejaculation. On digital rectal examination, you note that his prostate is boggy. The most common cause of his symptoms is: 1. Acute bacterial prostatitis. 2. Chronic bacterial prostatitis. 3. Chronic nonbacterial prostatitis with chronic pelvic pain syndrome. 4. Noninflammatory prostatitis. Rationales Option 1: Acute bacterial prostatitis primarily affects men aged 40 to 70. It manifests with sudden onset of symptoms, including fever, malaise, dysuria, low back pain, and/or perineal-prostatic pain. The prostate usually feels tender, warm, and edematous. Option 2: Chronic bacterial prostatitis, a recurrent bacterial infection of the prostate and urinary tract, affects men aged 50 to 80. It is accompanied by bladder obstruction symptoms such as weak urine stream, hesitancy or dribbling, hematuria, hematospermia, and/or painful ejaculation. The most common offending pathogen is Escherichia coli. Option 3: Chronic nonbacterial prostatitis with chronic pelvic pain syndrome is manifested by pelvic pain and may be accompanied by various complaints, such as mild low backache and hematospermia. It also may or may not include obstructive voiding symptoms. Option 4: Chronic nonbacterial prostatitis is subdivided into inflammatory and noninflammatory prostatitis. With noninflammatory chronic prostatitis, the prostate usually feels normal on examination. [Page reference: 754] You are referring a 73-year-old client for management of his prostate cancer with hormonal therapy. It is understood that goserelin acetate (Zoladex) acts as a method of androgen ablation by: 1. Blocking the release of follicle-stimulating hormone (FSH) and luteinizing hormone (LH). 2. Blocking 5-alpha-reductase, which converts testosterone into dihydrotestosterone. 3. Inhibiting the binding of testosterone to cancer cells. 4. Inhibiting the progression of cancer cells through the cell cycle. Rationales Option 1: Goserelin acetate (Zoladex) and leuprolide acetate (Lupron) block the release of FSH and LH. These preparations are administered intramuscularly. Option 2: Finasteride (Proscar) is used to block the enzyme 5-alpha-reductase, which converts testosterone into dihydrotestosterone. Option 3: Oral antiandrogen agents such as bicalutamide (Casodex) and flutamide (Eulexin) inhibit the binding of testosterone to cancer cells. Option 4: Chemotherapy, not hormonal therapy, exerts an effect on the cell cycle. [Page reference: 759] Question 9. Cynthia says that her health care provider wants to do a colposcopy. She asks you what this is. You tell her that a colposcopy: 1. Visualizes the cervical, vaginal, or vulvar epithelium under magnification to identify abnormal areas that may require a biopsy. 2. Involves removal of one or more areas of the endometrium by means of a small device without widening the cervix. 3. Allows visual examination of the uterine cavity with a small scope passed through the cervix. 4. Allows visualization of the abdominal and pelvic cavities through a small scope passed through a small abdominal incision. Rationales Option 1: A colposcopy visualizes the cervical, vaginal, or vulvar epithelium under magnification to identify abnormal areas that may require a biopsy. It is performed in the office. Option 2: An endometrial biopsy removes one or more areas of the endometrium by means of a curette or small aspiration device without cervical dilation. Option 3: A hysteroscopy allows visual examination of the uterine cavity with a small fiberoptic endoscope passed through the cervix. Option 4: A laparoscopy allows visualization of the abdominal and pelvic cavities through a small fiberoptic endoscope passed through a subumbilical incision. [Page reference: 734] Question 10. Which of the following statements do you use when instructing women about their fertile period (ie, when they are most likely to become pregnant)? 1. Ovulation occurs on the 14th day, plus or minus 2 days, before the next menses. 2. Sperm are viable for 24 hours. 3. The ovum is viable for 6 hours. 4. The ovaries always release 1 ovum per month. Rationales Option 1: When instructing women about their fertile period (ie, when they are most likely to become pregnant), tell them that ovulation occurs on the 14th day, plus or minus 2 days, before the next menses. Sperm are viable for 3 days, and the ovum is viable for 24 hours. It is essential for women to know these facts if they are using the calendar or rhythm method for preventing a pregnancy. Option 2: Sperm are viable for 3 days. Option 3: The ovum is viable for 24 hours. Option 4: The ovaries may release more than 1 ovum per month, or none at all, depending on the fertility status of the woman. [Page reference: 678] Question 11. Mary, age 33, delivered a healthy baby boy 6 months ago by cesarean delivery. Her baby weighed 9 lb 6 oz. Her prenatal care was complicated by gestational diabetes mellitus (GDM). She did not need to take any medications. She asks you about the possibility of getting diabetes later in life. The correct response should be: 1. “Up to 50% of women diagnosed with GDM will eventually develop diabetes.” 2. “No, you won’t have a problem later in life, but you may have the same problem with another pregnancy.” 3. “Most women are fine, but we can check your sugar levels when you turn 50.” 4. “If you had to take insulin, then you would have to worry about getting diabetes later on.” Rationales Option 1: GDM affects 2% to 10% of pregnancies in the United States. Although the carbohydrate intolerance of GDM usually resolves quickly after delivery, approximately one-third of women who had GDM will have diabetes or impaired glucose metabolism at 4 to 6 weeks postpartum. Up to 50% will develop diabetes in the future. Option 2: Up to 50% of women who had GDM will develop diabetes in the future. Option 3: Women with GDM should have a fasting plasma glucose test or an oral glucose tolerance test postpartum. Women with normal values at the postpartum visit should be screened every 3 years. Option 4: Up to 50% of women who had GDM will develop diabetes in the future. [Page reference: 925] Question 12. Darcy, age 57, is to undergo a core needle biopsy for a suspicious breast mass. This procedure includes: 1. A 21- or 22-gauge needle that is used to aspirate cells from the lesion for analysis. 2. Removal of a large core of tissue from the lesion for histological evaluation utilizing a large-gauge cutting needle. 3. Removal of a wedge of tissue for examination. 4. Removal of the entire lesion. Rationales Option 1: A 21- or 22-gauge needle is used to aspirate a cyst or extract cells from a palpable solid lesion for analysis in a fine-needle aspiration (FNA). Option 2: In a core needle biopsy, a large-gauge cutting needle is used to obtain a large core of tissue from the lesion for histological examination. The results of a core needle biopsy are as accurate as a surgical biopsy, but the procedure is less invasive, with better cosmetic results. A stereotactic or ultrasound-guided biopsy can be performed on nonpalpable lesions. Option 3: An incisional biopsy in which a large wedge of tissue is removed for histological examination may be done when a mass is very large and cannot be removed without major surgery. Option 4: An open surgical excisional biopsy (lumpectomy) involves the complete removal of a palpable mass or nonpalpable lesion (after stereotactic or ultrasound-guided biopsy or mammographic needle localization). [Page reference: 697] Question 14. Judi, age 24, has a seizure disorder. She would like to conceive within the next year. Her visit is for preconceptual counseling. What is the drug of choice for her during pregnancy? 1. Valproic acid (Depakene). 2. Trimethadione (Tridione). 3. Phenobarbital (Luminal). 4. Phenytoin (Dilantin). Rationales Option 1: Valproic acid (Depakene) is contraindicated during pregnancy. Option 2: Trimethadione (Tridione) is contraindicated during pregnancy. Option 3: Phenobarbital (Luminal) is the drug of choice in a pregnant woman with a seizure disorder. Ideally, if the woman has not had a seizure for 5 years prior to becoming pregnant, a prepregnancy trial of withdrawal from seizure medication should be tried. If phenobarbital is used, serum levels should be measured in each trimester, and the dosage should be such that it maintains the serum levels in the low-normal therapeutic range. Option 4: Phenytoin (Dilantin) is teratogenic during the first trimester. [Page reference: 102] Question 15. Sarah, age 29, complains of premenstrual syndrome (PMS). She states she was told that changing her diet might help in managing some of the symptoms. What change in her diet do you recommend? 1. Decreased intake of protein. 2. Increased intake of complex carbohydrates. 3. Increased intake of salt and salty foods. 4. Decreased intake of fatty foods. Rationales Option 1: Iron-rich foods, such as lean meats, should not be reduced before or during Sarah’s period. Option 2: Advise the client complaining of PMS to increase her intake of complex carbohydrates. A diet high in complex carbohydrates, such as whole grains, cereals, fruits, and vegetables, helps prevent low blood sugar levels and reduces fatigue, jitteriness, and irritability. It may also raise serotonin levels, thus improving mood. Eating several small meals at frequent intervals rather than 3 large ones also keeps blood sugar on an even level and reduces the feeling of bloating. Option 3: Women should restrict their intake of salt, caffeine, and alcohol during the week before their period. Option 4: Decreasing intake of fatty foods has not been shown to affect PMS symptoms. [Page reference: 713-714] Question 16. A 33-year-old woman complains of dyspareunia in the lower back during orgasm. The nurse practitioner should consider: 1. Endometriosis. 2. Cystitis. 3. Vaginitis. 4. Pelvic inflammatory disease. Rationales Option 1: When a client complains of dyspareunia in the lower back during orgasm, you should consider a diagnosis of endometriosis. Causes of dyspareunia are diverse and depend on when and where the pain occurs. Option 2: Cystitis should be considered when pain occurs in the vaginal canal and adjacent structures with the penis midvagina. Option 3: Vaginitis should be considered when pain occurs in the vaginal canal and adjacent structures with the penis midvagina. Option 4: Pelvic inflammatory disease should be considered if pain occurs in the deep pelvis on deep penile penetration with thrusting. [Page reference: 730] Question 17. A 21-year-old woman comes to your office and reports a history of genital warts. In reference to the human papillomavirus (HPV) vaccination (ie, Gardasil, Gardasil 9, and Cervarix), she should be educated that: 1. She is not in the correct age group and is not a candidate for vaccination. 2. She should receive HPV vaccination. 3. She has already been exposed to HPV; therefore, she is not a candidate for vaccination. 4. There is a vaccine coming out shortly specifically for those who have been exposed. She should wait. Rationales Option 1: Females who are sexually active may benefit from vaccination, but they may get less benefit from it than those who are vaccinated before they become sexually active. Option 2: HPV is the most common sexually transmitted infection (STI). There are more than 40 HPV types that can infect the genital areas, mouth, and throat of males and females. According to the Centers for Disease Control and Prevention (CDC), females should get the vaccine before they become sexually active and prior to becoming exposed to HPV. Females who are sexually active may also benefit from vaccination, but they may get less benefit from it. While this question involves a young woman, the CDC recommends that HPV vaccination be given to both boys and girls at age 11 or 12 so they are protected before ever being exposed to the virus. Option 3: Women who have an existing history of genital warts have generally been exposed to HPV types 6 and 11, which are not oncogenic. Few sexually active young women have been infected with all HPV types prevented by the vaccines, so most young women could still get protection from vaccination. Option 4: She should receive HPV vaccination now. It is unlikely that she is infected with all HPV types, so she could still get protection from vaccination. [Page reference: 735] Question 6. Jennifer, age 27, is complaining of lower abdominal pain. After doing some laboratory studies, you find leukocytosis, an elevated erythrocyte sedimentation rate, and an elevated C-reactive protein level. Which of the following is the most appropriate diagnosis? 1. Ovarian cyst. 2. Pelvic inflammatory disease. 3. Tubal pregnancy. 4. Diverticulitis. Rationales Option 1: An ovarian cyst may not produce any changes in blood work. A serum cancer antigen 125 (CA-125) assay checks for a substance called CA-125, which is associated with ovarian cancer. Option 2: In clients with pelvic inflammatory disease, leukocytosis is present in about 50% of cases, the erythrocyte sedimentation rate is classically elevated, and the C-reactive protein level is elevated (exceeding 20 mg/L) in about 74% of cases. Option 3: No single serum ß-human chorionic gonadotropin (ß-hCG) level is diagnostic of an ectopic pregnancy. Serial serum ß-hCG levels are necessary to differentiate between normal and abnormal pregnancies. Option 4: In diverticulitis, a hemogram may reveal leukocytosis and a left shift, indicating infection. However, the absence of leukocytosis does not rule out diverticulitis, as 20% to 40% of patients have a normal white blood cell count. HEMATOLOGICAL DISORDERS Mindy, age 6, was recently discharged from the hospital after a sickle cell crisis. You are teaching her parents to be alert to the manifestations of splenic sequestration and tell them to be alert to: 1. Vomiting and diarrhea. 2. Decreased mental acuity. 3. Abdominal pain, pallor, and tachycardia. 4. Abdominal pain and vomiting. Rationales Option 1: Vomiting and diarrhea do not necessarily accompany this complication. Option 2: A decrease in mental acuity does not necessarily accompany this complication. Option 3: Abdominal pain, pallor, and tachycardia are all manifestations of splenic sequestration. Early recognition of splenic sequestration can be a lifesaving skill. Parents can be taught to recognize signs of increasing anemia and enlarging spleen. Part of the educational plan for parents is teaching them how to recognize increasing abdominal girth or abdominal pain, as well as how to palpate the spleen. Option 4: Abdominal pain is a manifestation of splenic sequestration, but vomiting does not necessarily accompany this complication. [Page reference: 978-979] Question 2. Your client Mrs. Young, age 64, is here to see you because she has pain in her left breast. She reports no pain in her right breast and no noted lesions or masses on breast self-exam, which she performs monthly. You know that: 1. At her age, you do not need to worry about breast cancer. 2. Initial presentation of breast pain is usually not suspicious for malignancy. 3. She must be sent for a mammogram as soon as possible. 4. She has no personal or family history of breast cancer; therefore, you are not concerned. Rationales Option 1: Age 64 puts this client at a high risk of breast cancer. Option 2: In postmenopausal women, pain in one breast is highly suggestive of a malignant process. Option 3: In postmenopausal women, pain in one breast is highly suggestive of a malignant process despite past personal and/or family history. Additionally, you know that age 64 puts this client at a high risk of breast cancer. You must send her for a mammogram as soon as possible even if she had a negative one less than a year before. Option 4: In postmenopausal women, pain in one breast is highly suggestive of a malignant process despite past personal and/or family history. [Page reference: 695-696] Question 3. Jimmy, age 6 months, is newly diagnosed with sickle cell disease. His mother brings him to the clinic for a well-baby visit. Which of the following should you do on this visit? 1. Tell his mother that Jimmy will not be immunized because of his diagnosis. 2. Tell his mother that Jimmy should not go to day care. 3. Immunize Jimmy with the diphtheria, tetanus, and pertussis (DTaP); Haemophilus influenzae type b (Hib); hepatitis B virus (HBV); and poliomyelitis vaccines. 4. Immunize Jimmy with the measles, mumps, and rubella; Haemophilus influenzae type b (Hib); and hepatitis B virus (HBV) vaccines only. Rationales Option 1: Jimmy will be immunized; his diagnosis has no effect on vaccinations. Option 2: Children with sickle cell disease should be treated like other children, and their activities should not be limited unless they are experiencing a painful sickle cell crisis. Option 3: At 6 months of age, Jimmy should be immunized with the DTaP, Hib, HBV, and poliomyelitis vaccines. Children with sickle cell disease should receive all the standard well-baby care, but in addition to the standard immunizations, they should receive the pneumococcal vaccine at age 2 years. There is no cure for sickle cell disease. Option 4: Jimmy should receive all the standard immunizations, and he should receive the pneumococcal vaccine at age 2 years. [Page reference: 978-979] Question 4. Allie, age 5, is being treated with radiation for cancer. Her mother asks about the effect radiation will have on Allie’s future growth. Although she knows that a specialist will be handling Allie’s care, her mother asks for your opinion. How do you respond? 1. “Let’s worry about the cancer first and then see how her growth is affected.” 2. “Chemotherapy may affect her future growth, but radiation will not.” 3. “She will probably have growth hormone problems, in which case she can then begin growth hormone therapy.” 4. “That’s the least of your worries now; everything will turn out okay.” Rationales Option 1: Negating the mother’s concern by saying she should “worry about the cancer first and then see what happens” is always a poor choice. Option 2: Chemotherapy may result in a decrease in linear growth, but the child usually catches up when the chemotherapy is discontinued. Option 3: Growth complications depend on the direct damage to the endocrine tissue. Children with acute lymphocytic leukemia, brain tumors, nasopharyngeal cancers, and orbital tumors who have received radiation therapy are at the highest risk. Approximately 50% to 90% of these children will have some evidence of growth hormone deficiency, and they may benefit from growth hormone therapy. Spinal radiation inhibits vertebral body growth. Option 4: Assuring the mother that “everything will be okay” is always a poor choice. [Page reference: 985-986] Your client Jeannie, age 64, comes to you complaining of tinnitus and lightheadedness without loss of consciousness. On physical exam, you note splenomegaly. To sort out your differential diagnosis, you order an alkaline phosphatase and vitamin B12 level because you are ruling out a diagnosis of: 1. Liver cancer. 2. Pancreatic cancer. 3. Mononucleosis. 4. Polycythemia vera. Rationales Option 1: Liver cancer does not cause splenomegaly. Option 2: Pancreatic cancer does not cause splenomegaly. Option 3: Mononucleosis does not cause tinnitus or lightheadedness. Option 4: Polycythemia vera causes a hyperviscosity of the blood, which leads to decreased cerebral blood flow, resulting in symptoms of tinnitus, lightheadedness, and occasionally stroke and thrombosis. Alkaline phosphatase and serum vitamin B12 levels are elevated; leukocytosis and thrombocytosis are also present. Red blood cell mass is elevated, and arterial oxygen saturation is more than 92% (ie, the erythrocytosis is not secondary to hypoxia). [Page reference: 981-982] Julie’s brother has chronic lymphocytic leukemia (CLL). She overheard that he was in stage IV and asks what this means. According to the Rai classification system, stage IV is a stage: 1. In which the lymphocyte count is greater than 10,000 mm3. 2. Indicating absolute lymphocytosis, in which the client may live 7 to 10 years or more. 3. Of thrombocytopenia in which the life expectancy may be only 2 years. 4. Of anemia. Rationales Option 1: Stage 0 indicates a lymphocyte count greater than 10,000 mm3. Option 2: Stage 0 indicates absolute lymphocytosis. Option 3: Stage IV in the Rai classification system of CLL is the stage of thrombocytopenia where the life expectancy may be only 2 years. CLL is the only leukemia in which a staging system is commonly used because CLL has prognostic implications. A client with only absolute lymphocytosis (Rai stage 0) may live 7 to 10 years or more. In the Rai classification system, stage 0 indicates a lymphocyte count greater than 10,000 mm3, stage I indicates enlarged lymph nodes, stage II indicates an enlarged liver and/or spleen, stage III indicates anemia, and stage IV indicates thrombocytopenia. Option 4: Rai stage III indicates anemia. [Page reference: 991] Question 7. You have a new client, Robert, age 67, who presents with generalized lymphadenopathy. You know that this is indicative of: 1. Disseminated malignancy, particularly of the hematologic system. 2. Cancer of the liver. 3. Sjögren syndrome. 4. Pancreatic cancer. Rationales Option 1: Generalized lymphadenopathy is usually indicative of disseminated malignancy, typically hematologic in nature (such as lymphoma or leukemia); collagen vascular disease; or an infectious process such as mononucleosis, syphilis, cytomegalovirus, tuberculosis, AIDS, toxoplasmosis, and so on. Option 2: Cancer of the liver does not present with generalized lymphadenopathy. Option 3: Sjögren syndrome does not present with generalized lymphadenopathy. Option 4: Pancreatic cancer does not present with generalized lymphadenopathy. [Page reference: 957] Question 8. Your client Mr. Jones has Sjögren syndrome. Which treatment do you suggest? 1. Artificial tears and chewing sugarless gum. 2. Frequent rinsing of the mouth with mouthwash. 3. Drinking at least 1 glass of milk per day. 4. Removing wax from the ears at regular intervals. Rationales Option 1: Sjögren syndrome is a multisystem autoimmune disease characterized by dysfunction of the exocrine glands; it is specifically notable for dry eyes and dry mouth. Treatment is aimed at increasing comfort and lubrication. Artificial tears can be self-administered as needed; preservative-free products are usually better tolerated. For dry mouth, increasing hydration and chewing sugarless gum may be helpful. Oral pilocarpine (Salagen) 5 mg 4 times a day and cevimeline (Evoxac) 30 mg 3 times a day have been shown to increase saliva production. Option 2: Rinsing frequently with mouthwash that contains alcohol may prove more drying. Option 3: Drinking milk is not recommended. Option 4: Frequent removal of earwax is irrelevant to this problem. [Page reference: 1015] Stu, age 49, has slightly reduced hemoglobin and hematocrit readings. What is your next action after you ask him about his diet? 1. Repeat the laboratory tests. 2. Perform a fecal occult blood test. 3. Start him on an iron preparation. 4. Start him on folic acid. Rationales Option 1: There is no need to repeat the laboratory tests. Option 2: Tests for fecal occult blood should be done on all clients suspected of having iron deficiency anemia. In the early stages of iron deficiency anemia, both hemoglobin and hematocrit levels are normal to slightly reduced. It is necessary to determine whether the iron deficiency is related solely to inadequate dietary intake or to decreased absorption or chronic blood loss. Option 3: Before beginning treatment, it is necessary to determine the cause of the iron deficiency. Option 4: Before beginning treatment, it is necessary to determine the cause of the iron deficiency. [Page reference: 959-960] Question 10. Which hypersensitivity reaction results in a skin test that produces erythema and edema within 3 to 8 hours? 1. Anaphylactic reaction. 2. Cytotoxic reaction. 3. Immune complex–mediated reaction. 4. Delayed hypersensitivity reaction. Rationales Option 1: An anaphylactic reaction may occur with allergic rhinitis or asthma. The mediator of injury is histamine, and the skin test appears as a wheal and flare. Option 2: A cytotoxic reaction, such as a transfusion reaction, does not produce any reaction in a skin test. Option 3: An immune complex–mediated reaction results in a skin test that produces erythema and edema within 3 to 8 hours. It may result from serum sickness, systemic lupus erythematosus, or rheumatoid arthritis. Option 4: A delayed hypersensitivity (cell-mediated) reaction, such as contact dermatitis or that seen after a test for tuberculosis, produces erythema and edema within 24 to 48 hours. [Page reference: 997] Question 11. In which of the following circumstances is the reticulocyte count elevated? 1. Aplastic anemia. 2. Iron deficiency anemia. 3. Poisoning. 4. Acute blood loss. Rationales Option 1: The reticulocyte count is decreased in aplastic anemia because the bone marrow has shut down all production of cells. Option 2: The reticulocyte count is decreased in disorders of red blood cell (RBC) maturation such as iron deficiency anemias. Option 3: The reticulocyte count is decreased in poisonings. Option 4: The reticulocyte count indicates the percentage of newly maturing red blood cells (RBCs) released into the circulating blood from the bone marrow. As an RBC matures, it loses its endothelial reticulum. The reticulocyte count is elevated in cases of blood loss as the body tries to replace the loss; it might also be elevated during treatment of anemias (eg, iron, folic acid, and vitamin B12) and bone marrow disorders, when immature RBCs are displaced by other proliferating cells. [Page reference: 967] Question 12. Macrocytic normochromic anemias are caused by: 1. Acute blood loss. 2. An infection or tumor. 3. A nutritional deficiency of iron. 4. A deficiency of folic acid. Rationales Option 1: Acute blood loss and most hemolytic processes cause normocytic normochromic anemias in which the cell size is normal. Option 2: Infections or tumors may cause an anemia of chronic disease that produces a normocytic red blood cell. Option 3: Iron deficiency anemias are microcytic hypochromic and may result from dietary insufficiencies as well as acute blood loss. Option 4: A folic acid and/or vitamin B12 deficiency causes macrocytic normochromic anemias in which the cell size is large and irregular. [Page reference: 971] Question 13. Which of the following cancers is associated with Epstein-Barr virus? 1. Burkitt lymphoma. 2. Kaposi sarcoma. 3. Lymphoma. 4. Adult T-cell leukemia. Rationales Option 1: Burkitt lymphoma is associated with Epstein-Barr virus. Option 2: Kaposi sarcoma is associated with human cytomegalovirus. Option 3: Lymphoma is associated with human herpesvirus 6. Option 4: Adult T-cell leukemia is associated with human T-lymphotropic viruses. [Page reference: 1023-1025] Lorie, age 29, appears with the following signs: pale conjunctivae and nail beds, tachycardia, heart murmur, cheilosis, stomatitis, splenomegaly, koilonychia, and glossitis. What do you suspect? 1. Vitamin B12 deficiency. 2. Folate deficiency. 3. Iron deficiency anemia. 4. Chronic fatigue syndrome. Rationales Option 1: Signs of vitamin B12 deficiency include weakness of the extremities, ataxia, pallor, loss of vibratory and position sense, memory loss, changes in mood, and hallucinations. Option 2: Signs of a folate deficiency include weakness, pallor, and glossitis, with congestive heart failure occurring if the anemia is severe. Option 3: Lorie has the classic signs of iron deficiency anemia: pale conjunctivae and nail beds, tachycardia, heart murmur, cheilosis (reddened lips with fissures at the angles), stomatitis, splenomegaly, koilonychia (thin and concave fingernails with raised edges), and glossitis. Signs may also include esophageal webs (Plummer-Vinson syndrome), melena, and menorrhagia. Option 4: A person with chronic fatigue syndrome might have a fever, a sore throat, myalgia, and generalized headaches. [Page reference: 961-963] Question 15. Your client Jackson has decreased lymphocytes. You suspect: 1. Bacterial infection. 2. Viral infection. 3. Immunodeficiency. 4. Parasitic infection. Rationales Option 1: Lymphocytes are increased occasionally in bacterial infections (eg, pertussis and brucellosis). Option 2: Lymphocytes are increased primarily in viral infections (eg, hepatitis, infectious mononucleosis, cytomegalovirus [CMV], and herpes zoster). Option 3: A decrease in lymphocytes would be most consistent with immunodeficiency disorders, long-term corticosteroid therapy, or debilitating diseases such as Hodgkin lymphoma and systemic lupus erythematosus. Option 4: Eosinophils are elevated in parasitic infections such as malaria, trichinosis, and ascariasis. [Page reference: 1017, 1034-1035] Your client Shirley has an elevated mean corpuscular volume (MCV). What should you be considering in terms of diagnosis? 1. Iron deficiency anemias. 2. Hemolytic anemias. 3. Lead poisoning. 4. Liver disease. Rationales Option 1: MCV is decreased (microcytic) in iron deficiency anemias. Option 2: MCV is decreased (microcytic) in hemolytic anemias. Option 3: MCV is decreased (microcytic) in defects involving porphyrin synthesis, such as those caused by lead poisoning. Option 4: MCV indicates the average size of individual red blood cells (RBCs). The normal (normocytic) range is 76 to 96 fL. MCV is increased (macrocytic) in megaloblastic anemias (ie, vitamin B12 deficiency and folate deficiency) and liver disease (such as that resulting from alcohol abuse) as well as with the use of some drugs (eg, zidovudine). [Page reference: 971] Samuel, age 5, is receiving radiation therapy for his acute lymphoblastic leukemia. This treatment puts him at increased risk of developing which type of cancer as a secondary malignancy when he becomes an adult? 1. Chronic lymphocytic leukemia. 2. Brain tumor. 3. Liver cancer. 4. Esophageal cancer. Rationales Option 1: Chronic lymphocytic leukemia is not a result of radiation for childhood leukemia. Option 2: Children receiving radiation therapy for acute lymphoblastic leukemia are at an increased risk of developing a brain tumor as a secondary malignancy. This has been seen more often in children who were treated with radiation at age 5 or younger. In general, about 3% to 12% of children treated for cancer will develop a new cancer within 20 years of being treated for the primary cancer. Option 3: Liver cancer is not a result of radiation for childhood leukemia. Option 4: Esophageal cancer is not a result of radiation for childhood leukemia. [Page reference: 985] Question 18. Sam is being worked up for pancreatic cancer. He states the doctor wants to put a “scope” in and inject dye into his ducts. He wants to know more about this. What procedure is he referring to? 1. Percutaneous transhepatic cholangiography (PTC). 2. Endoscopic retrograde cholangiopancreatography (ERCP). 3. Angiography. 4. Upper gastrointestinal (GI) series. Rationales Option 1: A PTC is a procedure in which a thin needle is put into the liver through the skin on the right side of the abdomen and dye is injected into the bile ducts to visualize any blockages. Option 2: An ERCP uses an endoscope, which is inserted via the mouth and passed through the stomach and into the small intestine, where dye is injected into the pancreatic ducts and x-rays are taken to determine if any obstruction is apparent. All of the procedures listed in this question are used to produce pictures of the pancreas and nearby organs to assist in the diagnosis of pancreatic cancer. Option 3: An angiography is a procedure in which x-rays are taken of blood vessels after dye is injected. Option 4: An upper GI series is a series of x-rays of the upper digestive system taken after barium is ingested. It shows the outline of the digestive organs. [Page reference: 611] Which of the following is the most common cause of megaloblastic anemia? 1. Vitamin B12 deficiency. 2. Pernicious anemia. 3. Iatrogenesis. 4. Folate deficiency. Rationales Option 1: Vitamin B12 deficiency is the most common cause of megaloblastic anemia. However, vitamin B12 deficiency is most commonly caused by the autoimmune disease pernicious anemia. Option 2: Pernicious anemia is the most common cause of megaloblastic anemia, as it causes vitamin B12 deficiency. Option 3: Medications like methotrexate can cause megaloblastic anemia; however, this is not the most common cause. Option 4: Folate deficiency can cause megaloblastic anemia; however, it is not the most common cause. [Page reference: 971] Question 20. Which of the following is not considered an AIDS-defining condition? 1. A CD4 count of 150. 2. A human immunodeficiency virus (HIV) positive patient with candidiasis of the throat. 3. A human immunodeficiency virus (HIV) positive patient with Kaposi sarcoma. 4. A human immunodeficiency virus (HIV) positive patient with 1 episode of bacterial pneumonia and a CD4 count greater than 500. Rationales Option 1: A CD4 count of less than 200 is considered an AIDS-defining condition. Option 2: Candidiasis of the throat and lungs is considered an AIDS-defining condition. Option 3: Kaposi sarcoma is considered an AIDS-defining condition. Option 4: For bacterial pneumonia to be considered an AIDS-defining condition in someone with a CD4 count greater than 500, the patient would have to have 2 episodes in 1 year. [Page reference: 1030] Julia asks how smoking increases the risk of folic acid deficiency. You respond that smoking: 1. Causes small vessel disease and constricts all vessels that transport essential nutrients. 2. Decreases vitamin C absorption. 3. Affects the liver’s ability to store folic acid. 4. Causes nausea, thereby inhibiting the appetite and ingestion of foods rich in folic acid. Rationales Option 1: While smoking does cause small vessel disease, it does not cause folic acid deficiency. Option 2: Smoking decreases vitamin C absorption, which is necessary for folic acid absorption. Smoking increases vitamin requirements. Clients with a folic acid deficiency should be encouraged to eat foods that are high in folic acid (eg, asparagus spears, beef liver, broccoli, mushrooms, oatmeal, peanut butter, and red beans) daily because the liver can store folic acid for only a limited time. Option 3: The liver can store folic acid for only a limited time. This is not affected by smoking. Option 4: Smoking does not typically cause nausea in smokers. Question 3. Shelley has esophageal cancer and asks you if cigarettes and alcohol played a part in its development. How do you respond? 1. “Your cancer was caused by your cigarette smoking, nothing else.” 2. “Alcohol is also a carcinogen.” 3. “Alcohol directly alters DNA and causes mutations.” 4. “Alcohol modifies the metabolism of carcinogens in the esophagus and increases their effectiveness.” Rationales Option 1: Because Shelley smoked cigarettes and drank alcohol, she has an increased risk of oral, esophageal, and laryngeal cancers. Option 2: Ethanol in alcoholic drinks breaks down into acetaldehyde, a probable human carcinogen. Option 3: Alcohol can produce molecules that damage DNA. Option 4: Alcohol acts as a promoter by modifying the metabolism of carcinogens in the esophagus and liver, thereby increasing their effectiveness in some tissues. Question 4. Sally has human immunodeficiency virus (HIV) and asks which method of birth control, other than abstinence, would be best for her. You suggest: 1. Latex condoms. 2. The spermicide nonoxynol-9. 3. An intrauterine device (IUD). 4. An oral contraceptive. Rationales Option 1: Latex condoms, when used consistently and correctly, are the preferred contraceptive method for a client infected with HIV because they provide the most effective barrier between partners. Sally should continue to use condoms even when she is postmenopausal and birth control is no longer an issue in order to prevent spreading HIV to her partner. Option 2: The spermicide nonoxynol-9 has not been shown to prevent viral transmission in humans, especially when used alone. Option 3: Because IUDs increase menstrual blood flow, they expose a woman’s partner to a greater viral load. Option 4: Although the effect of oral contraceptive pills on HIV transmission is not known, estrogen and progestin can promote HIV progression through opportunistic infections; they can also promote cervical neoplasia via their immunomodulating effects. Question 6. Which of the following white blood cell types are elevated in parasitic infections, hypersensitivity reactions, and autoimmune disorders? 1. Neutrophils. 2. Eosinophils. 3. Basophils. 4. Monocytes. Rationales Option 1: Neutrophils are increased in acute infections, the stress response, myelocytic leukemia, and inflammatory or metabolic disorders. Option 2: Eosinophils are elevated in parasitic infections, hypersensitivity reactions, and autoimmune disorders. Option 3: Basophils are increased after a splenectomy and in hypersensitivity responses, chronic myelogenous leukemia, chickenpox, smallpox, and hypothyroidism. Option 4: Monocytes are increased in chronic inflammatory disorders, tuberculosis, viral infections, leukemia, Hodgkin disease, and multiple myeloma. Question 7. Jill has just been given a diagnosis of human immunodeficiency virus (HIV) and has a normal initial Pap smear. When do the Centers for Disease Control and Prevention (CDC) guidelines state she should have a repeat Pap test? 1. In 3 months. 2. In 6 months. 3. In 1 year. 4. She should have a colposcopy every year instead of a Pap test. Rationales Option 1: If a second evaluation is performed in 6 months and severe inflammation with reactive squamous cellular change is found, another Pap smear should be done within 3 months. Option 2: The CDC guidelines state that if a woman infected with HIV has a normal initial Pap test, then a second evaluation should be done in 6 months to reduce the likelihood of a false-negative initial test. If the initial 2 Pap smears are both negative, annual Pap smears are then adequate. If severe inflammation with reactive squamous cellular change is found, another Pap smear should be done within 3 months. Option 3: If a second evaluation is performed in 6 months and the result is negative, annual Pap smears are adequate. Option 4: A colposcopy is recommended if a Pap smear result is positive. Question 8. You are examining Joseph, age 9 months, and note a palpable right supraclavicular lymph node. You know that this finding is suspicious for: 1. Candidiasis. 2. Cryptococcosis. 3. Lymphoma of the mediastinum. 4. Abdominal malignancy. Rationales Option 1: Different lymph nodes will be palpable in the presence of infectious processes such as candidiasis. Option 2: Different lymph nodes will be palpable in the presence of infectious processes such as cryptococcosis. Option 3: Palpable supraclavicular lymph nodes are not normal in infants, children, or adults. A palpable right-sided node is more commonly associated with lymphoma of the mediastinum. Option 4: A palpable left-sided node is more commonly associated with an abdominal malignancy. Question 9. Marsha states that a relative is having a carcinoembryonic antigen (CEA) test done to detect some type of cancer. She wants to know what kind. You tell her a CEA is performed to detect: 1. Adenocarcinoma of the prostate. 2. Medullary cancer of the thyroid. 3. Adenocarcinomas of the colon, lung, breast, ovary, stomach, and pancreas. 4. Multiple myeloma. Rationales Option 1: Prostate-specific antigen (PSA) is the tumor marker for adenocarcinoma of the prostate. Option 2: Calcitonin is the tumor marker for medullary cancer of the thyroid. Option 3: CEA is a tumor marker for adenocarcinomas of the colon, lung, breast, ovary, stomach, and pancreas. Option 4: An immunoglobulin test will detect multiple myeloma. Tina, age 2, had a complete blood count (CBC) drawn at her last visit. It indicates that she has a microcytic hypochromic anemia. What should you do now at this visit? 1. Obtain a lead level. 2. Instruct Tina’s parents to increase the amount of milk in her diet. 3. Start Tina on ferrous sulfate (Feosol) and check the CBC in 6 weeks. 4. Recheck the CBC. Rationales Option 1: The provider should always check a lead level before starting iron supplementation in children because an elevated lead level will cause anemia despite a normal iron level. Option 2: Regular milk (cow’s milk) is often the cause of anemia in children; thus, a thorough diet history must be obtained. Children younger than age 1 year are usually on iron-fortified infant formulas, and when they switch to cow’s milk, they do not receive sufficient iron. Option 3: Before starting Tina on iron supplementation, it is important to determine why she has anemia. Supplementation can cause iron overload. Option 4: Rechecking the CBC will not add new information, as the goal now is to determine why the patient has anemia. Jan is having biological therapy for her pancreatic cancer. What kind of treatment is this? 1. Surgery. 2. Radiation therapy. 3. Immunotherapy. 4. Chemotherapy. Rationales Option 1: Two types of surgery—curative and palliative—are available for pancreatic cancer; however, these are not biological therapies. Option 2: Radiation therapy uses high-energy rays to damage cancer cells and prevent them from growing and dividing. Option 3: Immunotherapy is also called biological therapy. It is a form of treatment that uses the body’s natural ability (ie, the immune system) to fight disease or to protect itself from the adverse effects of treatment. Option 4: Chemotherapy uses drugs to kill cancer cells. Which of the following conditions is an X-linked recessive disorder commonly seen in African American males? 1. Sickle cell anemia. 2. Glucose-6-phosphate dehydrogenase deficiency. 3. Pyruvate kinase deficiency. 4. Bernard-Soulier syndrome. Rationales Option 1: Sickle cell anemia is an autosomal recessive disorder in which an abnormal hemoglobin leads to chronic hemolytic anemia, resulting in a variety of severe clinical consequences. Option 2: Glucose-6-phosphate dehydrogenase (G6PD) deficiency is an X-linked recessive disorder commonly seen in African American males. It is an enzyme defect that causes episodic hemolytic anemia because of the decreased ability of red blood cells to deal with oxidative stress. The other disorders listed are not X-linked. Option 3: Pyruvate kinase deficiency is a rare autosomal recessive disorder that causes chronic hemolytic anemia, usually beginning in childhood. Option 4: Bernard-Soulier syndrome is a rare autosomal recessive intrinsic platelet disorder that causes bleeding. Question 14. Nancy recently had a mastectomy and refuses to look at the site. Her husband does all the dressing changes. When she comes in to the office for a postoperative checkup, what would you say to her? 1. “You’ll look at it when you’re ready.” 2. “You must look at it today.” 3. “Everything’s going to be okay. It looks fine.” 4. “You have to accept this eventually; just glance at it today.” Rationales Option 1: Telling a client she will look at it when she is ready may mean she will never look at it. Option 2: She should not be forced, but you should take a matter-of-fact approach with an empathetic attitude that will assist in her eventual acceptance of the change in body image. Option 3: Practitioners should always avoid saying that everything is going to be okay because sometimes it is not. Option 4: With the loss of a body part, there is an initial stage of shock and denial. It is a protective mechanism and should be neither challenged nor promoted. The best response to Nancy, who refuses to look at her mastectomy scar, would be to say, “You have to accept this eventually; just glance at it today.” Then every day she could spend a little more time looking at it until she is able to care for the wound herself. Question 16. Which of the following indicate that Jim, a 32-year-old client with AIDS, has oropharyngeal candidiasis? 1. Small vesicles. 2. White, fissured, thickened patches. 3. Removable white plaques. 4. Flat-topped papules with thin, bluish-white spiderweb lines. Rationales Option 1: Herpes simplex is an acute viral disease that causes small vesicles on the lip borders (ie, cold sores). Option 2: Leukoplakia is a disease of the mucous membranes of the cheeks, gums, and tongue that involves white, thickened, fissured patches that may become malignant. Option 3: Oral candidiasis (thrush) appears as white plaques that can be scraped off, revealing an erythematous mucosal surface. Because of this, it is often referred to as a pseudomembranous lesion. Option 4: Flat-topped papules with thin, bluish-white spiderweb lines are lesions of lichen planus, a benign, inflammatory, pruritic disease of the skin and mucous membranes. Question 17. Sara comes today with numerous petechiae on her arms. You know she is not taking warfarin (Coumadin). What other drugs do you ask her about? 1. Aspirin or aspirin compounds. 2. Antihypertensive agents. 3. Oral contraceptives. 4. Anticonvulsants. Rationales Option 1: If your client has numerous petechiae, ask about the use of aspirin or aspirin compounds. Aspirin is a very effective antiplatelet agent. The recommended dosage for clients with chronic stable angina and certain other conditions is 325 mg (1 tablet) per day. Many clients assume if 1 tablet a day is good, 2 tablets a day is better. Depending on the individual clotting time, even 325 mg per day may result in bleeding into the tissues. Option 2: Antihypertensive agents by themselves do not affect platelet activity. Option 3: Oral contraceptives by themselves do not affect platelet activity. Option 4: Anticonvulsants by themselves do not affect platelet activity. Which of the following is the most common type of leukemia in the United States? 1. Acute lymphocytic leukemia. 2. Chronic myelogenous leukemia. 3. Acute myelogenous leukemia. 4. Chronic lymphocytic leukemia (CLL). Rationales Option 1: Chronic lymphocytic leukemia (CLL) is the most common type of leukemia in the United States. Option 2: Chronic lymphocytic leukemia (CLL) is the most common type of leukemia in the United States. Option 3: Chronic lymphocytic leukemia (CLL) is the most common type of leukemia in the United States. Option 4: CLL is the most common type of leukemia in the United States. Question 19. Which of the following tests is the most specific for diagnosing rheumatoid arthritis? 1. Anticyclic citrullinated peptide (anti-CCP). 2. Rheumatoid factor. 3. C-reactive protein. 4. Anti-Smith antibodies. Rationales Option 1: Anti-CCP is the most specific test for diagnosing rheumatoid arthritis. Option 2: Rheumatoid factor is often positive in rheumatoid arthritis but not as often as anticyclic citrullinated peptide (anti-CCP). Option 3: C-reactive protein is elevated with most inflammatory disorders and is nonspecific. Option 4: Anti-Smith antibodies are positive in lupus. Question 1. Mrs. Jameson complains of unilateral blurry vision and partial blindness in the left eye. On physical examination, you find decreased peripheral vision on her left side. Funduscopic examination reveals cotton wool spots. Your most likely diagnosis is: 1. Cryptococcosis. 2. Toxoplasmosis. 3. Cytomegalovirus infection. 4. Herpes simplex virus infection. Rationales Option 1: Cryptococcosis is a systemic fungal infection that may involve any organ of the body, including the lungs or skin, but has a marked predilection for the brain and its meninges. In the cerebral type, headache, dizziness, vertigo, and stiffness of the neck muscles are present. Option 2: Toxoplasmosis produces symptoms that may be so mild as to be barely noticeable or may be more severe and include lymphadenopathy, malaise, muscle pain, and little, if any, fever. It may also result in brain deterioration. Option 3: The classic signs and symptoms of cytomegalovirus infection on funduscopic examination include cotton wool spots, a “cottage cheese and ketchup” appearance, hemorrhage, and exudates. Decreased peripheral vision, blurriness, and partial blindness are other clinical manifestations. Referral to an ophthalmologist is imperative. Option 4: Herpes simplex virus infection is characterized by thin-walled vesicles that tend to recur in the same area, usually at a site where the mucous membranes join the skin; however, they may be limited to the gingiva, oropharynx, or conjunctivae. Your client George, age 60, presents with pruritus and complains of lymphadenopathy in his neck. He also complains of night sweats and has noticed a low-grade fever. He has not lost any weight and otherwise feels well. He is widowed and has been dating a new woman recently. On physical exam, you find enlarged supraclavicular nodes. You suspect: 1. Lung cancer. 2. Hodgkin lymphoma. 3. A lingering viral infection from a bout of flu he had 6 weeks ago. 4. Non-Hodgkin lymphoma. Rationales Option 1: Although supraclavicular nodes may be present in lung cancer, one does not typically see these other symptoms. Option 2: This presentation is classic for Hodgkin lymphoma. Although often seen in younger adults, this disease has a bimodal incidence that peaks at ages 15 to 35 and again after age 50. Pruritus is not uncommon, and up to 40% of clients have low-grade fever and recurrent night sweats. Option 3: These symptoms are not typically seen in a lingering viral infection after the flu. Option 4: These symptoms are not as commonly seen in cases of non-Hodgkin lymphoma. Question 3. Robin has human immunodeficiency virus (HIV) and is having a problem with massive diarrhea. You suspect the cause is: 1. Cryptococcosis. 2. Toxoplasmosis. 3. Cryptosporidiosis. 4. Cytomegalovirus. Rationales Option 1: Cryptococcosis is a fungal infection that usually appears as meningitis. Option 2: Toxoplasmosis is a protozoal infection that causes encephalitis in people with AIDS. Option 3: When clients with HIV infection have massive diarrhea, a protozoan of the Cryptosporidium genus is the most likely cause. The organism affects primarily the small intestine and produces massive diarrhea accompanied by nausea and fatigue. The diarrhea may exceed 4 L per day and can easily lead to dehydration and electrolyte imbalance if not treated promptly. Option 4: Cytomegalovirus is a significant opportunistic infection of the herpesvirus family that can be acquired during the perinatal period, the preschool years, or the sexually active years. Ms. Jones, your client, has an elevated platelet count. You suspect: 1. Systemic lupus erythematosus (SLE). 2. Infectious mononucleosis. 3. Disseminated intravascular coagulation (DIC). 4. Splenectomy. Rationales Option 1: Destruction of platelets is seen in SLE. Option 2: Destruction of platelets is seen in infectious mononucleosis. Option 3: Platelets are decreased in coagulation disorders such as DIC, septicemia, and eclampsia. Option 4: Increased platelet count is seen status post splenectomy and in myeloproliferative leukemias and polycythemia vera. Platelets are decreased in coagulation disorders such as disseminated intravascular coagulation (DIC), septicemia, and eclampsia. Increased destruction of platelets is seen in idiopathic thrombocytopenic purpura, systemic lupus erythematosus (SLE), and infectious mononucleosis. Decreased production of platelets is seen secondary to radiation and chemotherapy and in aplastic anemia and most leukemias Question 8. When the donor and recipient of a transplant are identical twins, this is referred to as what type of transplant? 1. Isograft. 2. Autograft. 3. Allograft. 4. Xenograft. Rationales Option 1: An isograft is a transplant in which the donor and recipient are identical twins. Option 2: An autograft is a transplant of the client’s own tissue; it is the most successful type of tissue transplant. Option 3: An allograft is a graft between members of the same species who have different genotypes and, in the case of humans, human leukocyte antigens. Option 4: A xenograft is a transplant from an animal species to a human, such as the pigskin used as a temporary covering after a massive burn. Question 9. Despite successful primary prophylaxis, which infection remains a common AIDS-defining diagnosis? 1. Pneumocystis jiroveci pneumonia 2. Cryptococcosis. 3. Cryptosporidiosis. 4. Candidiasis. Rationales Option 1: Before the appearance of AIDS, Pneumocystis jiroveci pneumonia was a rare disease that immunosuppressed people and those with leukemia sometimes developed. Today, Pneumocystis jiroveci pneumonia is usually the defining characteristic in people with AIDS in both the United States and Europe. Option 2: Cryptococcosis is a life-threatening systemic fungal infection that usually targets the central nervous system and the lungs, although it may attack anywhere. Option 3: Cryptosporidiosis is a protozoal infection responsible for diarrhea in people with AIDS. Option 4: Candidiasis is the most common fungal infection in AIDS, affecting 90% of all clients, although it is common in the general population as well. Question 10. Which of the following is a benign neoplasm? 1. Leiomyoma. 2. Osteosarcoma. 3. Glioma. 4. Seminoma. Rationales Option 1: A leiomyoma is a benign neoplasm of the smooth muscle. Option 2: An osteosarcoma is a malignant neoplasm of the bone tissue. Option 3: A glioma is a malignant neoplasm of the neuroglial cells. Option 4: A seminoma is a malignant tumor of the germ cells of the ectoderm and endoderm. Question 11. Mandy’s 16-year-old daughter has hepatitis A. Which of the following statements made by Mandy indicates she understands the teaching you’ve just completed? 1. “I guess she needs to be hospitalized until she’s recovered.” 2. “We’ll keep her at home with strict isolation precautions.” 3. “We’ll stop at the store and buy plastic eating utensils.” 4. “We’ll stop at the drugstore and pick up prescription medications immediately.” Rationales Option 1: Most clients can be cared for at home without undue risk. Option 2: Strict isolation is not necessary. Option 3: Clients with hepatitis A should have their eating and drinking utensils kept separate, or they should use disposable ones. Option 4: There is no specific medicine to treat hepatitis A. Question 15. Barbara, age 27, had her spleen removed after an automobile accident. You are seeing her in the office for the first time since her discharge from the hospital. She asks you how her surgery will affect her in the future. How do you respond? 1. “Your red blood cell production will be slowed.” 2. “Your lymphatic system may have difficulty transporting lymph fluid to the blood vessels.” 3. “You’ll have difficulty storing the nutritional agents needed to make red blood cells.” 4. “You may have difficulty salvaging iron from old red blood cells for reuse.” Rationales Option 1: The spleen is not part of red blood cell production. Option 2: The lymph system works without the spleen. Option 3: The spleen does not store the components needed to make red blood cells. Option 4: The spleen is not essential for life. When it is removed, the liver and bone marrow assume its functions. Although the bone marrow will produce and store hematopoietic stem cells, from which all cellular components of the blood are derived, it will not remove iron from old red blood cells for reuse. Question 16. Skip, age 4, is brought to the office by his mother. His symptoms are pallor, fatigue, bleeding, fever, bone pain, adenopathy, arthralgias, and hepatosplenomegaly. You refer him to a specialist. Which of the following tests do you expect the specialist to perform to confirm a diagnosis? 1. An enzyme-linked immunosorbent assay. 2. A monospot test. 3. A prothrombin time, partial thromboplastin time, bleeding time, complete blood count, and peripheral smear. 4. A bone marrow smear. Rationales Option 1: The enzyme-linked immunosorbent assay is the best test for rotavirus infection because it detects viral antigens. Option 2: The monospot test is a latex agglutination test that measures production of heterophile antibodies during acute and recent episodes of Epstein-Barr virus infection. Its use is limited because of false-negative readings of 10% to 20%. Option 3: A prothrombin time, partial thromboplastin time, bleeding time, complete blood count, and peripheral smear are included in an initial workup of bleeding disorders. Option 4: Skip has the characteristic symptoms of acute lymphoblastic leukemia. Diagnosis is made by the characteristic appearance found on a bone marrow smear. Lorie, age 29, appears with the following signs: pale conjunctivae and nail beds, tachycardia, heart murmur, cheilosis, stomatitis, splenomegaly, koilonychia, and glossitis. What do you suspect? 1. Vitamin B12 deficiency. 2. Folate deficiency. 3. Iron deficiency anemia. 4. Chronic fatigue syndrome. Rationales Option 1: Signs of vitamin B12 deficiency include weakness of the extremities, ataxia, pallor, loss of vibratory and position sense, memory loss, changes in mood, and hallucinations. Option 2: Signs of a folate deficiency include weakness, pallor, and glossitis, with congestive heart failure occurring if the anemia is severe. Option 3: Lorie has the classic signs of iron deficiency anemia: pale conjunctivae and nail beds, tachycardia, heart murmur, cheilosis (reddened lips with fissures at the angles), stomatitis, splenomegaly, koilonychia (thin and concave fingernails with raised edges), and glossitis. Signs may also include esophageal webs (Plummer-Vinson syndrome), melena, and menorrhagia. Option 4: A person with chronic fatigue syndrome might have a fever, a sore throat, myalgia, and generalized headaches. Which is the best serum test for spotting an iron deficiency early, before it progresses to full-blown anemia? 1. Hemoglobin. 2. Hematocrit. 3. Ferritin. 4. Reticulocytes. Rationales Option 1: Hemoglobin is the iron-containing pigment of red blood cells that carries oxygen from the lungs to the tissues. Hemoglobin values reflect the level at a given time only, without regard for the body’s stores. Option 2: Hematocrit is the volume of erythrocytes packed in a given volume of blood. Hematocrit values reflect the level at a given time only, without regard for the body’s stores. Option 3: A serum measurement of ferritin, the body’s iron-storing protein, can tell exactly how much iron is on hand in the body. It is the best way to spot an iron deficiency early, before it progresses to full-blown anemia. If the ferritin level is borderline, a dietary and supplemental regimen of iron will rebuild the iron stores. Option 4: Reticulocytes are the last immature stage of red blood cells. Question 19. Which of the following describes the pattern of inheritance that results in sickle cell anemia? 1. Autosomal recessive. 2. Autosomal dominant. 3. X-linked dominant. 4. X-linked recessive. Rationales Option 1: Sickle cell anemia is an autosomal recessive disorder, which means both parents must have the gene to pass it on to the child. Option 2: An example of an autosomal dominant disorder is Huntington disease. Option 3: An example of an X-linked dominant disorder is fragile X syndrome. Option 4: An example of an X-linked recessive disorder is hemophilia. Sherri’s blood work returns with a decreased mean corpuscular volume (MCV) and a decreased mean corpuscular hemoglobin concentration (MCHC). What should you do next? 1. Order a serum iron level and a total iron-binding capacity (TIBC) level. 2. Order a serum ferritin level. 3. Order a serum folate level. 4. Order a serum iron level, a total iron-binding capacity (TIBC) level, and a serum ferritin level. Rationales Option 1: A serum iron level, a TIBC level, and a serum ferritin level are needed to make a more final diagnosis. Option 2: A serum iron level, a total iron-binding capacity (TIBC) level, and a serum ferritin level are needed to make a more final diagnosis. Option 3: You would order a serum folate level if you had an elevated MCV and a normal MCHC, as this is indicative of macrocytic anemia. Option 4: Decreased MCV and MCHC are indicative of a microcytic hypochromic anemia. To make a more final diagnosis, you need to order a serum iron level, a TIBC level, and a serum ferritin level. Question 5. Maria, age 35, asks if being obese predisposes her to cancer. How do you respond? 1. “No, you have the same risk as a normal-weight individual.” 2. “You have less risk of cancer than normal-weight individuals because you have protein stores to combat mutant cells.” 3. “Yes, you have an increased risk of hormone-dependent cancers because of your obesity.” 4. “Yes, you have an increased risk because you have many more cells in all the organs of your body.” Rationales Option 1: People who are obese have an increased risk of hormone-dependent cancers because of their excess body fat. Option 2: People who are obese have an increased risk of hormone-dependent cancers because of their excess body fat. Option 3: People who are obese have an increased risk of hormone-dependent cancers because of their excess body fat. Because sex hormones are synthesized from fat, these people have excessive amounts of the hormones that feed hormone-dependent malignancies such as cancer of the breast, bowel, ovary, endometrium, and prostate. Option 4: While the obese patient does have more cells in their body, they are at increased risk of hormone-dependent cancers because of excess body fat, not because of an excessive number of cells. Frank, a 66-year-old white male who is on diuretic therapy, presents with an elevated hematocrit. He also has splenomegaly on examination as well as subjective complaints of blurred vision, fatigue, headache, and tinnitus. You suspect: 1. Multiple myeloma. 2. Waldenström macroglobulinemia. 3. Dehydration related to the use of diuretics. 4. Polycythemia vera. Rationales Option 1: In multiple myeloma, there is a malignancy of the plasma cells that is not typically characterized by an elevated hematocrit. Option 2: Waldenström macroglobulinemia is a malignant disorder of B cells that appears to be a hybrid of lymphocytes and plasma cells; these cells characteristically secrete an IgM paraprotein, and the disorder does not manifest itself with an elevated hematocrit. Option 3: An elevated hematocrit due to contracted plasma volume may be due to diuretic use. However, the associated signs and symptoms of splenomegaly, blurred vision, fatigue, headache, and tinnitus lead you to suspect polycythemia vera. Option 4: An elevated hematocrit due to contracted plasma volume, rather than increased red blood cell mass, may be due to diuretic use or may occur without obvious cause. However, the associated signs and symptoms of splenomegaly, blurred vision, fatigue, headache, and tinnitus lead you to suspect polycythemia vera. An elevated hematocrit due to contracted plasma volume is often called spurious polycythemia, and a number of conditions, such as hypoxia and high-altitude exposure, can cause a secondary polycythemia, but splenomegaly is absent in these cases. Primary polycythemia vera is an acquired myeloproliferative disorder that causes an overproduction of all 3 hematopoietic cell lines. Question 8. Joan had a modified radical mastectomy with radiation therapy 10 years ago. She asks when she can have her blood pressure and needle sticks performed in the affected arm. How do you respond? 1. “If it’s been 10 years and you’ve had no problems, you can discontinue these precautions.” 2. “Because you didn’t have a radical mastectomy, you can do those things now.” 3. “You must observe these precautions forever.” 4. “As long as you do limb exercises and have established collateral drainage, you can discontinue these precautions.” Rationales Option 1: Joan must observe these precautions forever because lymphedema may occur many years after a mastectomy or radiation therapy on the affected side. Option 2: Joan must observe these precautions forever because lymphedema may occur many years after a mastectomy (whether radical or modified) or radiation therapy on the affected side. Option 3: Lymphedema may occur many years after a mastectomy (whether radical or modified) or radiation therapy on the affected side. Procedures such as venipuncture and blood pressure measurements should never be done on the affected arm because there is a greater risk of infection and compromised wound healing in that limb. About 15% to 20% of women develop lymphedema after treatment for breast cancer by surgery or radiation—some not until many years later. Several interventions have been tried with lymphedema (ranging from nothing to aggressive surgical procedures) and have met with limited success. Most interventions include elevation, exercises, and pneumatic compression devices. Option 4: Joan must observe these precautions forever because lymphedema may occur many years after a mastectomy or radiation therapy on the affected side. A 28-year-old female presents to your primary care office for an annual physical. She denies any complaints other than heavy menstrual bleeding, which is something she has experienced her whole life. On routine complete blood count (CBC) evaluation, her hemoglobin is 11.0. Other pertinent values include a decreased serum ferritin and elevated total iron-binding capacity (TIBC). Her mean corpuscular volume (MCV) is 75. Which of the following is the likely cause of her anemia? 1. Iron deficiency anemia. 2. Thalassemia. 3. Sideroblastic anemia. 4. Anemia of chronic disease (ACD). Rationales Option 1: This patient’s lab values point toward microcytic anemia. With low iron stores and high ability to bind iron, iron deficiency anemia is the best choice. Option 2: Thalassemia is diagnosed via electrophoresis of a blood sample as well as a CBC. Option 3: A Prussian blue stain would need to be done in order to diagnose sideroblastic anemia. Option 4: The difference between iron deficiency anemia and ACD is that ACD can also be normocytic and would have a low TIBC. Question 19. A male patient presents to the emergency department complaining of severe knee pain. He admits to social alcohol use and a diet consisting of meat and fish. He states he has had no trauma to his knee but the pain has been getting worse for 4 days. He notes warmth and erythema of the knee as well as decreased range of motion. The patient’s x-ray is negative. You aspirate the patient’s knee, and the fluid that is aspirated is cloudy and thick. What would you see in the joint aspirate to make a diagnosis of gout? 1. A white blood cell count of 90 000. 2. Negatively birefringent, needle-shaped crystals. 3. Positively birefringent, needle-shaped crystals. 4. Positive Lyme serology. Rationales Option 1: This is consistent with a diagnosis of septic arthritis, not gout. Option 2: This presentation is consistent with a diagnosis of gout. Option 3: Pseudogout, which is a possibility given this history, has positively birefringent, rhomboid-shaped crystals. Option 4: Lyme disease can cause a joint effusion—typically in the knee—but the fluid that is aspirated is generally light green in color. Question 20. A 30-year-old male nurse presents to the emergency department. He was stuck with a needle yesterday while working and found out today that the patient involved in the needle stick is human immunodeficiency virus (HIV) positive. What is the time window for starting postexposure prophylaxis to help prevent the transmission of HIV? 1. This nurse is already past the window for starting postexposure prophylaxis. 2. 24 hours. 3. 48 hours. 4. 72 hours. Rationales Option 1: Postexposure prophylaxis needs to be started within 72 hours of exposure to HIV in order to be effective in reducing the risk of HIV transmission. Option 2: Postexposure prophylaxis needs to be started within 72 hours of exposure to HIV in order to be effective in reducing the risk of HIV transmission. Option 3: Postexposure prophylaxis needs to be started within 72 hours of exposure to HIV in order to be effective in reducing the risk of HIV transmission. Option 4: Postexposure prophylaxis needs to be started within 72 hours of exposure to HIV in order to be effective in reducing the risk of HIV transmission. Question 2. Caroline, 65, is homeless and has iron deficiency anemia. She smokes and drinks when she can and has a stomach ulcer. Which of the following is not one of her risk factors for iron deficiency anemia? 1. Smoking. 2. Poverty. 3. Ulcer. 4. Age greater than 60. Rationales Option 1: Smoking is not one of the risk factors for iron deficiency anemia. The risk of iron deficiency anemia increases in those older than 60, those who live in poverty, and those with a recent illness, such as ulcer, diverticulitis, colitis, hemorrhoids, and gastrointestinal tumors. Iron supplements should be taken. Option 2: The risk of iron deficiency anemia increases in people who live in poverty. Option 3: The risk of iron deficiency anemia increases in people with ulcers. Option 4: The risk of iron deficiency anemia increases in people older than 60. Question 8. Which of the following diagnostic tests confirms the diagnosis of sideroblastic anemia? 1. Prussian blue stain. 2. Complete blood count (CBC). 3. Red blood cell electrophoresis. 4. Erythrocyte sedimentation rate (ESR). Rationales Option 1: The Prussian blue stain is the diagnostic test used to identify sideroblastic anemia. Ringed sideroblasts would be seen in this stain. Option 2: A CBC is a general blood test used for the diagnosis of platelet disorders, disorders of the white blood cells, and anemia in general. Option 3: Red blood cell electrophoresis can help diagnose thalassemia. Option 4: ESR is a general marker of inflammation that is normally used in rheumatology. Question 10. A 15-year-old male presents to your office complaining of fatigue, shortness of breath, and joint pain for 2 weeks. The parent has blamed these symptoms on a “growth spurt.” On physical exam, you notice multiple areas of bruising. His throat is not erythematous, and no exudates are seen. He has palpable splenomegaly, and his rapid strep and mono tests are negative. Which of the following is the most likely diagnosis? 1. Acute lymphocytic leukemia (ALL). 2. Chronic myelogenous leukemia (CML). 3. Acute myelogenous leukemia (AML). 4. Chronic lymphocytic leukemia (CLL). Rationales Option 1: ALL is the most common form of pediatric leukemia. Option 2: CML is seen most commonly in middle-aged adults. Option 3: AML is seen most commonly in adults over the age of 40. Option 4: CLL is seen most commonly in adults over the age of 60. [Show More]

Last updated: 1 year ago

Preview 1 out of 112 pages

Reviews( 0 )

$20.50

Add to cart

Instant download

Can't find what you want? Try our AI powered Search

OR

GET ASSIGNMENT HELP
109
0

Document information


Connected school, study & course


About the document


Uploaded On

Mar 02, 2021

Number of pages

112

Written in

Seller


seller-icon
Solution101

Member since 3 years

15 Documents Sold


Additional information

This document has been written for:

Uploaded

Mar 02, 2021

Downloads

 0

Views

 109

Document Keyword Tags

Recommended For You


$20.50
What is Browsegrades

In Browsegrades, a student can earn by offering help to other student. Students can help other students with materials by upploading their notes and earn money.

We are here to help

We're available through e-mail, Twitter, Facebook, and live chat.
 FAQ
 Questions? Leave a message!

Follow us on
 Twitter

Copyright © Browsegrades · High quality services·